Anda di halaman 1dari 134

Universidad La Salle. Facultad Mexicana de Medicina.

Curso de Extensin Universitaria para la Preparacin del Examen Nacional para Aspirantes a Residencias Mdicas. Examen Diagnstico.

Nombre: Examen Diagnstico. Nmero de intentos: 3. Vigencia: 1 de Febrero del 2012. Horario: 7:00 p.m. a 9:00 p.m. Programar aleatorio. (5 bloques con 20 preguntas cada uno).

1.- Femenino que presenta insuficiencia suprarrenal primaria en ste caso podemos encontrar las siguientes alteraciones de laboratorio: a) Hipercalcemia e hipernatremia b) Hipernatremia e hiperglucemia c) Hipocalcemia y leucopenia d) Hipercalemia e hipoglucemia

En la insuficiencia suprarrenal primaria se observa disminucin en los niveles sricos de sodio, cloruro y bicarbonato, mientras que el potasio srico se eleva. Esta se debe a los efectos combinados de dficit de aldosterona, reduccin del filtrado glomerular y acidosis. Y sobre todo durante el estrs puede haber disminucin en los niveles de glucosa.

1.

Datos de laboratorio

Hiponatremia: Los niveles sricos bajos de sodio se debe a su prdida por la orina por dficit de aldosterona y al desplazamiento del sodio hacia el compartimento intracelular. Esta prdida de sodio extravascular reduce el volumen plasmtico y acenta la hipotensin. 2. Hiperkalemia: Aumento de los niveles sricos de potasio. Se debe a los efectos combinados del dficit de aldosterona, la reduccin del filtrado glomerular y la acidosis. 3. Hipocortisolemia: Los niveles de cortisol y aldosterona son bajos y no aumentan con la administracin de ACTH. 4. Hipercalcemia: Aumento de los niveles sricos de calcio. Ocurre en un 10-20% de los pacientes de causa desconocida.

5. Cambios electrocardiogrficos: Suelen ser inespecficos, aunque con ldentificacin generalizada del trazado. 6. Hemograma: Puede haber anemia normoctica, linfocitosis relativa y eosinofilia moderada. 7. Prueba de estimulacin de ACTH: Prueba principal que confirma el diagnstico de insuficiencia suprarrenal, al evaluar la capacidad de las suprarrenales para producir esteroides, que suelen estar ausentes o disminuidos tanto en sangre como en orina tras la estimulacin de ACTH. 8. Determinacin de la ACTH: En la insuficiencia suprarrenal primaria o Enfermedad de Addison, la ACTH y sus pptidos afines, estn elevados en plasma ante la prdida del mecanismo de retroalimentacin del eje hipotlamo-hipfisariosuprarrenal. 9. Hipertermia: La hormona del hipotlamo no controla la homeostasis.

Williams GH, Dluhy RG. Enfermedades de la corteza suprarrenal. En Jameson JL (ed.) Harrison. Endocrinologa. 1a ed. Madrid. MacGraw-Hill Espaa, 2006: 137-138.

2.- Un nio de 7 meses presenta fiebre alta desde hace 3 das, acompaada de hiperemia farngea. La fiebre cede al cuarto da de la enfermedad, momento en que aparece un exantema morbiliforme, que desaparece en un plazo de 3 das. El diagnstico ms probable es:

a) b) c) d)

Rubola Reaccin medicamentosa a antitrmicos Infeccin por enterovirus Infeccin por herpes virus humano 6

Nos encontramos ante un caso de exantema sbito tpico (tambin llamado rosola infantil o 6 enfermedad). Se trata de una enfermedad exantemtica, de mayor incidencia entre los 6 y 12 meses (como el caso de la pregunta) y que se presenta con una frecuencia del 90% antes de los aos de vida. No muestra preferencia por ningn sexo y es algo ms frecuente al final de la primavera y principios del verano. En el 80-92% de los casos de exantema sbito el agente etiolgico es el Herpes Virus Humano. Lo ms caracterstico de este perodo febril es la ausencia de hallazgos fsicos suficientes para explicar la fiebre, as como el buen estado general del lactante a pesar de la misma. Al tercer o cuarto da cede la fiebre y aparece una erupcin maculosa o maculopapulosa que comienza en tronco y se extiende a los brazos y al cuello. Tambin afecta algo a piernas y

cara. La erupcin desaparece en unos 3 das, normalmente no dejando descamacin ni despigmentacin residual. El diagnstico de este proceso es fundamentalmente clnico, por la edad y la secuencia de las manifestaciones clnicas. Es caracterstico una BHC realizada en 24 -36h puede mostrar leucocitosis con neutrofilia, pero ms all de 48h aparece el patrn tpico de leucopenia con neutropenia absoluta y linfocitosis relativa.

Nelson, Tratado de Pediatra 15 Ed., pgs. 1120-21

3.- Madre primigesta de 28 aos con control prenatal regular, cursa con ruptura de membranas a las 30 semanas de gestacin, se interrumpe el embarazo por cesrea. Es probable que el recin nacido prematuro disminuya la posibilidad de presentar enfermedad de membranas hialinas por la siguiente medicacin materna: a) Oxitocina IV b) Sulfato de magnesio IV c) Antibioticos IV d) Esteroides IM

Uso prenatal, un pilar en neonatologa Por su efecto en la maduracin fetal, los corticoesteroides se han utilizado prenatalmente desde hace ms de tres dcadas. Liggins y Howie en 1972, reportaron por primera vez los efectos benficos de los esteroides, administrados prenatalmente, en la maduracin pulmonar y en la disminucin de la incidencia de SDR. Desde ese primer reporte a la fecha, se han descrito otros efectos positivos. Su uso "adecuado" y oportuno ha demostrado tener beneficio en el recin nacido pretrmino. Ahora se sabe que la administracin de esteroides a la madre embarazada se ha asociado a disminucin en la incidencia de SDR, hemorragia intraventricular, displasia broncopulmonar y mortalidad neonatal. Se ha observado que el mximo beneficio en el recin nacido se obtiene de 24 horas a siete das despus de la administracin materna de esteroides. Sin embargo, an antes de 24 horas de administrados, se ha reportado beneficio. Aunque los cambios bioqumicos generados con los esteroides desaparecen despus de siete das, los cambios estructurales persisten. Algunos estudios clnicos han demostrado efecto benfico de esteroides an despus de siete das de administrados. Un metanlisis de 12 estudios, realizado por Crowley y col. en 1990, demostr que el uso prenatal de esteroides disminua la incidencia de SDR en aproximadamente 50%. Es claro que el efecto ms importante se encuentra en los fetos menores de 34 semanas de edad gestacional; sin embargo, tambin se ha demostrado disminucin en la incidencia de SDR en neonatos de mayor edad gestacional.18

Probablemente el efecto benfico ms importante asociado al uso prenatal de esteroides sea la disminucin en la mortalidad neonatal. Crowley y Col demostraron en el metanlisis, con ms de 3, 000 neonatos, que sta disminua considerablemente (razn de momios 0.59, intervalo de confianza =0.470.75). La disminucin en mortalidad se ha detectado an en pacientes menores de 800 g.

Referencia bibliogrfica: Bol. Med. Hosp. Infant. Mex. v.62 n.5 Mxico sep. /oct. 2005. Esteroides en Neonatologa: Entusiasmo, uso, abuso y desuso. Dnde est el justo medio? Steroids in neonatology: Enthusiasm, use, abuse and disuse. Is there an equilibrium?

Liggins GC, Howie RN.A controlled trial of antepartum glucocorticoid treatment for prevention of the respiratory distress syndrome in premature infants Pediatrics, 1972; 50: 51525. Crowley P,Chalmers I, Keirse MJNC. The effects of corticosteroid administration before preterm delivery: An overview of the evidence from controlled trials. Br. J Obstet Gynaecol, 1990; 97: 1125. Van Marter LJ, Leviton A, Kuban KCK. Maternal glucocorticoid therapy and reduced risk of bronchopulmonary dysplasia. Pediatrics, 1990; 86: 331 6.

Morales WJ, Diebel D, Lazar AJ, Zadrozny D.The effect of antenatal dexamethasone administration on the prevention of respiratory distress syndrome in preterm gestations with premature rupture of membranes. Am J Obstet Gynecol 1986; 154:5915. Gamsu HR, Mullinger BM, Donnai P, Dash CH. Antenatal administration of betamethasone to prevent respiratory distress syndrome in preterm infants: Report of a UK multicentre trial. Br J Obstet Gynaecol. 1989; 96:40110. Young BK, Klein SA, Katz M. Intravenous dexamethasone for prevention of neonatal respiratory distress: A prospective controlled study. Am J Obstet Gynecol. 1980; 138:2039.

Doyle LW, Permezel MJ, Kitchen WH. Is there a lower limit for birth weight/gestational age and antenatal steroid therapy? Aust NZ J Obstet Gynaecol. 1992; 32: 1935.

4.- Paciente de 50 aos, con mioma uterino de tamao equivalente a una gestacin de 12 semanas, que presenta hipermenorreas y hemoglobinemia de 9 gr%. No se demuestra patologa asociada. Se encuentra en espera para la prctica de una histerectoma programada a realizar en 4 meses. En esta paciente est indicado el tratamiento preoperatorio con: a) Estrgenos. b) Inhibidores de la fibrinlisis. c) Anlogos de la GnRH. d) Derivados del cornezuelo del centeno.

Anlogos de la GnRH: Son derivados de la hormona GnRH en donde se ha realizado una sustitucin peptdica en posicin 6 y en algunos casos en la 10, obteniendo compuestos hasta unas 200 veces ms potentes debido a mayor afinidad por los receptores y a su resistencia a la degradacin por peptidasas. Aunque su accin inicial produce un incremento en la produccin de FSH-LH (efecto flure-up o llamarada) tras 56 das de exposicin contnua, los receptores son internalizados producindose un estado de hipogonadismo hipogonadotropo y niveles de estradiol similares a los de la postmenopausia. Los anlogos de GnRH estn disponibles en distintas frmulas: administracin nasal (varias aplicaciones al da), subcutnea (aplicacin diaria) o intramuscular (preparados depot mensuales o trimestrales) (Shaw RW 1999). Marco Filicori y sus colaboradores de la Universidad de Bolonia fueron los primeros en utilizar en 1983 los aGnRH en un estudio que confirm su eficacia para reducir el tamao de los miomas uterinos y secundariamente sntomas como alteraciones menstruales, dolor plvico y sntomas de presin local. Otros autores como Minaguchi H y colaboradores continan comprobando la efectividad de los anlogos de la GnRH en el tratamiento del mioma uterino tras evaluar en el ao 2000 seis estudios con un total de 602 pacientes tratadas con nafarelina. La disminucin del tamao se calcula entre un 30-70%, y se ha observado como el mayor porcentaje de reduccin ocurre tras el primer mes de tratamiento, no existiendo reducciones o siendo stas mnimas despus del tercer mes (Healy et al 1986; Friedman et al 1989; Matta et al 1989; Williams y Shaw 1990). En miomas pediculados o con gran proporcin de calcio o colgeno (hialinizacin) la repuesta es tambin menor. Debe tenerse en cuenta que si despus de dos meses de tratamiento no se ha producido un significativo descenso del tamao del mioma, ste ya no debe ser esperado y debe pensarse en la posibilidad de la existencia de un tumor muscular maligno no diagnosticado (Messia AF et al 1998). Tras finalizar el tratamiento y recuperarse el estado de hipogonadismo, el mioma retorna rpidamente a su tamao inicial (Friedman AJ et al 1987; Matta WH et al 1989).

En casos prximos a la menopausia, la reduccin del tamao del mioma y su sintomatologa, permitira hablar de una solucin mdica del problema, pero en todo caso la utilizacin de aGnRH facilitara la intervencin quirrgica al acortar el tiempo de intervencin, la hemorragia y el acceso a localizaciones complicadas como el caso de miomas interligamentarios o situados en istmo o crvix. En el caso de la ciruga histeroscpica la reduccin del tiempo de ciruga permitira reducir el volumen de fluidos aportado a cavidad uterina y los riegos de absorcin e hiponatremia. Los mecanismos de accin por los que los aGnRH actan son: o Hipoestrogenemia: Es necesario mantener la hipoestrogenemia, pues la elevacin de sus niveles lleva a un rpido incremento del tamao del mioma. El crecimiento del mioma es dependiente de los niveles de estrgenos (aumentan de tamao con el embarazo y se reducen durante la menopausia o el tratamiento con aGnRH, pudiendo volver a crecer durante la THS), pero aunque los estrgenos parecen ser importantes en el crecimiento del mioma, su relacin debe ser algo ms compleja pues no se han descrito incrementos significativos del tamao de miomas durante el tratamiento con gonadotrofinas en RA (situaciones con elevados niveles de E2), algunos de ellos no se modifican durante el embarazo o incluso decrecen y se han encontrado crecimientos despus del tratamiento con citrato de clomifeno (antiestrgeno). En relacin con la hipoestrogenemia podran estar los cambios inducidos en el flujo vascular uterino (incrementos en el ndice de resistencia de las arterias uterinas) que suponen una reduccin de la vascularizacin o las modificaciones de distintos factores de crecimiento. Cambios histolgicos: El tratamiento con aGnRH puede producir degeneracin roja, infiltracin linfocitaria, y necrosis, as como reduccin de la proliferacin celular e incremento de la apoptosis. Pero en otras circunstancias no es posible encontrar 7 diferencias. No se ha encontrado una relacin entre los cambios histolgicos y el porcentaje de reduccin del tamao del tero, y existe una gran variabilidad entre distintas pacientes o entre distintos miomas de una misma paciente, no existiendo pues un patrn histolgico caracterstico de respuesta ante el tratamiento con aGnRH. BIBLIOGRAFA 1. Abad L, Abad de Velasco L, Parilla JJ. Etiopatogenia. Papel de las hormonas esteroideas, factores de crecimiento y otras sustancias. Cuad Med Reprod 1999;5(1):15-29. 2. Albano C, Platteau P, Devroey P. Gonadotropin-releasing hormone antagonist: how good is the new hope? Curr Opin Obstet Gynecol 2001;13(3):257-62. 3. Coutinho EM.Treatment of large fibroids with high doses of gestrinone. Gynecol Obstet Invest 1990;30(1):44-47. 4. Chavez NF, Stewart EA. Medical treatment of uterine fibroids. Clin Obstet Gynecol 2001;44(2):327-84. 5. De Leo V, la Marca A, Morgante G. Shortterm treatment of uterine fibromyomas with danazol. Gynecol Obstet Invest 1999;47(4):258-262.

6. Eldar-Geva T, Healy DL. Other medical management of uterine fibroids. Baillieres Clin Obstet Gynaecol 1998;12(2):269-88. 7. Felberbaum RE, Germer U, Ludwig M, Riethmuller-Winzen H, Heise S, Buttge I, Bauer O, Reissmann T, Engel J, Diedrich K. Treatment of uterine fibroids with a slow-release formulation of the gonadotrophin releasing hormone antagonist Cetrorelix.HumReprod 1998;13(6):1660-8.

5.- Femenino que cursa con 36. 5 semanas de gestacin acude al servicio por referir malestar general, fosfenos, nausea y vmito, aprecia moderada ictericia, usted sospecha de un sndrome de HELLP Que alteraciones de laboratorio espera encontrar al confirmar el diagnstico? a) b) c) d) Anemia Hemoltica, trombocitopenia, enzimas hepticas elevadas. Trombocitosis, enzimas heptica elevadas, anemia hemoltica. Anemia hemolitica, trombocitosis, fosfatasa alcalina elevada. Trombocitopenia, Leucopenia, hipertensin arterial.

DEFINICIN: Es una complicacin de la preeclampsia en la cual adems de la Hipertensin Arterial y proteinuria hay presencia de anemia hemoltica, enzimas hepticas elevadas y recuento bajo de plaquetas.

EPIDEMIOLOGIA: Se presenta en un 4 a 10% de las preeclmpticas, diagnosticndose anteparto en un 70% de los casos preferentemente antes de las 37 semanas, mientras que el 30% de los casos restantes enferma en los primeros 7 das del puerperio, sobre todo en las 48 hrs. iniciales. La proteinuria e hipertensin pueden estar ausentes en un 15 al 20% de los casos. Incidencia mayor en multigestantes y en edades avanzadas. Ocurre ms frecuentemente cuando se demora la salida del feto y cuando se presenta desprendimiento de la placenta. Mortalidad materna del 24% y mortalidad perinatal del 30-40%.

CLASIFICACION: Sndrome de HELLP. Clasificacin de Mississipi. CLASE 1 2 Plaquetopenia Severa <50000 Moderada >50000 <100000 Ligera >100000 <150000 >150000 LDH >600 IU/L >600 IU/L AST-ALT >70 IU/L >70 IU/L

3 PE severa Eclampsia (sin HELLP)

>600 IU/L <400 IU/L

>40 IU/L <70 IU/L <40IU/L

MANIFESTACIONES CLINICAS: Malestar general, fatiga y molestias inespecficas 90% Cefalea 70% Epigastralgia 64% Vmito 22% Fosfenos 15% Visin Borrosa 11% Acfenos 3% Ictericia Anemia no explicada Oliguria

Si se aade una HEMORRAGIA HEPTICA, el paciente puede quejarse de dolor en el HOMBRO DERECHO y EL CUELLO, adems de las molestias abdominales. Equimosis en los sitios de punciones venosas, petequias en los sitios de presin del brazo, pero pueden tener pruebas de Rumpel Leed negativas.

En casos severos se pude presentar ascitis como causa de hipertensin portal.

DIAGNOSTICO: El diagnstico clnico del sndrome de HELLP se plantea en gestantes o purperas con preeclampsia severa-eclampsia, excepto en el 15-20%, en las cuales esta asociacin no puede ser demostrada, en tanto se cumplan los criterios de Sibai:

MANIFESTACIONES CLINICAS: Malestar general, fatiga y molestias inespecficas 90% Cefalea 70% Epigastralgia 64% Vmito 22% Fosfenos 15% Visin Borrosa 11% Acfenos 3% Ictericia Anemia no explicada Oliguria

Si se aade una HEMORRAGIA HEPTICA, el paciente puede quejarse de dolor en el HOMBRO DERECHO y EL CUELLO, adems de las molestias abdominales. Equimosis en los sitios de punciones venosas, petequias en los sitios de presin del brazo, pero pueden tener pruebas de Rumpel Leed negativas. En casos severos se pude presentar ascitis como causa de hipertensin portal.

DIAGNOSTICO: El diagnstico clnico del sndrome de HELLP se plantea en gestantes o purperas con preeclampsia severa-eclampsia, excepto en el 15-20%, en las cuales esta asociacin no puede ser demostrada, en tanto se cumplan los criterios de Sibai:

HEMOLISIS Frotis perifrico anormal (eritrocitos fragmentados) Hematocrito (>24%) Bilirrubina indirecta (>1.2mg/dL) Deshidrogenasa lctica (>218 UI/L)

ENZIMAS HEPTICAS ELEVADAS

LDH >218UI/L AST >30UI/L ALT >37UI/L

PLAQUETAS BAJAS <100.000/mm3

BIBLIOGRAFIA: Sibai baha, El sndrome HELLP. Universidad de Valencia, revista quincenal de Obstetricia clnica y ginecologa, Octubre 2003. V. Cararach, Sndrome de HELLP y Repercusiones maternas. X curso intensivo de formacin continuada materno fetal. Enero de 2003. Toirac, Abelardo. Sndrome de Weistein HELLP Hospital Ginecoobstetrico Tamara Bunke. Junio 2002

De la Fuente, David. Sndrome HELLP. Medicina Universitria 2003; 5 (19): 101 -9 Andrea G. Witlin, DO, Baha M. Sibai, MD. Diagnosis and Management of women with Hemolysis Elevate Liver Enzymes, and Pletelet Count (HELLP) syndrome. Hospital Physician. Febrero 1999. CIFUENTES B, Rodrigo. Ginecologa y obstetricia basadas en las evidencias. Bogot: Distribuna, 2006. Sexta edicin. 447 - 283 p.

6.- En su jurisdiccin se reportaron 44 casos de hepatitis A entre los alumnos de segundo grado de una escuela secundaria durante la primera quincena del mes de septiembre. La tasa de ataque es de 26.4%. Esta situacin nos indica que estamos ante un(a): a) Brote b) Endemia c) Epidemia d) Pandemia

De acuerdo a la NOM- 017 un brote se define como la ocurrencia de dos o ms casos asociados epidemiolgicamente (tiempo, lugar y persona) entre s. La medida cuantitativa de la extensin de un brote es la Tasa de Ataque (TA) que se calcula dividiendo el nmero de casos nuevos entre el total de personas expuestas por 100. Greenberg R. S; Epidemiologa mdica, Manual Moderno, 2. Ed. Pgs. 77-79.

7.- Se presenta ante usted femenino de 34 aos con referencia por diagnstico de lupus eritematoso generalizado, Cul es el sntoma dentro de los criterios de clasificacin de de ste padecimiento? a) Alopecia b) Fotosensibilidad c) Artralgias d) Prdida de peso Criterios de Clasificacin para el Diagnstico de Lupus Eritematoso Sistmico (LES) Erupcin malar: Eritema fijo, plano o alto, sobre las eminencias malares, que no suele afectar los surcos nasogenianos. Erupcin discoide: Placas eritematosas altas, con descamacin queratsica adherente y tapones foliculares; puede haber cicatrices atrficas en las lesiones ms antiguas. Fotosensibilidad: Erupcin cutnea a causa de una reaccin inslita a la luz solar, referida por el paciente u observada por el mdico. lceras bucales: Ulceracin nasofarngea, por lo comn indolora, observada por un mdico.

Artritis: Artritis no erosiva que afecta dos o ms articulaciones perifricas, caracterizada por dolor a la palpacin, tumefaccin o derrame. Serositis: Pleuritis o pericarditis documentada por electrocardiograma o frote o evidencia de derrame pericrdico. Enfermedad renal: Proteinuria persistente mayor a 0,5g/da o 3+ o cilindros celulares. Transtorno neurolgico: Convulsiones o psicosis en ausencia de otra causa conocida. Transtorno hematolgico: Anemia hemoltica o leucopenia (< 4.000/mm3) o linfopenia: (<1.500/mm3) o trombocitopenia (< 100.000/mm3) en ausencia de frmacos que produzcan esta alteracin. Trastorno inmunolgico: Anti-DNA, anti-Sm, y/o Anticuerpos antifosofolipdicos (AFL). Anticuerpo antinuclear: Un ttulo anormal de ANA por inmunofluorescencia o anlisis equivalente en cualquier momento y en ausencia de medicamentos relacionados con el sndrome de lupus de origen farmacolgico. Cualquier combinacin de 4 o ms de los 11 criterios, bien documentado durante cualquier intervalo de la historia del paciente, hace el diagnsticos de LES (especificidad y sensibilidad son del 95% y 75% respectivamente).

Petri M. Review of classification criteria for systemic lupus erythematosus. Rheum Dis Clin North Am. 2005 May;31(2):245-54.

8.- Femenino que cursa con diagnstico de Salmonella Typhy. cultivos permite aislar con mayor frecuencia ste agente causal?

Cul de los siguientes

a) b) c) d)

Hemocultivo Coprocultivo Mielocultivo Urocultivo

DIAGNOSTICO DE LABORATORIO Dadas las variadas manifestaciones clnicas de las salmonelosis, la confirmacin del diagnstico de estas infecciones, requiere de mtodos microbiolgicos que permitan el aislamiento o identificacin del agente causal o de pruebas serolgicas que facilitan reconocer anticuerpos especficos presentes en el suero de los pacientes. Hemocultivo: Es el procedimiento de eleccin, cuando se realiza apropiadamente y en medios selectivos a base de bilis. Coincidiendo con la fisiopatologa de la infeccin, son positivos especialmente durante la primera semana de la infeccin; se calcula que al final de la tercera semana de positividad solamente alcanza un 50%. Mielocultivo: el cultivo del aspirado de mdula sea se considera como el mejor mtodo para el aislamiento de salmonella en los pacientes con fiebre tifoidea y paratifoidea. Aunque el procedimiento produce una molestia transitoria, en general es bien tolerado y los

cultivos son ms rpidamente positivos. Se recomienda sea practicado por personal con experiencia. Pueden ser positivos an cuando los hemocultivos sean negativos. Coprocultivo: Puede ser positivo desde el comienzo de la infeccin, aunque su mxima positividad en la infeccin aguda, se observa durante la tercera semana. Es particularmente til para el control postratamiento de los pacientes y para detectar los portadores crnicos. Cultivo de bilis duodenal: Obtenido por aspiracin o utilizando la tcnica que lleva un dispositivo en cpsulas de gelatina. No es superior al hemocultivo y con certeza no supera a la asociacin del hemocultivo con el coprocultivo. Urocultivo: Su valor diagnstico es muy limitado pues la bacteriuria no es continua. Su mxima positividad est en la tercera semana. La Salmonella tambin puede ser aislada de otros productos como las manchas rosadas o reoseolas tficas, de la secrecin bronquial, del lquido articular, etc.

DIAGNOSTICO SEROLOGICO Reaccin de seroaglutinacin (Widal): Es de poco valor como prueba diagnstica. En la infeccin no tratada slo cerca del 50% de los pacientes pueden tener un aumento significativo de las aglutininas contra el antgeno "O", en algn momento de la enfermedad. Las aglutininas contra el antgeno "H" no tienen valor diagnstico aunque puedan observarse ttulos elevados de ellas. En muchos casos de fiebre tifoidea no hay elevacin de los ttulos de aglutininas durante el curso de la infeccin y en ocasiones se pueden observar elevaciones no especficas, debido a reacciones cruzadas. Diagnstico inmunoenzimtico: La deteccin de anticuerpos IgM e IgG contra el lipopolisacarido por tcnica ELISA an no est disponible para uso rutinario. Con fines de investigacin se han utilizado otras pruebas dentro de las cuales estn la reaccin de polimerasa en cadena (PCR), las pruebas de fagotipificacin, las de susceptibilidad antimicrobiana y la investigacin del perfil plasmdico de algunas cepas. En los estudios epidemiolgicos se usan las pruebas de fagotipificacin, de susceptibilidad contra los antimicrobianos y el perfil plasmdico, las cuales han demostrado ser tiles y complementarias para el estudio de cepas aisladas de alimentos, o de aguas contaminadas, y en brotes de salmonelosis en los cuales se requiere establecer una fuente comn de infeccin.

LECTURAS RECOMENDADAS 1. Borrego JJ, Castro D, Jimnez Notorio M, Luque A, Rodrguez Avial C, Picazo JJ. Comparison of epidemiological markers of salmonella strains isolated from differents sources in Spain. J Clin Microbiol 30: 3058, 1992. De Los Rios O, Restrepo J, Carvajal CD. Salmonelosis: Revisin de conceptos. Comportamiento epidemiolgico en Antioquia. Bol Epidenmol Antioquia 7:19. Edelman R, Levine MM. Sumary of an international workshop on typhoid fever. Rev. Inf Dis 8:329, 1996. Prada G. Infecciones por especies de Salmonella. En: Medicina Interna. Chalem, Escandn, Campos, Esguerra editores. Fundacin Instituto de Reumatologa e Inmunologa. Editorial Presencia. Santaf de Bogot. Tacket C. Molecular epidemiology of salmonella. Epidemiologic Reviews 11:99.

2. 3. 4.

5.

9.- Masculino de 77 aos portador de virus C y diabtico desde hace 10 aos, que presenta lesiones ampollosas de contenido serohemorrgico en extremidades superiores e inferiores, y erosiones a nivel de la mucosa bucal. La biopsia cutnea muestra una ampolla subepidrmica, se realiza inmunofluorescencia directa que muestra la presencia de depsitos de IgG a nivel de la membrana basal. El diagnstico ms probable es: a) Dermatitis herpetiforme. b) Penfigoide ampolloso. c) Toxicodermia ampollosa. d) Porfiria cutnea tarda.

Penfigoide ampolloso Casi siempre afecta a mayores de 60 aos. Se trata de una enfermedad ampollosa crnica autoinmune causada por la unin de autoanticuerpos tipo IgG y complemento C3 al antgeno del penfigoide, una glucoproteina transmembrana de 180 (antgeno menor) o 230 kD (antgeno mayor, desmoplaquina I) situada en los hemidesmosomas del queratinocito basal, a nivel de la lmina lcida. Aunque se haba dicho que el penfigoide poda ser paraneoplsico, existen estudios epidemiolgicos que demuestran que el incremento de carcinomas asociados se debe unicamente a que ambas patologas, el penfigoide y el cncer, se dan con mayor frecuencia al mismo grupo de edad.

Patognesis Se cree que la unin de los auto anticuerpos IgG al antgeno del penfigoide produce activacin del complemento, lo que da lugar a la sntesis de C3a y C5a que causan desgranulacin de los mastocitos. stos a su vez liberan mediadores entre los que destaca el factor quimiotctico de los eosinfilos. Los eosinfilos reclutados liberan enzimas lisosomales que destruyen los hemidesmosomas y los filamentos de anclaje con lo que se separa la unin dermoepidrmica en la lmina lcida. Clnica Se caracteriza por lesiones ampollosas grandes y tensas de contenido seroso o serohemtico, localizadas o generalizadas, con distribucin preferente en flexuras y que curan sin dejar cicatriz residual. La aparicin de ampollas puede ir precedida de un cuadro urticarial o eczematoso. Pueden ser o no pruriginosas. En cerca del 30% de los casos existe afectacin mucosa (generalmente mucosa oral).

Penfigoide ampolloso Diagnstico: El diagnstico se establece por criterios clnicos, histolgicos e inmunopatolgicos. En el estudio histolgico de la biopsia cutnea de una ampolla se observa una hendidura subepidrmica en el interior de la cual se aprecia un rico infiltrado inflamatorio mixto con predominio de eosinfilos que tambin abundan en la dermis. Mediante inmunofluorescencia directa se comprueba la existencia de depsitos lineales de IgG y/o C3 en la regin de la membrana basal. La inmunofluorescencia indirecta, utilizando fluido de la ampolla u orina, pone de relieve en cerca de la mitad de los pacientes la existencia de IgG y/o C3 circulantes.

Bibliografa:
1. Worjnarowska F, Kirtschig G, Highet AS, Venning VA, Khumalo NP. Guidelines for the management of bullous pemphigoid. Br J Dermatol 2002; 147:214-221. 2. Downham TF, Chapel TA. Bullous pemphigoid. Arch Dermatol 1978; 114:1639-1642. 3. Chosidow O, Saas V, Diquet B. Correlation between the pretreatment number of blisters and the time to control bullous pemphigoid with Prednisone 1mg/Kg/day. Br J Dermatol 1992; 127:185-195. 4. Fiveson D, Breneman D, Rosen G et al. Nicotinamide and tetracycline therapy of bullous pemphigoid. Arch Dermatol 1994; 130:753-758. 5. Fleming TE, Korman NJ. Cicatricial pemphigoid. J Am acad Dermatol 2000; 43:571-591. 6. Eisen D, Ellis CN, Voorhees JJ. Topical Cyclosporine for oral bullous disorders. J Am Acad Dermatol 1990; 23:936-937.

10.- Al realizar ultrasonografa renal usted observa una masa bien delimitada, en el que se interrumpe el contorno renal, sin presencia de ecos en su interior y refuerzo posterior, Qu diagnstico es el ms probable?

a) Absceso renal. b) Hipertrofia de columna de Bertin. c) Carcinoma renal. d) Quiste renal simple.

Los criterios para el diagnstico ultrasonogrfico de un quiste simple renal son los siguientes: 1.- Forma ovoide o esfrica 2.-Ausencia de ecos internos (quiste anecoico) 3.-Presencia de una pared delgada y lisa bien definida que lo separa del parnquima. 4.-.El refuerzo acstico ms all de la pared posterior del quiste es proporcional a su contenido lquido. 5.-Se observa una banda estrecha de forma acstica por fuera del borde externo. Si se cumplen con todos estos criterios no es necesario realizar ms estudios para diagnosticar la masa.

Sin embargo, ocasionalmente la ecografa puede no resultar la tcnica ms idnea (por ejemplo cuando hay calcificacin de la pared del quiste, ste es hemorrgico o existen mltiples quistes). -Brenner and rector The Kidney 2 volmenes 2004 Saunders 2525 pag.

11. - A 6-year-old child presents with flesh-colored papules on the hand that are not pruritic. Examination reveals lesions that are approximately 4 mm. in diameter with central umbilication. A halo is seen around those lesions undergoing regression. Which of the following is the most likely diagnosis?

a) b) c) d)

Verruca vulgaris Molluscum contagiosum Keratoacanthoma Herpetic whitlow

El Molusco Contagioso es el nombre de una infeccin viral del grupo de los Poxvirus. Es frecuente, transmisible, autoinoculable (uno mismo la puede trasmitir a diferentes partes del cuerpo), se puede curar sola y es benigna. Afecta principalmente a los nios menores de 10 aos, adultos sexualmente activos y a pacientes inmunosuprimidos (con defensas bajas) como los pacientes con SIDA, ocurriendo en un 5 18 por ciento. Afecta ms a hombres que a mujeres, y su frecuencia aumenta tambin en climas tropicales y durante el verano. CLINICA El periodo de incubacin de la infeccin es de 14 a 50 das, aunque hay datos de recin nacidos con lesiones al cabo de 7 das postparto. Las lesiones se inician como neoformaciones que miden generalmente de 2 a 6 mm. aunque pueden llegar a medir 3 cm (13,14) son hemisfricas, cupuliformes, lisas, del color de la piel o perladas, algunas (20%) tienen una umbilicacin central(15); la base es levemente eritematosa y son de consistencia firme(13,14). Se localizan en cualquier parte del cuerpo e incluso pueden afectar mucosas, generalmente se agrupan en un rea especfica, pero pueden estar diseminadas en personas infectadas con el virus de la inmunodeficiencia humana adquirida, siendo un marcador de enfermedad avanzada (16). En el caso de los nios las lesiones se localizan normalmente en cara, tronco, brazos y piernas a diferencia de los adultos jvenes en quienes el molusco contagioso se adquiere por transmisin sexual, las lesiones tienen predileccin por genitales, abdomen y cara interna de los muslos; en otras series no hay diferencia (17).

Las lesiones del molusco suelen aparecer entre los 14 das a 6 meses despus de la exposicin, hay datos de recin nacidos con lesiones al cabo de 7 das postparto. Se pueden propagar por auto inoculacin, pero es tpico que se resuelvan espontneamente en pocos meses. Las lesiones son asintomticas en la mayora de los pacientes (13,18), aunque en el 10% de los casos puede haber prurito y desarrollarse una reaccin eccematosa (14). CRITERIO DIAGNSTICO El diagnstico se hace clnicamente y en algunos casos dudosos puede efectuarse biopsia con tincin H-E donde se encuentran los cuerpos de molusco (inclusiones intracitoplasmticas grandes) o de Hendersen-Paterson; el 90% de los pacientes posee Ac tipo Ig G. Puede realizarse microscopa electrnica, PCR, Elisa, e inmunohistoqumica. REFERENCIA BIBLIOGRAFICA: 1. Schotz J, Rosen-Wolft A, Bugert J et al. Molecular epidemiology of molluscum contagiosum. J Infect Dis 1988; 158: 898-900. 2. Porter CD, Nlake NW, Archard LC et al. Molluscum contagiosum virus type in genital and non genital lesions. Br J Dermatol 1989; 120: 37-41. 3. Overfield TM, Briody JA. An epidemiologic study of molluscum contagiosum in Achorage, Alaska. J Pediatr 1966; 69: 640-42. 4. Gottlieb SL, Myskowwski PL. Moluscum contagiosum. Intern J Dermatol 1994; 33: 45361. 5. Telner P, Solomon LM. Eruptive molluscum contagiosum in atopic dermatitis. Can Med Assoc J 1966; 95: 978-79. 6. Pauly CR, Artis WM, Jones HE. Atopic dermatitis, impaired cellular immunity and molluscum contagiosum. Arch Dermatol 1978; 114: 391-93. 7. Dohil, P. Lin, J. Lee, A. Lucky, A. Paller, L The epidemiology of molluscum contagiosum in children. J Am Acad Dermatol. 2006;54:47-54. 8 Rosenberg EW, Yusk JW. Molluscum contagiosum. Eruption following treatment with prednisone and methotrexate.Arch Dermatol 1970; 101: 439-41. 9. Koopman JJ, Van Merrienboer FCJ, Vreden SGS, Dolmans WMV. Molluscum contagiosum: a marker for advanced HIV infection. Br J Dermatol 1992; 126: 528-29. 10. Schwartz JJ, Myskowski PL. HIV-related molluscum contagiosum presenting as a cutaneous hom. Int J Dermatol1992; 31: 142-44.

12.- Masculino que se presenta a consulta externa, refiere que su pareja ha sido diagnosticada por infeccin de virus del papiloma. La forma ms frecuente de la infeccin por papiloma virus en varones es? a) b) c) d) Verrugas plantares Verrugas larngeas Subclnica en genitales Condilomas prepuciales

En diferentes estudios de varones compaeros de mujeres con lesiones cervicales por virus del papiloma humano se demostr que el 88% tenan signos histolgicos de condilomas, de los cuales el 72% eran en forma subclnica es decir no haba ningn tipo de lesin visible o sntoma. Otros estudios han demostrado que hasta el 66% o mas de los compaeros de mujeres con neoplasias intraepiteliales cervicales tienen infecciones subclnicas por virus del papiloma humano en el pene.

Obstet Gynecol 2001;185:308-17. Kurtycz DFI, Hoerl HD. Thin-Layer Technology: Tempered Enthusiasm. Diag Cytopath 2000;23(1):1-5.

13.- Femenino de 39 aos de edad la cual inicia con un cuadro de exoftalmos axial de varias semanas de evolucin, con predominio en su ojo derecho. En la exploracin se aprecia una conjuntiva con sntomas discretos de hiperemia y edema, una queratitis de carcter punteado en tercio inferior corneal y se sospecha una retraccinpalpebral al observar cmo el borde del prpado superior se encuentra por encima del limbo, permitindonos visualizar la esclertica. La paciente no refiere diminucin de visin ni alteraciones tipo visin doble y toma presin intraocular con parmetros dentro de los normales. Cul de los siguientes diagnsticos le parece ms compatible con el cuadro?

a) Tumor intraorbitario. b) Enfermedad de Graves-Basedow. c) Queratoconjuntivitis epidmica. d) Tumor intraocular.

OFTALMOPATA TIROIDEA Es la causa ms frecuente de exoftalmos tanto bilateral como unilateral en adultos. La forma tpica aparece en pacientes con enfermedad de Graves-Basedow, en los que podemos encontrar exoftalmos y sndrome palpebro-retrctil. Este sndrome ocular puede aparecer en pacientes eutiroideos o hipotiroideos, pudiendo constituir el signo ms precoz de una tirotoxicosis incipiente.

CLNICA Se distinguen dos formas clnicas: a) Tirotxica (hipersensibilidad a las catecolaminas y habitualmente hay hipertiroidismo) exoftalmos moderado depresible. Hay edema del contenido orbitario, pero no fibrosis ni oftalmoparesia. b) Maligna (inflamacin orbitaria autoinmune y puede haber normo o hipotirodismo): exoftalmos irreductible severo con oftalmopejia progresiva y queratitis por exposicin. Puede afectar al nervio ptico por compresin y producir prdida visual. Hay fibrosis de la grasa y vientres musculares. Manifestaciones oculares asociadas: Retraccin palpebral bilateral que permite ver la esclera por encima del limbo, disminucin de la motilidad palpebral, alteracin de la pigmentacin palpebral, hiperemia conjuntival.

DIAGNSTICO. Por los signos clnicos descritos y exploraciones complementarias, como la exoftalmometra (medida de la protrusin ocular), la radiologa (aumento de densidad de los tejidos blandos), el engrosamiento del vientre de algunos msculos extraoculares (apreciados en la TC, la RM y la ecografa orbitaria) y la analtica sistmica.

Bibliografa: AACE Thyroid Task Force. American Association of Clinical Endocrinologists medical guidelines for clinical practice for the evaluation and treatment of hyperthyroidism and hypothyroidism. Endocr Pract. 2002;8 (6).

Davies TF, Larsen PR. Thyrotoxicosis. In: Kronenberg HM, Melmed S, Polonsky KS, Larsen PR, eds. Williams Textbook of Endocrinology. 11th ed. Philadelphia, Pa: Saunders Elsevier; 2008: Chap 11.

14.- Ante un probable infarto agudo al miocardio, Cul de los siguientes marcadores tiene mayor sensibilidad para realizar ste diagnstico? a) CPK b) Transaminasa c) Troponina d) Deshidrogenada lctica

Es muy conocida la falta de especificidad de la CPK. La elevacin de la actividad de esta enzima se produce tanto en el infarto de miocardio (IAM) como en afecciones caracterizadas por un grado variable de necrosis muscular. Por ello en los ltimos aos se han realizado, y se siguen realizando, considerables esfuerzos para encontrar nuevos marcadores analticos que sean capaces de diferenciar estos cuadros. Entre ellos, la determinacin de la isoenzima miocrdica de la CPK de forma cuantificada (CPK-MB-Masa) resulta tambin poco especfica ya que se eleva tambin en caso de necrosis de msculo estriado; est descrita su elevacin en traumatismos, rabdomiolisis, convulsiones1, miopatas agudas y crnicas2, insuficiencia renal en dilisis3 e incluso en el ejercicio intenso4,5; y lo mismo podemos afirmar de la mioglobina6. Ms recientemente se han determinado las troponinas T e I en sus isoformas especficas de msculo cardiaco, que tienen una secuencia diferente de aminocidos a las de msculo estriado no cardiaco, lo que permite el desarrollo de inmunoensayos especficos. El complejo de las troponinas T, C, e I est estrechamente unido al filamento de tropomiosina. La T se encarga de la unin a tropomiosina; la C es iniciadora de la contraccin tras unirse al calcio, y la I se llama as por ser inhibidora de la contraccin en reposo7. Esta especificidad ha sido demostrada en varios trabajos que muestran valores normales de troponina-I en procesos que cursan con necrosis de msculo estriado2,8. Adems de ser muy especficas, las troponinas son altamente sensibles en el infarto de miocardio. Mair y Col encuentran una sensibilidad del 100% para el diagnstico si se hace la determinacin de troponina-I pasadas 6 horas del comienzo del dolor9. La cintica de estos marcadores en pacientes con IAM puede resumirse de esta forma: 1. La CPK total comienza a elevarse a las 4-8 horas del comienzo de los sntomas, y permanece elevada hasta que se normaliza a las 48-72 horas10. 2. La isoenzima MB de la CPK (CPK-MB) aparece en el suero tres horas despus del comienzo del IAM, con un pico mximo a las 18-20 horas, alcanzando valores 16 veces superiores al normal. A partir de este punto desciende lentamente y persiste elevada al menos 2 das11. 3. La troponina-I se eleva a partir de las 2-3 horas del comienzo de los sntomas, con un valor mximo a las 16 horas. Desciende bruscamente hasta las 48 horas, y a partir de entonces se produce un lento descenso; puede detectarse todava el 7-8 da11. 4. La mioglobina es la primera que se eleva. Da las cifras ms altas de sensibilidad en el plazo de dos horas del comienzo del dolor10 con respecto a la troponina-I y la CPK-MB. Alcanza su pico a las 8 horas y desciende bruscamente a valores normales a las 18 horas11

Utilidad de la troponina-I, CPK-MB y mioglobina en el diagnstico del infarto de miocardio y de los procesos de necrosis muscular de origen no cardiaco

J.M. Olite3, I. Idoate3, I. Berrozpe1, E. Arina1, L. Metola1, J. Sesma1

Use of troponin-I, CPK-MB and myoglobin in the diagnosis of myocardial infarct and processes of muscular necrosis of non-cardiac origin J.I. Ibez1, R. Sobrado1, M. Rivero2,

1. Unidad de Urgencias. 2. Servicio de Medicina Interna. 3. Servicio de Bioqumica Clnica. Hospital Virgen del Camino Pamplona.

15.- Masculino de 40 aos que de manera inesperada y espontnea inicia cuadro constituido por disnea, sensacin de ahogo o de paro respiratorio, sensacin de inestabilidad, siente que se desmaya, palpitaciones o taquicardia, mareo, nauseas, dolor abdominal. Cul de los siguientes el diagnstico mas probable?

a) Trastornos de ansiedad generalizada. b) Trastorno fbico. c) Trastorno de pnico. d) Trastorno mixto ansioso-deperesivo.

La caracterstica fundamental del Trastorno de Pnico es la presencia de Crisis de Pnico recurrentes, inesperadas y que no se encuentran relacionadas con ninguna circunstancia en particular (es decir son espontneas), sin un factor externo que las desencadene, es decir que no son desencadenadas por una exposicin a una situacin social (como es el caso de la Fobia Social) o frente a un objeto temido (en cuyo caso se tratara de una Fobia Especfica ). Otra caracterstica del Trastorno de Pnico es el miedo persistente a padecer una nueva Crisis de Pnico, esto es miedo al miedo y se lo denomina Ansiedad Anticipatoria. Esta ansiedad puede a llegar a ser tan importante que puede llevar a confundir el diagnstico de Trastorno de Pnico con el de Trastorno de Ansiedad Generalizada (TAG). Estas Crisis (o Ataques) de Pnico, inesperadas y recidivantes, suelen presentar cuatro o ms de los siguientes sntomas: Miedo intenso a morir o a estar sufriendo un ataque cardaco o alguna enfermedad fsica grave que ponga en riesgo la vida. Miedo intenso a volverse loco o a perder el control de si mismo. Palpitaciones (percepcin del latido cardaco) o pulsaciones aceleradas (taquicardia) Sudoracin. Palidez. Temblores o sacudidas musculares. Sensacin de ahogo o falta de aire. Opresin en la garganta (sensacin de no poder respirar) o en el pecho. Nuseas, vmitos o molestias y dolores abdominales. Inestabilidad, mareos o desmayos. Sensacin de irrealidad (sentir al mundo externo como algo extrao) Sensacin de no ser uno mismo (despersonalizacin) Hormigueos (parestesias) Escalofros o sensacin de sufrir fro intenso.

Las crisis se inician bruscamente, alcanzan su mxima intensidad en los primeros diez a 15 minutos y suelen durar menos de una hora. Dejan a quien las sufre en un estado de total agotamiento psicofsico y con un gran temor (ansiedad anticipatoria) a volver a padecer una nueva crisis (miedo al miedo). Tambin pueden aparecer sntomas de tipo depresivo, miedo a salir o alejarse del hogar o necesidad de hacerlo acompaado por una ser muy cercano (familiar). La persona suele sentirse muy hipersensible y vulnerable. Este tipo de trastorno es tan traumtico de experimentar que quien lo padece suele cambiar en forma brusca y desfavorable sus hbitos de vida: No querer salir solo de la casa o viajar, retraimiento social, abandono de sus actividades laborales o acadmicas. Muchas veces la persona que se encuentra padeciendo una Crisis de Pnico tiene la necesidad de "salir corriendo" del lugar donde se encuentra o de consultar urgentemente a un mdico en el caso que crea que se esta muriendo de verdad. Si esta crisis se experimenta por primera vez en un lugar determinado, un tren o autobs por ejemplo, suele quedar temor de volver a ese mismo sitio o medio de locomocin, desarrollndose de este modo una fobia al mismo. Un paciente que sufra de Crisis de Pnico describi su primera crisis de la siguiente forma:

"Sbitamente sent una oleada de miedo sin que hubiera razn alguna. El corazn me lata apresuradamente, me faltaba el aire, senta que no poda respirar, el corazn lata tan fuerte que pareca que iba a salirse por mi boca. Me dola el pecho, tena mareos, ganas de vomitar, no poda parar de temblar. Senta que me mora. Era como una agona mortal que nunca terminaba"
Este trastorno se lo clasifica dentro de las neurosis, no tiene nada que ver con la locura (psicosis), ni desencadena con el tiempo en un cuadro de locura. Es importante recalcar esto ya que muchos pacientes que sufren este trastorno, o los familiares del mismo, piensan que se estn volviendo locos. Lo que s es frecuente es que las personas que padecen de Trastorno de Pnico desarrollen, si no son tratados a tiempo y adecuadamente, cuadros de tipo depresivos, fobias mltiples (especialmente agorafobia) o abuso de sustancias (alcohol o drogas).

Referencias Bibliogrficas:
- Diagnostic and Statistical Manual of Mental Disorders, 4 edition (DSM-IV TR). American Psychiatric Association. American Psychiatric Press, 2000. - Kaplan and Sadock's Synopsis of Psychiatry, 9 edition. Lippincott Williams & Wilkins Press, 2003.

16.- Masculino de 56 aos, indigente, es trado por paramdicos quienes refieren tos purulenta y mal oliente. A su ingreso al servicio de urgencias saturacin de oxgeno de 72%. En radiografa de trax se observan opacidades en lbulos medio e inferior de pulmn derecho. Qu tratamiento est indicado en este paciente?

a) b) c) d)

Cefuroxime IV. Metronidazol IV. Ceftriaxona IV. Clindamicina IV.

La neumona por aspiracin se desarrolla despus de la inhalacin de material orofarngeo colonizado. La aspiracin de secreciones colonizadas de la orofaringe es el mecanismo primario por el cual la bacteria gana entrada a los pulmones. Los signos de hipoxemia, como la taquipnea, taquicardia y cianosis, surgen en forma inmediata y persisten durante varias horas. La auscultacin torcica indica la presencia de jadeos, estertores y roncus, y es factible que el paciente expectore grandes volmenes de esputo sanguinolento y espumoso. La broncoaspiracin grave da por resultado insuficiencia respiratoria con una combinacin de acidosis metablica y respiratoria. Los antibiticos de eleccin para los casos de neumona por aspiracin son la penicilina y la clindamicina. Georges CB. Neumona por broncoaspiracin, empiema y absceso pulmonar. En: Tintinalli JE, Krome RL, Ruiz E. Medicina de Urgencias. 4 edicin. Interamericana McGraw Hill. Mxico.

17.- Masculino con diagnstico de Displasia del desarrollo de cadera, El mtodo ms frecuentemente usado para el tratamiento conservador de sta patologa es: a) Uso de triple paal b) Uso de cojn de Frejka c) Uso de arns de Pavlik d) Uso de cojn de Creig

ARNES DE PAVLIK La displasia en el desarrollo de la cadera es la principal indicacin para el uso del arns de pavlik, teniendo como funcin el mantener la cadera centrada y de esta forma se permita un mejor desarrollo de la misma

1- Dezateux C. Rosendahl K.Developmental dysplasia of the hip. Lancet 2007; 369: 154152. 2- Lee M., Craig P. Growth and Development of the Childs Hip. Orthop Clin N Am 37 (2006) 119 132. 3- U.S. Preventive services task force. Screening for Developmental Dysplasia of the Hip: Recommendation Statement. American Family Physician Volume 73, Number 11, Junio 1, 2006. 4- Langman S. Embriologia Medica con orientacion clinica. Decima edicion. Panamericana. 5- Rosselli P., Duplat J., Uribe i., Turriago C. Ortopedia Infantil. Editorial Panamericana. 6- Gelfer P, Kennedy K. Developmental Dysplasia of the Hip. J Pediatr Health Care. (2008). 22, 318-322. 7- Rachio KH. Simultaneuos open reduction and salter innominate osteotomya for developmentaldysplasia of the hip. J Bone Joint Surg 2004; 78B47:1-6.

18.- Femenino de 45 aos que tras 20 min. Posteriores al inicio de una transfusin presenta fiebre, ansiedad, dolor lumbar severo, nausea y enrojecimiento facial, por lo que usted sospecha: a) Reaccin hemoltica aguda b) Reaccin febril c) Toxicidad por citrato d) Hepatitis

R e a c i n Hemoltica Definicin: Destruccin acelerada del eritrocito. De acuerdo a la causa puede ser inmune o no inmune, por el sitio de destruccin puede ser intra o extravascular y por el tiempo de aparicin puede ser aguda o retardada. Incidencia Reaccin hemoltica aguda: Las referencias internacionales reportan una incidencia de reaccin hemoltica aguda de 1 en 6 000 en 30,000 unidades transfundidas, con una tasa de mortalidad de 1 en 500, 000 a 1 en 1 000 000 de unidades. Del total de las reacciones hemolticas agudas, el 6% resultan fatales. La FDA reporta que alrededor del 41% de las muertes por transfusin son causadas por incompatibilidad ABO, con una incidencia de mortalidad de 1 en 200 000 pacientes transfundidos. Reaccin hemoltica retardada. La incidencia es de 1 en 2,500 a 1 en 4,000 unidades transfundidas. La mortalidad es de 1 en 3.85 millones de unidades y de 1 en 1.15 millones de pacientes transfundidos.

Fisiopatogenia
La reaccin hemoltica transfusional ms grave se presenta cuando interactan los eritrocitos transfundidos con los anticuerpos preformados en el receptor. La reaccin antgenoanticuerpo puede no activar complemento de acuerdo a la inmunoglobulina implicada, lo que conduce a hemlisis intra o extravascular. En la hemlisis extravascular algunas citocinas con actividad inflamatoria y vasoactiva intervienen en la reaccin como: Factor de necrosis tumoral alfa, Interleucina 1, 6, 8 y Protena quimioatractante de macrfago (MCP), as como la liberacin de sustancias tromboplsticas que explican el cuadro clnico caracterstico de la reaccin hemoltica transfusional. En la hemlisis extravascular el eritrocito sensibilizado es destruido por el sistema fagoctico mononuclear.

REACCION HEMOLITICA AGUDA INMEDIATA DE TIPO INMUNE: Es la reaccin ya descrita por incompatibilidad ABO y como consecuencia de ella al iniciar la transfusin en pocos minutos pueden ser lisados la mitad ms de los de los hemates transfundidos. La reaccin hemoltica aguda se produce de forma brusca, tras la infusin de varios mililitros de sangre aparece un cuadro de fiebre, escalofrios, dolor lumbar, dolor opresivo en torax, cefalea, nauseas con sin vomitos e incluso shock, dependiendo de la cantidad y rapidez de la sangre transfundida.

Los sintomas dependen de los pacientes pero la fiebre aparece en todos los casos. La reaccin es inmediata al inicio de la transfusin En los enfermos anestesiados estas reacciones quedan enmascaradas y ser mnimas, por lo que la hemlisis puede debutar con taquicardia e hipotensin brusca trs el inicio de la transfusin,si persiste la transfusin puede aparecer oliguria,hemoglobinuria,CID fracaso renal postoperatorio debido a algn efecto txico de la Hb a la situacin de shock.

Diagnstico Diferencial Contaminacin Bacteriana del componente sanguneo Hemlisis no inmune: - Mecnica - Trmica - Osmtica

Lecturas recomedadas: Rev Mex Med Tran, Vol. 3, Nm. 1, pp 18-21 Enero - Abril, 2010 Hospital de Pediatra CMN Siglo XXI Boletn La Calidad Abr May Jun 2004.

19.- Mujer de 25 aos de edad que presenta en la cara diseminado comedones, ppulas y pstulas, crecimiento excesivo de vello en mejillas mentn y cuello, acompaada de seborrea, refiere cada de pelo exagerada. Inici desde la adolescencia. El diagnstico clnico es? a) b) c) d) Acn e Hirsutismo. Sndrome de masculinizacin. Lupus eritematoso discoide. Alopecia androgentica.

El hirsutismo es el crecimiento excesivo de vello terminal en mujeres siguiendo un patrn masculino de distribucin, en zonas andrgeno-dependientes: patillas, barbilla, cuello, areolas mamarias, trax, en rea inmediatamente superior o inferior al ombligo, as como en muslos, espalda. Frecuentemente se asocia a acn, calvicie con patrn masculino (alopecia andrognica) e irregularidades menstruales. Es un trastorno que afecta aproximadamente al 10% de las mujeres en edad frtil, y puede ser leve, lo que representa una variacin del patrn de crecimiento normal, y en raras ocasiones es signo de un trastorno subyacente grave. Por lo general, es idioptico, pero puede estar relacionado al exceso de andrgenos, como el sndrome de ovario poliqustico o la hiperplasia suprarrenal congnita. Los pacientes con acn presentan de un modo ms o menos frecuente seborrea de cuero cabelludo, con o sin pitiriasis. Otras alteraciones andrgeno-dependientes pueden asociarse, pero no de un modo habitual, como sucede con el hirsutismo y la alopeca, que pueden presentarse en menos del 10% de las pacientes. En el Sndrome de Cushing Hipofisario, la ACTH estimula la produccin suprarrenal de hormonas andro-gnicas, resultando esto en acn e hirsutismo. En el sndrome de poliquistosis ovrica (Stein-Leventhal), hay un incremento de la secrecin gonadal de andrgenos a nivel del hilio y de la teca interna. La hipersecrecin de andrgenos dotados de bioactividad puede inducir acn e hirsutismo. La presencia de acn en una mujer -adolescente o adulta - con niveles de testosterona srica superiores a 300 ng/dl sugiere la presencia de un tumor ovrico. En la Anorexia Nerviosa, el acn se desencadena generalmente en la fase de recuperacin, donde puede coexistir con un ovario poliqustico. En fase de amenorrea LHRH, LH, FSH, estrgenos y progesterona estn disminudos. El cortisol plasmtico est elevado con vida media prolongada y prdida de la variacin diurna, otro factor que puede producir acn.

No hay que olvidar aquellos sndromes donde el acn coexiste con importantes trastornos osteoarti-culares, como ser el Sndrome de Apert y el Sndrome SAPHO. El acn se halla generalmente presente en los pacientes epilpticos - tal vez provocado en gran medida por la medicacin - y en pacientes psiquitricos (manaco-depresivos y obsesivo-compulsivos). Es importante tener en cuenta la asociacin de una neurosis de angustia inespecfica y el acn.

1. Azziz R, Carmina E, Sawaya ME. Idiopathic hirsutism. Endocr Rev 2000;21:347-62. Hirsutismo. Gonzalez Guerra. Servicio de dermatologa.Fundacin Jimnez Daz (Madrid)

20.- Masculino de 42 aos alcohlico con cirrosis y ascitis. Es hospitalizado por agitacin y comportamiento extrao. Cul de los siguientes hallazgos es el ms til para hacer el diagnstico de encefalopata heptica?

a) b) c) d)

Ictericia Asterixis de las manos Hemangiomas superficiales Signo de la ola positivo

ENCEFALOPATA Alteracin en el estado mental, en el comportamiento y en el sueo que progresa a la desorientacin y al coma. Indica una insuficiencia heptica severa. Fisiopatologia: Las toxinas son inactivadas por el hgado pero entran a la circulacin portal por los cortos circuitos. Toxinas no son inactivadas y excretadas. Las toxinas se encuentran elevadas en el lquido cerebroespinal. TOXINAS Amonio: Neuroexcitatorio y depresor del sistema nervioso central. Aminocidos aromticos: Son precursores de neurotransmisores. Si incrementan en sangre la encefalopata no progresa. Acido gammaaminobutrico (GABA) Incrementado significativamente en la encefalopata.

Benzodiacepinas endgenas: Compiten por los receptores de GABA y barbitricos.

CUADRO CLINICO El diagnstico de la encefalopata heptica depende de la existencia de una enfermedad heptica, desde la insuficiencia heptica aguda y la toxicidad por frmacos, hasta las enfermedades crnicas, tales como la hepatropata alcohlica o la cirrosis. En la historia clnica se hace nfasis especial en los antecedentes de enfermedades del hgado diagnosticadas con anterioridad, una historia de alcoholismo o de hepatitis. Transfusiones de sangre previas o abuso de drogas por va intravenosa pueden sugerir una hepatitis crnica B o C. La utilizacin de frmacos hepatotxicos, como la metildopa, la nitrofurantoina o la isonicida, puede ser causa de una hepatopata crnica, mientras que una dosis alta de paracetamol puede ser causante de una necrosis hepatocelular fulminante. El cuadro clnico incluye tres elementos: cambios en el estado mental, hedor heptico y asterixis. El hedor heptico se refiere al aliento ftido del paciente. La asterixis es un temblor por sacudidas, irregular y bilateral de las manos, debido a una interrupcin momentnea y brusca del tono muscular de los antebrazos. El examen fsico se centra en la bsqueda de los estigmas caractersticos de las enfermedades hepticas: ictericia, nevus en araa, ginecomastia, atrofia testicular, venas distendidas en la pared abdominal (cabeza de medusa) y ascitis. La exploracin neurolgica durante los estados precoces suele mostrar apraxia de construccin y dificultad para escribir. Otros hallazgos fsicos son la rigidez de las extremidades e hiperreflexia. En el estado de coma profundo suele haber prdida del tono muscular y disminucin de los reflejos tendinosos profundos.

RESUMEN: Petequias. Rinofima. Contractura de Dupuytren (aponeurosis palmar). Telangiectasias en el tronco. Ascitis. Asterixis. Eritema palmar. Atrofia testicular.

Blibliografa:
Shakelfords. Surgery of the alimentary tract. 5a. Ed. 2002. Tomo 3. Feldmans. Gastroeneterology. 2002. Perez. Anatoma y fisiologa del hgado. Univ. Catlica de Chile. 2005. Bratiz. Serum laboratory test in cirrhosis. Journal of Hepatology. Slovakia. 2005. Paradis. Glycomics. Journal of hepatology. Ireland. Agosto 2005.

21.- Un nio de 10 aos acude a su pediatra con fiebre e inflamacin dolorosa de la rodilla izquierda. Hace 2 semanas tena la garganta irritada, y un cultivo fue positivo para estreptococo hemoltico. El nio se trat desde entonces con un ciclo de 3 das de penicilina oral. Cul de las siguientes ofrece la mejor evidencia, por si sola, del diagnstico de fiebre reumtica aguda? a) b) c) d) Velocidad de sedimentacin eritrocitaria elevada. Alta valoracin de anticuerpos estreptoccicos. Soplo sistlico apical. Intervalo PR prolongado visible en el electrocardiograma.

La carditis es el sntoma diferenciador de fiebre reumtica, y un soplo sistlico apical, es el hallazgo inicial ms comn. Los dems casos enumerados en la lista son propios de pacientes con fiebre reumtica, pero tambin se encuentran en otros trastornos.

Manifestaciones Mayores
Carditis Poliartritis Corea (Sydenham) Ndulos subcutneos Eritema marginado

Clnicas Artralgias Fiebre Antecedentes de brote reumtico Laboratorio y Gabinete Elevacin de reactantes de fase aguda Prolongacin del intervalo PR Evidencia de infeccin Estreptoccica (Grupo A):

Manifestaciones Menores

Antiestreptolisinas Exudado farngeo PAC. Libro 1, Parte A Fiebre Reumtica Pag. 3,7. Dr. Jos F. Guadalajara Boo. Jefe de Servicio Clnico. Instituto Nacional de Cardiologa Ignacio Chvez. Miembro numerario de la Academia Nacional de Medicina. Gobernador por Mxico del American College of Cardiology. Presidente de la Sociedad Mexicana de Cardiologa.

22.- Un nio capaz de correr, subir escaleras solo, sin alternar los pies, hacer una torre de seis cubos y elaborar frases de tres palabras, presenta un desarrollo psicomotor propio de la siguiente edad:

a) b) c) d)

15 meses 18 meses 24 meses 36 meses

Durante el desarrollo neurolgico lo esperable es que a los 15 meses camine solo, gatee escaleras, hace torres de 3 cubos y nombra objetos familiares. A los 18 meses sube escaleras cogido de una mano, hace torres de 4 cubos y nombra partes de su cuerpo. A los 24 meses sube y baja escaleras escaleras, escal a escaln, hace torres de 7 cubos. Frases de 3 palabras (sujeto, verbo y complemento).

Referencias: Bender L.: "Test Guestltico Visomotor de Bender". Ediciones Paids - Barcelona (2006) 2. Brazelton, T.B. y Nugent, J.K. "Escala para la evaluacin del comportamiento neonatal". Paids. Barcelona (1997)3 3. .Broderick P.: "Pediatric vision screnning for the Family Physycian". American Family Psysician Journal. Vol. 58 No 3 (1998) 1.

23.- Femenino al que se diagnostica hirsutismo asociado a ovario poliqustico, usted elige el siguiente frmaco para su tratamiento por ser el ms adecuado:

a) Clomifeno. b) Estrgenos. c) Corticoide. d) Acetato de ciproterona.

El acetato de ciproterona parece ser ms efectivo que otros frmacos para el hirsutismo en mujeres causado por la produccin ovrica excesiva de andrgenos. Una de las causas de hirsutismo (crecimiento piloso excesivo) en mujeres es la hiperproduccin de andrgenos a partir del ovario. Varios frmacos pueden utilizarse para contrarrestar los efectos del andrgeno. El acetato de ciproterona es un frmaco antiandrognico. Los efectos adversos informados con su uso fueron aumento de peso, depresin, fatiga, sntomas mamarios y disfuncin sexual. La revisin de los ensayos encontr que el acetato de ciproterona parece ejercer un efecto en el hirsutismo similar a otros frmacos utilizados para el tratamiento del hirsutismo por exceso de andrgenos. No existen pruebas suficientes para comparar los efectos adversos de las opciones de tratamiento. Van der Spuy ZM, le Roux PA. Acetato de ciproterona para el hirsutismo (Revisin Cochrane traducida). En: La Biblioteca Cochrane Plus, nmero 4, 2007. Oxford, Update Software Ltd. Disponible en: http://www.update-software.com. (Traducida de The Cochrane Library, 2007 Issue 4. Chichester, UK: John Wiley & Sons, Ltd.)

24.- Femenino de 19 aos, atendida en sala de urgencias ginecoobsttricas, Antecedente: cursa embarazo de 38 SDG. Exploracin Fsica: En trabajo de parto. Repentinamente presenta sangrado profuso transvaginal y dolor abdominal. La causa ms probable de la sintomatologa de esta paciente es:

a) Laceracin vaginal por coito b) Cervicitis c) Placenta previa d) Abruptio placentae

DESPRENDIMIENTO PREMATURO DE PLACENTA NORMOINSERTA (DPPNI): Constituye la separacin de la placenta de su rea de insercin antes del 3 perodo del parto. La mortalidad fetal es muy alta (superior al 15%) y la materna es tres veces superior a la esperada. La aparicin frecuente de SFA, prematuridad, anemia, etc., hace que el nmero de secuelas tanto sensitivas como motoras sea alto. Desde el punto de vista materno, complicaciones secundarias a la hemorragia, a las alteraciones de la coagulacin o bien a la embolia pulmonar tambin tienen una tasa muy alta. Desde la antigedad se identific al cuadro clnico caracterizado con la trada sintomtica de hipertona, metrorragia y muerte fetal, con pronstico materno comprometido. En 1775, Ricci diferenci la separacin prematura de una placenta de insercin normal, de aquella de insercin baja; a la primera llam hemorragia accidental, y a la segunda hemorragia imprevisible. Couvelaire introdujo el trmino de desprendimiento prematuro de placenta y describi la aparicin de la apopleja uterina (tero de Couvelaire)

Obstetricia. Scwarcz, Sala, Duverges. 7 edic. Edit. El Ateneo. (Biblioteca Fac. Med. UNNE).

25.- Mujer de 42 aos. Acude a consulta por cefalea y acfenos. Tiene antecedente de DM en padre. EF: peso 78 kg. , estatura 1.62, permetro abdominal 108 cm, TA: 140/100. Laboratorio: glucosa de ayuno 116, prueba de tolerancia a la glucosa, resultado a las 2 horas de 189. Los diagnsticos que se establecen en la paciente son: a) b) c) d) Obesidad y diabetes mellitus. Sndrome metablico con intolerancia a la glucosa. Obesidad e intolerancia a la glucosa. Sndrome metablico y diabetes mellitus.

GLUCOSA: <100 mg/dl = normal 100 y < 126 = glucosa de ayuno alterada (GAA) 126 mg/dl = diabetes mellitas Glucosa a las 2 horas postcarga:

1 40 mg/dl = normal 1 40 a 1 99 mg/dl = intolerancia a la glucosa 200 mg/dl = diabetes mellitus

El sndrome metablico es la agrupacin de diferentes factores de riesgo asociados con el sndrome de resistencia a la insulina. En la literatura existen variaciones importantes en la prevalencia de este sndrome, dependiendo de los criterios o definicin que se empleen para su diagnstico. En la actualidad hay ms de cinco definiciones del sndrome metablico; la descrita en 2001 y su actualizacin en 2005 por el Programa Nacional de Educacin para el Colesterol, Panel de Tratamiento del Adulto III (NCEP-ATPIII), es la ms empleada en la prctica clnica. Esta definicin establece que se requiere la presencia de tres o ms de los siguientes cinco criterios: glucosa en ayuno 100 mg/dl, triglicridos 150 mg/dl, colesterol HDL bajo (< 40 mg/dl en el hombre o < 50 mg/dl en la mujer), tensin arterial 130/85 mm Hg o en tratamiento para la hipertensin y obesidad abdominal detectada mediante medicin del permetro abdominal (> 102 cm en hombres y > 88 cm en mujeres).1 La obesidad ha alcanzado proporciones epidmicas en los pases occidentales y, por lo tanto, es un importante problema de salud. De hecho, Mxico ocupa el segundo lugar a nivel mundial en sobrepeso y obesidad. La resistencia a la insulina que se promueve por la obesidad abdominal o fenotipo de obesidad androide se considera la responsable de algunos factores de riesgo que se agrupan en el sndrome metablico 2. Estudios recientes indican que la prevalencia del sndrome metablico en la poblacin general en Mxico es de aproximadamente 26.6 %, similar a los resultados informados por la Tercera Encuesta de Evaluacin Nacional en Salud y Nutricin en poblacin de Estados Unidos, que fue de 26.7 %.3 Sin embargo, la prevalencia aumenta conforme avanza la edad, llegando a ser hasta de 44 % en individuos mayores de 50 aos. 4. El sndrome metablico no slo incluye algunos factores de riesgo cardiovascular tradicionales, sino otros componentes que representan aspectos involucrados en el desarrollo y progresin de enfermedades cardiovasculares, como la disfuncin endotelial, marcadores de inflamacin y alteraciones en la coagulacin. La sinergia de la combinacin de estos factores se considera causante del aumento en el riesgo de morbilidad y mortalidad asociada con enfermedades cardiovasculares en este grupo de pacientes. Los pacientes que completan criterios para sndrome metablico tienen tres veces ms riesgo de padecer enfermedad arterial coronaria y cerebrovascular.5 Ms an, en pacientes con enfermedad vascular sintomtica se encontr que el sndrome metablico se asociaba con la extensin del dao vascular. 6,7. El sndrome metablico es uno de los principales problemas de salud pblica del siglo XXI. El diagnstico es sencillo, principalmente con base en datos clnicos y bioqumicos sistemticos, los cuales estn al alcance de cualquier sistema de salud Sndrome metablico, impacto clnico y angiogrfico en pacientes con sndrome coronario agudo.

Volumen 78, No. 2, Marzo-Abril 2010

Alejandra Madrid- Miller, * Antonio Alcaraz- Ruiz, * Gabriela Borrayo- Snchez, ** Eduardo Almeida- Gutirrez, * Rosa Mara Vargas- Guzmn, * Ricardo JureguiAguilar***

26.- Se trata de masculino de 24 aos que acude por esterilidad; es azoosprmico, tiene fenotipo eunucoide, ginecomastia, distribucin feminoide del vello pbico, testculos pequeos y cromatina sexual positiva en 8%. El diagnstico a considerar es? a) Sndrome de Noonan b) Sndrome de Turner c) Sndrome de Klinefelter d) Varn XX

SINDROME DE KLINEFELTER (SK) Es la cromosomopata ms frecuente y la causa ms habitual de hipogonadismo hipergonadotrpico en el varn. Descrito en 1942 como un sndrome caracterizado por hipogonadismo, testes pequeos y duros, azoospermia y ginecomastia. Se comprob posteriormente que el cuadro corresponde a una patologa gentica, cuya alteracin cromosmica ms habitual es la presencia de un cromosoma X adicional, reflejando un cariotipo 47 XXY, que representa el 80% de los casos de SK, pero se han descrito otras variantes como mosaicismos: 47XXY/46XY, 47XXY/46XX, 47XXY/46XY/45X, etc. y formas con ms de un cromosoma X Y (48XXYY 47XXY/46XX/poliX). Esto hace que estos sujetos presenten una cromatina de Barr con masa presente, siendo esto propio de las mujeres por la presencia de 2 cromosomas X. La aparicin de ms de 2 cromosomas X, ocasiona una patologa que se diferencia del cuadro clsico de SK y se denomiona polisoma X del varn: 48XXY, 49XXXXY. La frmula XXY se debe a una no disyuncin del cromosoma X en la primera segunda divisin meitica, siendo ms frecuente la aparicin de SK en relacin a la edad materna ms avanzada. Su frecuencia se estima en 1/1000 recin nacidos varones, pero posiblemente sea ms elevada: en torno a 1/500 concepciones masculinas, y en varones con retraso mental moderado la frecuencia aumenta a 8/1000 aproximadamente. CLINICA: en general el diagnstico es tardo, debido a la pobre expresividad clnica de este sndrome en la infancia. 1) Talla alta: Es propia del sndrome de Klinefelter en la edad de la adolescencia, pero no lo es en la poca prepuberal de forma habitual. Las proporciones corporales son eunucoides, con aumento desproporcionado de las extremidades inferiores y con inversin del cociente: segmento superior/inferior. Cuando este cociente es inferior a 1 en la edad puberal, sugiere fuertemente la existencia de un SK si se asocia a un cuadro de hipogonadismo aunque sea leve. La velocidad de crecimiento suele aumentar a partir de los 4 5 aos, y se adopta ya en la infancia un cierto hbito eunucoide. La edad sea es normal poco retrasada, coincidiendo con el marcado retraso de otras variantes benignas de retraso puberal.

2) Alteraciones genitales: Los testes pequeos y duros en la edad prepuberal, pueden presentarse en ocasiones, siendo por el contrario un hallazgo tpico pasada la adolescencia. La presencia de criptorqudea, micropene, hipospadias etc. y a veces un fenotipo peculiar, puede orientar al diagnstico. En la edad puberal, la atrofia testicular es un signo constante, los testes son pequeos y su tamao va involucionando con el tiempo, contrastando con un desarrollo normal del pene y del relativamente normal desarrollo puberal. La histologa del testculo demuestra atrofia testicular con hialinizacin de los tbulos seminferos. 3) Ginecomastia: Est presente en un elevado n de casos (ms del 60%), siendo el riesgo de neoplasia mamaria 20 veces superior al de la poblacin normal. 4) Desarrollo intelectual: La inteligencia en general suele ser normal, bordeline, especialmente es el rea verbal la ms precoz y afectada, lo que condiciona dificultades de aprendizaje y en general peor rendimiento escolar. 5) Alteraciones hormonales: En la edad prepuberal, tanto el nivel de gonadotrofinas como de testosterona se encuentran en rango normal, tanto basal como tras estmulo. En la pubertad se va estableciendo ya respuesta propia del hipogonadismo hipergonadotrpico, especialmente el nivel de FSH que se eleva de forma llamativa. 6) Otras alteraciones asociadas: Anomalas del tracto urinario, cbito valgo etc. (se recogen en la tabla adjunta). Especial relevancia por la ayuda que puede suponer para el pediatra, es la relativamente aumentada incidencia de incontinentia pigmenti. Son frecuentes las asociaciones del SK con neoplasias especialmente las de clulas germinales mediastnicas, y con otros tumores en la edad adulta. Rossodivita A, Colabucci F. Short stature in a patient with Klinefelter syndrome and growth hormone deficiency.Am J of Med Genet 1994; 49:244-6.

27.- Al realizar un ensayo clnico controlado doble ciego para evaluar un nuevo antiinflamatorio, Qu condicin deben cumplir los participantes del estudio?

a) Ni el grupo de estudio ni el grupo control conocen a los observadores. b) Ni el observador ni los sujetos conocen cual grupo recibe el nuevo medicamento y cual el placebo. c) Los sujetos del grupo control no conocen a los sujetos del grupo en estudio. d) La asignacin del tratamiento no es conocida por los pacientes.

Para considerar el efecto de placebo y reducir los sesgos debido a las concepciones de los pacientes y los investigadores el estudio puede conducirse bajo un patrn ciego. En un estudio doble ciego, la asignacin al tratamiento no es conocida por los pacientes ni por los mdicos.

Greenberg R. S; Epidemiologa mdica, Manual Moderno, 2. Ed.

Pg. 1 1 6.

28.- Un paciente de 65 aos presenta dolor, calor y tumefaccin en la rodilla derecha. Cul de los siguientes hallazgos sera ms til para establecer el diagnstico de pseudogota en este enfermo? a) Crecimiento de articulaciones interfalngicas proximales e interfalngicas distales. b) Altas concentraciones sricas de cido rico. b) Cristales con birrefringencia negativa del lquido de la rodilla. d) Calcio en el menisco en una radiografa de la rodilla afectada.

Allen R. M. MMS Medicina Interna. 5. Edicin. National Medical Series. Mc. Graw Hill. 2006. (captulo 10 IV B 5 b). El hallazgo de un menisco calcificado en la radiografa de la rodilla afectada es un dato diagnstico de enfermedad por dihidrato de pirofosfato de calcio, que sugiere que la inflamacin de la rodilla es causada por seudogota. El crecimiento de las articulaciones interfalngicas proximales y distales sugiere slo osteoartritis y no una causa especfica. El aumento srico de urato se relaciona con gota. El aumento de cristales con birrefringencia en examen de luz polarizada compensada roja, de lquido sinovial, es especfico para el diagnstico de seudogota. Hay muchas causas de derrame inflamatorio aparte de esta entidad.

29.- Masculino de 34 aos con diagnstico de absceso heptico amibiano. Indic tratamiento con Metronidazol, con lo que se observ respuesta clnica favorable. Posteriormente complementa el tratamiento con el siguiente frmaco: a) b) c) d) Emetina Cloroquina Albendazol Iodoquinol (hidroxiquinolena)

Posterior al tratamiento para la forma invasiva (trofozoito), para el cual el tratamiento ms efectivo al momento es el Metronidazol, pero sobre el que tienen tambin efectos la emetina, Cloroquina y tinidazol, se recomienda utilizar un agente con actividad cisticida (actividad luminal), como el iodoquinol o la Paromomicina.

Kasper DL, Braunwald E, Fauci AS, Hauser SL, Longo DL, Jameson JL. Harrisons Principles of Internal Medicine. McGraw Hill. 16 Ed. 1214-1217 pp.

30.- Cuando se observa una imagen histolgica de acantosis con elongacin de las crestas interpapilares-que incluso se fusionan entre s, hiperparaqueratosis y acmulos epidrmicos de leucocitos polimorfonucleares, estamos hablando de:

a) Icitiosis. b) Dermatitis (eccema). c) Psoriasis. d) Epidermlisis.

Psoriasis: La psoriasis es una enfermedad cutnea caracterizada por un curso crnico, que cursa a brotes y tiene hallazgos clnicos variables. Las manifestaciones cutneas de esta enfermedad son tan caractersticas (Tabla 1) que el diagnstico suele realizarse con facilidad. En la actualidad se contempla la psoriasis como una enfermedad de base inmunolgica, mediada por los linfocitos T, que asocia inflamacin drmica y secundariamente hiperplasia epidrmica. Es probablemente la enfermedad dermatolgica mediada inmunolgicamente ms prevalente. Tabla 1 Caractersticas clnicas de la psoriasis 1. 2. 3. 4. 5. Placa eritematosa Escamas blanquecinas Buena delimitacin Signo de Auspitz Fenmeno de Koebner

Psoriasis. Hiperparaqueratosis con acmulos intracrneos de neutrfilos (microabscesos de Munro). HE, x 400

Cambios histolgicos de la psoriasis: Infiltracin de clulas mononucleares. Hiperplasia epidrmica. Acantosis epidermica regular, con hipogranulosis, hiperqueratosis y paraqueratosis. Atrofia epidermica suprapapilar. Infiltracininflamatoria. Pustula espongiforme de Kogog. Microabscesos de Munro. Infiltrado de linfocitos CD4 y CD8 en dermis. Cambios vasculares. Proliferacin de vasos capilares dilatados ocupando las papilas.

Bibliografa:
Arnold HL, James WD, Odom RB. Andrews : tratado de dermatologa. 1993. 4 ed. Barcelona: Masson; 1993. Ashcroft DM, Po AL, Williams HC, Griffiths CE. Systematic review of comparative efficacy and tolerability of Calcipotriolin treating chronic plaque psoriasis. BMJ 2000; 320:963-967. [Medline][Texto completo] Chalmers RJG, O'Sullivan T, Owen CM, Griffiths CEM. Intervenciones para la psoriasis guttata (Revisin Cochrane). En: La Cochrane Library Plus, Nmero 2, 2002. Oxford: Update Software. Ferrndiz Foraster C. Dermatosis eritematoescamosas (I). Psoriasis. Eritrodermias. En: Ferrndiz C, ed. Dermatologa Clnica.2 ed. Madrid: Harcourt; 2001. p. 165-175. Freedberg I, Eisen A, Katz SI, Wolff K, Fitzpatrick TB, Goldsmith LA et al, eds. Dermatology in general medicine. 5th ed. New York: McGraw-Hill; 1999. Riffiths C E, Clark C M, Chalmers R J, Li Wan Po A, Williams H C. A systematic review of treatments of severe psoriasis. Health Technology Assessment, 2000;4(40):1-125. Naldi L, Rzany B. Chronic plaque psoriasis. Clin Evid 2002; 7: 1488-1507.

31.- Un cuadro de diarrea con una duracin de ms de dos semanas, pero que generalmente no se extiende por ms all de cuatro se define como: a) b) c) d) Diarrea aguda Diarrea persistente Diarrea crnica Diarrea acuosa

La clasificacin de la diarrea de acuerdo al tiempo de duracin de este sntoma la divide en: aguda, con duracin de menos de 14 das, crnica con duracin de ms de 4 semanas, y persistente ha la que tiene una duracin menor del mes, y mayor a las dos semanas. Kasper DL, Braunwald E, Fauci AS, Hauser SL, Longo DL, Jameson JL. Harrisons Principles of Internal Medicine. McGraw Hill. 16 Ed. 225 p.

32. - Which of the following physical sings and syntoms is indicative of left ventricular failure? a) b) c) d) Neck vein distensin Ascites Anorexia Orthopnea

INSUFICIENCIA CARDIACA IZQUIERDA: Los sntomas ms caractersticos son la disnea paroxstica nocturna, la ortopnea y la tos. Al examen fsico se constata la existencia de taquicardia, R3 y/o R4, pulso alternante, estertores inspiratorios de predominio en las bases. Algunas veces se auscultan sibilancias. Para confirmar su existencia se deben solicitar los siguientes estudios: a. Radiografa de trax. Para constatar la existencia de cardiomegalia, vasos sanguneos prominentes, lneas B de Kerley, (patrn en "alas de mariposa" por edema pulmonar gravitacional) y derrame pleural.

b. Gases arteriales

c.

Cuadro hemtico

d. Creatinina

e.

Electrocardiograma

f.

Ecocardiografa

LECTURAS RECOMENDADAS: Bigger JT. Why patients with congestive heart failure die. Circulation 75 (suppl, IV):28, 1997. Braunwald E. Heart Disease. En: Textbook of Cardiovascular Medicine. WB Saunders Co. Philadelphia, 1990. Matiz H. Insuficiencia cardiaca congestiva En: Diagnstico y Tratamiento Integral en Medicina. Editado H Matiz. Coleccin Educacin Mdica Vol. 5. Fundacin Escuela Colombiana de Medicina Santaf de Bogot, 1991.

33.- Un hombre de 35 aos acude a consulta porque tiene dolor sordo perineal y persistente, disuria de 6 meses de evolucin. Niega IVUS y descarga uretral. Su temperatura es de 37C. Al examen rectal la prstata es ligeramente dolorosa, pero no esta aumentada de tamao ni indurada. Uroanlisis normal. Secrecin prosttica, muestra 30 leucos por campo sin bacterias. Cultivos de secrecin prosttica y orina negativos. Cul es el diagnstico ms probable.

a) b) c) d)

Cistitis aguda. Prostatitis aguda. Prostatitis bacteriana crnica. Prostatitis no bacteriana crnica.

La Prostatitis es uno de los ms comunes problemas urolgicos. Cerca del 50% de los hombres experimentan un episodio de prostatitis una vez en la vida. Debido a que la Hiperplasia Prosttica Benigna (HPB), el Cncer de Prstata (CP) y la Prostatitis Crnica Abacteriana (PCA) pueden coexistir, es difcil distinguir entre estas entidades.

Los sntomas de PCA y de HPB se superponen de tal forma que muchos ancianos reciben equivocadamente ste ltimo diagnstico. Aunque a diferencia de la HPB y el CP que son preferentemente del hombre anciano, la PCA puede presentarse a cualquier edad. La Prostatitis puede elevar los niveles de Antgeno Prosttico Especfico (PSA) lo que conduce a la realizacin de un gran nmero de biopsias prostticas innecesarias. En la Prostatitis Crnica se desconoce no slo qu la causa sino cul es la mejor forma de manejarla. Drach y Col clasificaron y definieron a la Prostatitis de la siguiente forma: * Prostatitis Bacteriana Aguda: Se define por la recuperacin de bacterias del fluido prosttico, fluido purulento y sntomas sistmicos de infeccin (fiebre, mialgia, etc.) * Prostatitis Bacteriana Crnica: Recuperacin de bacterias en nmero significativo del fluido prosttico en ausencia de infeccin urinaria o signos significativos de infeccin sistmica. * Prostatitis Abacteriana: Sin recuperacin de nmero significativo de bacterias del fluido prosttico pero con pus microscpica en el mismo. * Prostatodinia: sin recuperacin de bacterias o pus en el lquido prosttico, pero el paciente tiene urgencia urinaria persistente, disuria, discomfort prosttico. El National Institute of Health (NIH) de USA estableci en 1995 un nuevo sistema de clasificacin. Es el siguiente:

* Prostatitis Tipo I: infeccin aguda de la prstata. Corresponde a la Prostatitis bacteriana aguda de la antigua clasificacin.

* Tipo II: infeccin recurrente de la prstata. Corresponde a la Prostatitis Bacteriana Crnica.

* Tipo III: Sin infeccin demostrable. Corresponde a Prostatitis Crnica Abacteriana/Sindrome de Dolor plvico crnico. Tipo III A: leucocitos en semen. Tipo III B: Sin leucocitos en semen.

* Tipo IV: sin sntomas subjetivos, detectado por biopsia prosttica o por la presencia de leucocitos en secreciones prostticas. Corresponde a la Prostatitis inflamatoria asintomtica.

La etiologa de la Prostatitis es desconocida en el 90% de los casos, siendo bacteriana en el 10% de los casos. La literatura antigua describe como determinantes potenciales de esta condicin los niveles de hormonas sexuales, dieta, enfermedades del tracto urinario, stress, factores psicolgicos, alergia. Estudios ms recientes examinaron la edad, etnicidad, agentes infecciosos, niveles de cido rico, actividad sexual, clculos y quistes prostticos, citoquinas proinflamatorias y biopsia prosttica. Bibliografa: 1- McNaughton C. Diagnosis and Treatment of Chronic Abacterial Prostatitis: a Systematic review. Ann Intern Med 2000 ; 133 : 367-381. 2- Leskinen M , Lukkarinen O, Marttila T. Effects of finasteride in patients with inflammatory chronic pelvic pain syndrome: a double blind, placebo-controlled, pilot study. Urology 1999 ; 53 : 502-505.

34.- Masculino recin nacido que presenta una conjuntivitis neonatal, para indicar el tratamiento usted relaciona los siguientes grmenes por ser los de mayor frecuencia causantes de esta enfermedad: a) b) c) d) Clamidia, gonococo, estafilococo aureus. Clamidia, treponema, gonococo. Estreptococo grupo A, listeria. Listeria, pseudomonas.

Las conjuntivitis neonatales (CN) siguen siendo uno de los motivos ms frecuentes de consulta, pudiendo llegar a comprometer seriamente la capacidad visual del beb, conduciendo incluso a la ceguera. La incidencia de esta enfermedad oscila entre 1,6%-12% en el primer mes de vida. Las CN pueden ser de origen infeccioso o qumico. Las CN de origen infeccioso pueden ser adquiridas durante la gestacin, durante el parto por contacto con la secrecin uretrovaginal, o bien en el post-parto y el mbito de convivencia diaria. Las CN que se manifiestan entre las 24 y las 48 horas del nacimiento son de origen qumico, debidas a la profilaxis efectuada por instilacin de solucin de nitrato de plata en el saco conjuntival del neonato, para prevenir la proliferacin de Neisseria gonorrhoeae. Se debe tener presente la adquisicin de la infeccin perinatal a partir del ambiente en que se halla el beb. La etiologa ms frecuente en este caso corresponde a Staphylococcus aureus, Haemophilus influenzae y Streptococcus pneumoniae).

En las ltimas dcadas, Chlamydia trachomatis ha resultado ser el microorganismo prevalente entre los agentes causales de enfermedades de transmisin sexual, con una prevalencia del 2 al 35% en embarazadas. La transmisin vertical se da entre el 60 y 70% en los hijos de madres infectadas .El riesgo de adquirir conjuntivitis en estos bebs es del 18 al 60%, mientras que el de neumona es del 10 al 25%. En el caso de N. gonorrhoeae , la infeccin ocular puede complicar el cuadro con ceguera y artritis. Desde el ao 1995, en nuestro hospital se lleva a cabo en forma rutinaria la bsqueda de grmenes comunes (GC) y C. trachomatis en todo recin nacido con conjuntivitis.

Bibliografa:
Krachmer, Manis, Holland. Cornea and External Disease: Clinical Diagnosis and Management, Vol II. Mosby, 1997. Yanoff, Ophtalmology. Mosby, 1999.

35.- Masculino de 56 aos que llega al servicio de urgencias con dolor torcico de ms de 12 hrs. de evolucin, que inicia en forma sbita, mejora al estar sentado, no tiene antecedentes de importancia, habr que descartar de primera instancia:

a) b) c) d)

TEP Enfermedad cido pptica Cardiopata isqumica Lesin de grandes vasos

El manejo de un paciente con este tipo de dolor, requiere una evaluacin de la severidad, localizacin y caractersticas peculiares de dicho dolor. Muy pocos sntomas suponen una urgencia tan obligatoria como lo es el dolor torcico. Tanto el mdico como el paciente saben que la isquemia miocrdica puede ser causa de muerte sbita, pudiendo generar ansiedad en ambos. La importancia y dificultad en la valoracin del dolor torcico radica en la multitud de causas posibles y en el diferente pronstico segn la patologa subyacente. Al problema diagnstico inherente a un sndrome esencialmente clnico, se aade la dificultad de etiquetar el dolor en poco tiempo (ayudados slo por la clnica, una Rx de trax y un ECG), dada la importancia de iniciar con prontitud el tratamiento ms adecuado en los pacientes con patologa potencialmente letal. Aunque el dolor o la molestia precordial constituye una de las manifestaciones principales de cardiopata, es muy importante recordar que puede originarse no slo en el corazn, sino tambin en: estructuras cardiacas intratorcicas como la aorta, la arteria pulmonar, rbol broncopulmonar, pleura, mediastino, esfago y diafragma; tejidos del cuello o la pared torcica, incluidos piel, msculos torcicos, regin cervicodorsal, uniones costocondrales, mamas, nervios sensoriales o

mdula espinal y rganos abdominales como estmago, duodeno, pncreas o vescula biliar; adems de dolor artificial o funcional.

Manifestaciones clnicas en cardiopata isqumica


Estas se pueden agrupar en cuatro grandes grupos o categoras, las cuales son:

persistente como para causar muerte del tejido muscular cardaco; hay tres tipos que son la angina estable, la de prinzmetal y la inestable.

Angina de pecho: en donde la obstruccin del riego arterial no es lo suficientemente

Infarto del miocardio: en este caso la obstruccin del riego arterial es lo suficientemente
duradera o persistente como para causar necrosis tisular isqumica del miocardio.

Cardiopata isqumica crnica : son pacientes que generalmente han sufrido uno o ms ataques cardacos y han sobrevivido a ellos, pero continan teniendo problemas cardacos debido a que la parte del miocardio que no muere se hipertrofia para suplir las necesidades del cuerpo y esto a su vez causa un aumento de la demanda cardaca debido al aumento de los componentes estructurales de las clulas cardacas, trayendo ms problemas porque no se podr suplir adecuadamente al corazn debido a la obstruccin coronaria. Estos pacientes constituyen el 50% de los que reciben trasplantes cardacos. Muerte sbita cardaca : Es el paro cardaco en el cual se presentaron sntomas en una hora antes de la muerte, o no se presentaron nunca. Causas: aterosclerosis coronaria, estenosis artica, hipertensin sistmica, comnmente arritmias letales (asistlicas y fibrilacin ventricular)
BIBLIOGRAFIA: 1. Goldman L., Braunwald E. Molestias torcicas y Palpitaciones. En Isselbacher KJ., Braunwald E., Wilson JD., Fauci AS., Kasper DL., eds. Harrison, Principios de Medicina Interna. McGraw-Hill. Interamericana de Espaa. 1994. 2. Braunwald E. Tratado de Cardiologa. Interamericana. Mcgraw-Hill. 1993. 3. Harkins SW. Geriatric pain. Pain perceptions in the old age. Clin Geriatric Med 1996. 4. Coto lpez, A., Morales JM., Gutierrez Rodero, F., Gonzalez E., .Dolor Torcico. Manual de diagnstico y teraputica mdica. Gutierrez Rodero F y Garca Daz JD. 2 ed. Madrid, 1990; pag. 165-172. 5. Durn Serantes, M., Caldern de la Barca Gzquez, J.M., Romero Moreno M., Martinez Guilln, J., Montero Prez, FJ., Jimenez Murillo, L., Cardiopata Isqumica ( I ): Angor. Protocolos de actuacin en Medicina de Urgencias. Jimenez Murillo L y Montero Prez FJ. Mosby/Doyma Libros SA. Barcelona 1996; pg. 51-56.

6. James H. Chesebro. La clnica del dolor torcico en el Servicio de Urgencias: abordaje de los pacientes y relacin coste-eficacia. Grandes temas de la cardiologa: avances hacia el cambio de siglo. 1998, American College of Cardiology. 7. Tresch DD, Aronow Ws. Clinical manifestations and clinical diagnosis of coronary artery disease. Clin Geriatr Med. 1996. 8. Owens, G.M.: Chest pain. Primary Care, 1986. 13; pg: 55-61. 9. Rutherford, J.D.; Braunwald, E.: Diagnstico diferencial del dolor precordial. En: Braunwald E: Tratado de Cardiologa, 4 edicin. Interamericana McGraw-Hill, Madrid, 1993; pg: 1448-1449. 10. Williams, E.S.: Approach to the patient with chest pain. En: Kelly WN, ed. Textbook of Internal Medicine. Filadelfia, J.B. Lippincott Company, 1989; pg. 374-379.

36.- Recurren a consulta los padres de un menor, que cuenta con cinco aos de edad, lleva varias noches despertndose agitado como si hubiera soado algo que le angustia. Cuando acuden a su lado por la noche, el nio les mira y dice palabras que no tienen ningn significado. Al cabo de un rato vuelve a dormirse y por la maana no recuerda nada de lo ocurrido. El diagnstico sera:

a) b) c) d)

Terrores nocturnos. Pesadillas Disomnia. Sonambulismo.

Criterios para el diagnstico de F51.5 Pesadillas (307.47) A. Despertares repetidos durante el perodo de sueo mayor o en las siestas diurnas, provocados por sueos extremadamente terrorficos y prolongados que dejan recuerdos vividos, y cuyo contenido suele centrarse en amenazas para la propia supervivencia, seguridad o autoestima. Los despertares suelen ocurrir durante la segunda mitad del perodo de sueo. B. Al despertarse del sueo terrorfico, la persona recupera rpidamente el estado orientado y despierto (a diferencia de la confusin y desorientacin que caracterizan los terrores nocturnos y algunas formas de epilepsia). C. Las pesadillas, o la alteracin del sueo determinada por los continuos despertares, provocan malestar clnicamente significativo o deterioro social, laboral o de otras reas importantes de la actividad del individuo.

D. Las pesadillas no aparecen exclusivamente en el transcurso de otro trastorno mental (p. ej., delirium, trastorno por estrs postraumtico) y no se deben a los efectos fisiolgicos directos de una sustancia (p. ej., drogas, frmacos) o de una enfermedad mdica.

Criterios para el diagnstico de F51.4 Terrores nocturnos (307.46) A. Episodios recurrentes de despertares bruscos, que se producen generalmente durante el primer tercio del episodio de sueo mayor y que se inician con un grito de angustia. B. Aparicin durante el episodio de miedo y signos de activacin vegetativa de carcter intenso, por ejemplo, taquicardia, taquipnea y sudoracin. C. El individuo muestra una falta relativa de respuesta a los esfuerzos de los dems por tranquilizarle. D. Existe amnesia del episodio: El individuo no puede describir recuerdo alguno detallado de lo acontecido durante la noche. E. Estos episodios provocan malestar clnicamente significativo o deterioro social, laboral, o de otras reas importantes de la actividad del individuo. F. La alteracin no se debe a los efectos fisiolgicos directos de una sustancia (p. ej., drogas, frmacos) o de una enfermedad mdica.

37.- Una mujer de 40 aos de edad en buena salud general experimenta dolor retroesternal sbito con fiebre y falta de aire. Es fumadora y no toma medicamentos excepto anticonceptivos orales. En la exploracin fsica se encuentran taquipnea y temperatura de 38C. Los datos de auscultacin, percusin y radiogrficos del trax son normales. Cul de los siguientes diagnsticos es ms probable?

a) Traqueobronquitis b) Neumona atpica c) Embolia pulmonar d) Neumona bacteriana

Allen R. M. MMS Medicina Interna. 5. Edicin. National Medical Series. Mc. Graw Hill. 2006. (captulo 2 VIII E 1, 2 a; captulo 8 V C 3, 4).

El diagnstico ms probable es embolia pulmonar. El inicio agudo descarta neumona atpica y hace poco probable el cncer pulmonar. Sin pruebas de tos productiva es poco probable que haya traqueobronquitis, trastorno tambin subagudo. La neumona bacteriana es muy improbable junto con la radiografa de trax. El tabaquismo y el uso de anticonceptivos orales predisponen a trombosis venosa profunda y embolias pulmonares.

38.- Masculino de 58 aos que cursa con gastritis atrfica, fatiga, palidez de tegumentos, se sospecha de deficiencia de factor intrnseco lo que produce el siguiente tipo de anemia: a) Micorctica b) Drepanoctica c) Talasmica d) Perniciosa

Carencia del factor intrnseco El factor intrnseco es una sustancia natural que normalmente se encuentra en el estmago y es necesaria para absorber la vitamina B12 de los alimentos. Una carencia del factor intrnseco ocasiona anemia perniciosa y deficiencia de vitamina B12, lo cual puede causar anemia y problemas del sistema nervioso y del cerebro. Las causas ms comunes de anemia perniciosa abarcan: Debilitamiento del revestimiento del estmago (atrofia de la mucosa gstrica) El sistema inmunitario ataca las clulas que producen el factor intrnseco (autoinmunidad contra las clulas parietales gstricas) Autoinmunidad contra el factor intrnseco en s.

El comienzo de la enfermedad es lento y puede tomar dcadas para establecerse por completo. Aunque la forma congnita ocurre en nios, la anemia perniciosa por lo general no aparece antes de los 30 aos en adultos y la edad promedio del diagnstico es a los 60 aos.

Referencias Antony AC. Megaloblastic anemias. In: Goldman L, Ausiello D, eds. Cecil Medicine. 23rd ed. Philadelphia, Pa: Saunders Elsevier; 2007: chap 170.

39. - Masculino de 25 aos, cursando 24 horas de postquirrgico por drenaje de hematoma subdural. Antecedentes: Traumatismo craneoenceflico. Exploracin fsica: T/A 110/70 mmHg, FC 78 x, FR 17 x, Temp 36.5. Sbitamente presenta crisis convulsiva tnicoclnica generalizada, la cual usted mitiga con un bolo de diacepam, al mismo tiempo solicita laboratorios reportando: Na 127 mEq/l, K 4.0 mEq/l, Cl 97 mEq/l, glucosa 70 mg/dl, creatinina 1.0 mg/dl. El diagnstico inicial en este paciente es: a) b) c) d) Sx de Diabetes inspida. Sx hipoglucmico. Sx de hipernatremia. Sx de secrecin inapropiada de hormona antidiurtica.

Sndrome de secrecin inapropiada de hormona antidiurtica: Es la causa ms comn de hiponatremia en pacientes hospitalizados. En un paciente que se presenta con hiposmolalidad srica, pero con orina no diluida al mximo (>50 mOms/kg H2O en jvenes). Criterios: Hipoosmolaridad (<300mmol/Kg) + hiponatremia, Osmolaridad urinaria > Osmolaridad plasmtica, Sodio urinario semejante al ingerido, Ausencia de otras causas que alteren la dilucin de orina (funcin renal, suprarrenal, tiroidea normales), Mejora de la natremia a la restriccin acuosa, hormona antidiurtica (ADH) plasmtica no suprimida. Test de sobrecarga de agua (evala osmolaridad plasmtica y urinaria), Ausencia de edema, ortostatismo, deshidratacin. Hernando L, Nefrologa Clnica, Ed. Panamericana, 3Edicin, 2008, Pg.40-41.

40.- Ingresa al servicio de Traumatologa masculino de 35 aos postraumatizado por colisin automovilstica con cuadro de prdida progresiva de fuerza en miembros inferiores, los reflejos osteotendinosos estn abolidos. Al realizarse radiografas simples muestran una fractura por compresin de L1 con desplazamiento del muro posterior y acuamiento anterior de un 50%. Para valorar la ocupacin del canal raqudeo, cual de las siguientes pruebas indicara?

a) Una gammagrafa sea. b) Una tomografa cervical. c) Rx ap y lateral con foco en L5 d) Una TAC vertebral centrada en regin dorso lumbar.

. Tomografa computarizada (TC) En lneas generales, podemos decir que sus indicaciones (con carcter urgente) en los TRM, son todas aquellas lesiones detectadas o sospechadas en las radiografas simples, que puedan suponer riesgo de lesin medular por desplazamientos ulteriores (lesiones inestables), o que ya estn produciendo dao neurolgico susceptible de mejorar o estabilizarse tras descompresin quirrgica. Por lo tanto debe realizarse en todos los pacientes con fracturas, luxaciones y fracturas-luxaciones inestables, y en aquellos con dficit neurolgico, preferentemente incompleto. En pacientes con lesiones estables en las radiografas simples y sin dficit neurolgico, puede diferirse en funcin de la presin asistencial y de la disponibilidad del servicio de Radiologa. En la prctica, suele indicarse tambin cuando no se visualizan determinadas zonas de la columna, generalmente C1-C2 y C6-Dl. En pacientes con TCE grave deben realizarse cortes de estas zonas, si no se ven claramente en las radiografas o en el "scout" cervical, e incluso de forma rutinaria, dada la frecuente asociacin de estas lesiones. La TC proporciona una excelente visualizacin de las estructuras raqudeas principalmente de los elementos posteriores y del canal medular, por lo que se pueden ver con nitidez los desplazamientos y fragmentos seos que puedan estrecharlo o invadirlo. Asimismo, pueden verse fracturas o desplazamientos inadvertidos en las radiografas y nos permite valorar mejor estas lesiones, definiendo claramente las lneas de fractura, cuantificando exactamente los desplazamientos, y en definitiva, evaluando la estabilidad vertebral. Habitualmente se realizan cortes de 5 mm. de espesor, aunque la exploracin detallada de la columna cervical pueda requerir cortes ms delgados, lo que alargar el tiempo de estudio, factor importante en el manejo de los traumatismos. Los equipos de ltimas generaciones han conseguido acortar este tiempo y permiten adems, la reconstruccin sagital o coronal a partir de los cortes axiales, mejorando la definicin y la evaluacin del canal medular. La TC es til tambin para valorar las articulaciones interapofisarias y los agujeros de conjuncin, as como los hematomas paravertebrales y retroperitoneales. Una ventaja adicional es la de ofrecernos informacin suplementaria sobre ciertas partes blandas del cuello y de las cavidades torcica y abdominal. Las fracturas horizontales que no coincidan con el plano de la TC pueden no visualizarse, como la de odontoides o algunas por compresin. Los hematomas epidurales, hematomielia y hernias discales pueden verse tambin con TC, aunque la RM define mejor estas lesiones. La contusin y el edema medular, las lesiones y avulsiones radiculares y los desgarros durales requieren estudio mielogrfico adicional y/o RM. La mielo-TC se realiza con inyeccin de contraste intratecal por puncin lumbar o cervical, que obliga a la movilizacin del paciente o retirada del collarn cervical, adems de otros inconvenientes como tiempo de estudio y reacciones adversas, por lo que generalmente no son tiles en el manejo urgente del paciente con TRM.

Referncias Bibliogrficas:

1.-Balliger, Phillip W. Cerril. Atlas de posiciones radiogrficas y procedimientos radiolgicos. 7. ed.; Espaa: Masson. 2.-Goaz P. W. Radiologa oral (principios e interpretacin). 3. Ed. Espaa; ed.; Mosby.1995.

41.- Un nio de 2 aos acude al centro de urgencias con letargo despus de haber estado jugando sin vigilancia. El nio hasta entonces estaba sano, con un desarrollo normal. En la historia familiar aparece como dato importante que uno de sus hermanos tiene epilepsia y toma fenobarbital. La exploracin revela un nio letrgico sin fiebre que abre brevemente los ojos ante estmulos dolorosos. No hay signos de traumatismo ni anomalas focales. Las pupilas son pequeas pero reactivas. Cul es la etiologa ms probable para el letargo del nio? a) b) c) d) Convulsiones no observadas. Intoxicacin. Hemorragia intracraneal. Traumatismo craneoenceflico no observado.

La causa mas comn de inicio agudo de somnolencia en un nio que empieza a caminar, es la intoxicacin. La presencia de agentes txicos potenciales en la casa es importante en la historia clnica. La miosis tambin sugiere intoxicacin y la falta de signos de traumatismo o anomalas focales hace que una hemorragia sea poco probable.

LECTURA RECOMENDADA: PROGRAMA DE ACTUALIZACION CONTINUA EN PEDIATRIA INTOXICACIONES EN PEDIATRA Dr. Miguel Angel Montoya Cabrera 1.-Jefe del Departamento de Admisin Continua y Toxicologa, Hospital de Pediatra. 2.-Centro Mdico Nacional Siglo XXI, Instituto Mexicano del Seguro Social. 3.-Miembro de la Academia Nacional de Medicina, de la Academia Mexicana de Ciruga y de la Academia Mexicana de Pediatra.

42.- Femenino de 36 aos nulpara tras 2 aos de relaciones sexuales sin contracepcin, que desde hace 1 ao presenta dismenorrea, dispareunia y sangrado vaginal intermenstrual. Cul sera la primera orientacin diagnstica?: a) Insuficiencia lutenica. b) Enfermedad inflamatoria plvica. c) Dismenorrea funcional. d) Endometriosis.

La endometriosis consiste en la aparicin y crecimiento de tejido endometrial fuera del tero, sobre todo en la cavidad plvica como en los ovarios, detrs del tero, en los ligamentos uterinos, en la vejiga urinaria o en el intestino. Es menos frecuente que la endometriosis aparezca fuera del abdomen como en los pulmones o en otras partes del cuerpo. La endometriosis es una enfermedad relativamente frecuente, que puede afectar a cualquier mujer en edad frtil, desde la menarquia hasta la menopausia, aunque algunas veces, la endometriosis puede durar hasta despus de la menopausia. La endometriosis altera la calidad de vida de las mujeres que la padecen, afectando a sus relaciones de pareja, familiares, laborales y de reproduccin.

Sntomas Los sntomas clsicos son la dismenorrea, dolor intermestruales y en muchos casos, esterilidad. El dolor no tiene que ver con el tamao y la severidad de la lesin; generalmente cuanto menor es la lesin mayor dolor produce. El dolor se agrava con las menstruaciones y en los casos en que la lesin ocupa el fondo de saco de Douglas, puede dar dispareunia. Existe un aumento de la PGF2 alfa y PGE2 y un aumento de las contracciones uterinas que podra deberse a un depsito de endometrio en la cavidad peritoneal. La esterilidad debido a la endometriosis podra deberse a distintas causas de acuerdo a la severidad de la patologa. En los casos de endometriosis severa puede haber un factor tuboperitoneal con adherencias y alteracin en la anatoma de la pelvis que interfiera con el transporte del esperma y el vulo. En los casos de endometriosis leve hay varios mecanismos propuestos que justifican su relacin con la infertilidad: Foliculognesis plvico, dispareunia, sangrados

alterada, fase ltea inadecuada, fagocitosis espermtica, mala calidad ovocitaria, embriotoxicidad y alteracin a nivel de la implantacin.. La produccin de prostaglandinas por el endometrio ectpico puede afectar la motilidad tubaria, la foliculognesis y la funcin del cuerpo lteo. Puede haber un aumento de la activacin de los macrfagos peritoneales en la endometriosis que cause la fagocitosis de los espermas o la secrecin de citoquinas que pueden ser txicas para el embrin. Segn algunos investigadores habra un 60% de las mujeres con endometriosis que presentan un sndrome de Folculo Luteinizado no roto (LUF) en el cual el folculo no se rompe en la ovulacin y el vulo queda atrapado.

Referencias bibliogrficas:

1. Ruiz V. Endometriosis y fertilidad. Ed. Acosta y Warman, pp. 99. 2. Lpes,VH. Palomo E. Incidencias de endometriosis en una poblacin infrtil. XXI Congreso nacional de Ginecologa y Obtetricia. Guatemala, 1993. 3. El-Eoley, et al. Danazol but not ginadotropin releasing hormone agonists suppresses autoantibodies in endomeriosis. Fertil Steril 1990; 54:725. 4. Acosta AA. Buttram VC Jr. Besch PK, Malinak LR, Van Der Heyden J. A.proposed classfication of pelvic endometriosis. Obstet Gynecol 1973;42:19. 5. Buttran VC Jr. Evolution of the revised American Fertility classification of endometriosis. Fert. Steril 1985; 43: 347. 6. Lpez VH. Tratamiento mdico-quirrgico de la endometriosis. Simposio El rostro cambiante de la endometriosis panam 3. 12. 1993. 7. Steinleitner A. Heterolous transplation of activated murine peritonel macrophages inhibitis gamete interaction in vivo; A paradigm fo endometriosis associted subfertility. Fertil Steril 1990; 54:725. 8. Damewood M. Effect of serum from patients with minimal to mild endometriosis on mouse embryo growth. Fertil Steril 1990; 54: 917. 9. Proug S. Peritoneal fluid fracctions from patients with endometriosis do not promote two-cell mouse embryo growth. Fertil Steril 1990; 54: 927.

43.- Posterior a un trabajo de parto con expulsin normal, y tras una hora aproximada en periodo de alumbramiento en el que se practic masaje uterino y se increment moderadamente la dosis de oxitocina, no aprecian signos de desprendimiento placentario, se indica una extraccin manual de placenta, que resulta imposible por no existir plano de separacin entre la placenta y la pared uterina. Cul es el diagnstico ms probable?

a) Engatillamiento placentario. b) Placenta succenturiata con cotiledn aberrante. c) Placenta circunvalata d) Placenta adherente por acretismo placentario.

Se denomina a la placenta como acreta cuando sta se implanta en zonas donde la decidua es deficiente o anormal y por tanto hay una infiltracin del miometrio por vellosidades coriales; esta infiltracin puede ser focal, parcial o total. A su vez esta condicin se subdivide en acreta, increta y percreta. La placenta increta y percreta infiltran todo el espesor de la pared miometrial, en la percreta adems las vellosidades, perforan la serosa y llegan en algunas ocasiones a infiltrar rganos vecinos, especialmente la vejiga. La PA est limitada a la superficie miometrial. La frecuencia de presentacin del AP vara entre 10 y 48 por 10.000 partos. (Oishi A 1999, Hung TH 1999, Zaki ZM, 1998). En las mujeres con acretismo placentario se han visto factores de riesgo, dentro de los cuales se encuentran: 1. 2. 3. Edad y multiparidad: La presentacin AP, aumenta con la paridad de la paciente y la edad, siendo muy rara en primparas. Placenta previa: esta se ha encontrado en el 30% de los casos de PA. Igualmente se ha visto PA en el 9.3% de las pacientes con placenta previa (Miller DA, 1997). Cesrea anterior, o cirugas uterinas previas: Se ha visto este antecedente en el 25% de los casos. En el 29% de los casos la PA estaba implantada en la cicatriz uterina y solo en el 5% la placenta estaba implantada en otro sitio. (Miller Da, 1997). Dilatacin y legrado, en el 25% de los casos. Infeccin uterina previa, remocin manual de la placenta, leiomiomas y otras anomalas uterinas: La asociacin con estas entidades es inconstante. Niveles anormalmente elevados de feto-protena y de b-HCG, en el segundo trimestrre. (Hung TH, 1999).

4. 5. 6.

Una placenta adherente o penetrante no es fcil de diagnosticar antes del alumbramiento. Despus de ste, se manifiesta como retencin placentaria y sangrado uterino. El diagnstico generalmente se realiza, despus de intentar la extraccin manual de la placenta.

Las manifestaciones clnicas propias de la placenta adherente, de la placenta acreta y de la placenta increta, consisten en una manifiesta dificultad o imposibilidad para la expulsin o extraccin de la placenta. Como consecuencia de la atona parcial y de la hemostasis insuficiente en las zonas de despegamiento placentario, se producir una hemorragia ms o menos grave que en nada se diferenciar de la hemorragia de la atena uterina. Y no ser solamente al intentar el alumbramiento artificial que se pondran de manifiesto las razones ntimas de la retencin placentaria; alumbramiento que ser engorroso en. la placenta ahderente e imposible en las variedades acreta e increta. En varias ocasiones puede no existir hemorragia y en estos casos la nica manifestacin de este estado morboso ser la prolongacin del perodo del alumbramiento. La placenta adherente, como toda placenta retenida, es pronto presa de un proceso infeccioso sin embargo se han sealado casos de placentas retenidas aspticamente durante muchos mese3. al cabo de los cuales han sido expulsadas sin causar trastorno alguno; pero hay que hacer observar que en estas enfermas se ha tratado de retensin de mebranas por abortes ovulares.

Referencias Bibliogrficas:
1. 2. 3. 4. 5. 6. 7. 8. 9. Arredondo-Soberon F, Sabella V, Garza-Leal J, Valente PT. Placenta increta en primer trimestre de embarazo. Ginecol Obstet Mex 1995; 63: 279-81. Cantanzarite V, Stanco L, Schrimmer S et al. Managing placenta previa/accreta. Contemp Obstet Gynecol 1996; 41: 66-95. Ecker JL, Sorem KA, Soodak L, et al. Placenta Increta Complicating a FirsTrimester Abortion A case report. Journal Reproductive Medicine. 1992; 37-10. Finberg G, William J. Placenta accreta: prospective sonographic diagnosis in patients with placenta previa / accreta. Contemp Obstet Gynecol 1996: 41: 66-95. Gist RS, Voung V, Brody S, Rees P, Landry AD. Placenta increta occurring in a bligter ovum. South Med J. 1996; 89(5): 545-7. Harden,MA, Walters MD, Valente PT Postabortal hemorahage due to placenta increta: A case report. Obstet Gynecol. 1990; 75: 523. Hudon L, Belfort MA, Broome DR. Dosis and management of placenta percreta: A review. Obster Gynecol survey 1998; 53: 509-517. Hung TH, Shau WY, Hsieh CC, et al. Risk factors for placenta accreta. Obstec Gynecol 1999; 93: 545-50. Kinoshita T, Ogawa K, Yusumizu T, Kato J. Spontaneous rupture of the uterus due to placenta percreta at 25-weeks gestation: a case report J Obster Gynaecol Res 1996; 22: 125-8. Kirkinen P, Helin-Martikainen HL, Vanninen R, Patanen K. Placenta accreta: imaging by gray-scaleand contrast enhanced color Doppler somography and magnetic resonance imaging. J Clin Ultrasound 1998; 26: 90-4.

10.

44.- Usted es un cirujano general y descubre que haciendo algunas modificaciones a una tcnica quirrgica disminuye el tiempo de uso de quirfano, pero quiere comparar si tiene los mismos resultados clnicos que cuando aplica la tcnica clsica , Qu tipo de estudio debe realizar? a) b) c) d) Casos y controles. Cohorte. Ensayo clnico controlado. Transversal.

Los ensayos clnicos controlados son estrategias diseadas para evaluar la eficacia de un tratamiento en el ser humano mediante la comparacin de la frecuencia de un determinado evento de inters clnico o desenlace en un grupo de enfermos tratados con la terapia en prueba con la de otro grupo de enfermos que reciben un tratamiento control.

Calva M. J. J. Estudios Clnicos Experimentales. Salud Pblica de Mxico vol. 42, nm. 4, 2000 (349).

45.- Masculino de 49 aos de edad que se queja de dolor y rigidez en las articulaciones de 3 meses de evolucin. Ambos pies y ambas manos estn calientes y las articulaciones edematizadas. Cul de los siguientes sugiere el diagnstico de artritis reumatoide?

a) b) c) d)

Rigidez matutina que dura 1 hr. Adormecimiento y palidez de los dedos al ser expuestos al fro. Afeccin simtrica de las articulaciones distales interfalngicas. Lquido sinovial con alta viscosidad y 30 mil linfocitos /mm3.

Los criterios diagnsticos establecidos por el Colegio Americano de Reumatologa, incluyen alguno de los siguientes: Presencia de artritis de ms de 6 semanas de duracin. Rigidez ar t icular mat ut ina pr olongada (+ de una hor a) Pr esencia de ndulos car act er st icos en la piel . Er osiones ar t icular es visibles por r adiologa . Posit ividad analt ica de un ant icuepo que se conoce como factor reumatoide, si bien r el 25% de los pacientes con AR nunca desarrollarn este factor y, dicho anticuerpo, puede aparecer en sujetos que no tienen AR.

BIBLIOGRAFA RECOMENDADA:

1) Klippel JH, Stone JH, Crofford LJ, White PH, editors. Primer on the rheumatic diseases. 13th ed. New York: Springer-The Arthritis Foundation; 2008. 2) Martnez-Elizondo P, editor. Introduccin a la Reumatologa. 4a ed. Mxico: Colegio Mexicano de Reumatologa A.C./Intersistemas S.A. de C.V.; 2008. 3) Firestein GS, Budd RC, Harris ED Jr, McInnes IB, Ruddy S, Sergent JS, editors. Kelleys Textbook of Rheumatology. 8th ed. Philadelphia: Saunders Elsevier; 2009.

46.- Femenino de 45 aos de edad que se encuentra en terapia intensiva por manejo de spsis abdominal. Lleva dos das con tratamiento a base de ceftriaxona, metronidazol y amikacina, el cual provee de una cobertura conveniente para los microorganismos probablemente involucrados. Despus de revisar al paciente que ha tenido una evolucin favorable, usted sugiere que se aplique el aminoglucsido en monodosis ya que:

a) b) c) c)

De esta forma aumenta su potencia. Como tiene un efecto post antibitico corto su efecto es mejor. Disminuyen sus efectos adversos. Hace sinergismo con el Metronidazol.

Una caracterstica farmacolgica de los aminoglucsidos es su actividad dependiente de concentracin, por lo que su aplicacin en bolos, con lo que se logran niveles pico ms altos, es de mayor conveniencia para aprovechar este fenmeno. Adems, el importante efecto post-antibitico que tiene, permiten que la actividad dure ms all del tiempo en que sus concentraciones se encuentran por arriba de la mnima inhibitoria, favoreciendo una posologa cmoda cada 24 horas. Por otra parte, se ha visto que los efectos adversos (nefrotoxicidad y ototoxicidad), se encuentran ms relacionados con los niveles valle. Estos se mantienen a niveles bajos cuando se utiliza el esquema en monodosis, disminuyendo la probabilidad de la presentacin de estos efectos adversos.

Kasper DL, Braunwald E, Fauci AS, Hauser SL, Longo DL, Jameson JL. Harrisons Principles of Internal Medicine. McGraw Hill. 16 Ed. 789-806 pp.

47.- Se trata de femenino de 22 aos, que presenta lesiones eritematoescamosas, edema y alguna vescula en la cara, escote, dorso de las manos y antebrazos. Las lesiones tienen 12 horas de evolucin y han aparecido tras una escursin al campo. Entre los antecedentes personales destaca acn vulgar en tratamiento con retinoides tpicos y doxiciclicina oral. El diagnstico ms probable es:

a) Erupcin lumnica poliforma. b) Eritrodermia por frmacos. c) Urticaria solar. d) Reaccin fototxica.

DEFINICIN Enfermedades cutneas que se producen por el aumento de capacidad de reaccin de la piel a las radiaciones lumnicas tras la administracin de una sustancia fotosensibilizante. Se conocen como reacciones de fotosensibilidad y pueden desencadenarse tanto por contacto como por la administracin sistmica del agente fotosensibilizante. Si existe implicacin inmunolgica se denomina dermatitis fotoalrgica y si no dermatitis fototxica.

Dermatitis fototxica No existe un mecanismo inmunolgico, puede afectar a muchas personas siempre que exista dosis elevada de irradiacin y cantidad suficiente de sustancia qumica. Las lesiones aparecen tras la primera exposicin, son monomorfas, y se caracterizan por eritema intenso, edema y vesiculacin en reas de piel fotoexpuestas, marcando claramente los bordes de las zonas descubiertas, y onicolisis ungueal . Formas particulares de fototoxia: fitofotodermatitis (dermatitis de los prados, apio) (Fig. 3), dermatitis de Berloque, fotosensibilidad en tatuajes (sulfuro de cadmio), frmacos (tetraciclinas, AINEs, amiodarona (color azulado), clorpromacina (color gris).

1. Litt Jz. Drug eruption reference manual 2001. New York: Parthenon, 2001. 2. Sullivan JR, Shear NH. Drug eruptions and other adverse drug effects in aged skin. Clinics in geriatric medicine 2002;18(1). 3. Lim HW, Gigli. Complement-derived peptides in phototoxic reaction. En: Daynes RA, Spikes JD, editors. Experimental and clinical photoimmunology. Boca Raton: CRC Press, 1983:81-93. 4. Torinuki W, Tagami H. Role of complement in chlorpromazine-induced phototoxicity. J Invest Dermatol 1986;86:142-4. 5. Hearst JE, Issacs ST, Kanne D, Rapoport H, Straub K. The reaction of the psoralens with deoxyribonucleic acid. Q Rev Biophys 1984;45:891-5. 6. Athar M, Elmets CA, Bickers DR, Mukhtar H,. A novel mechanism for the generation of superoxide anions in hematoporphyrin derivative-mediated cutaneous photosensitization. Activation of the xantine oxidase pathway. J Clin Invest 1989;83:1137-43. 7. Matsuo I, Inukai N, Fujita H, Ohkido M. Possible involvement of oxidationmof lipids in inducing griseofluvin photosensitivity. Photodermatol Photoimmunol Photomed 1990;7:2137. 8. Harber LC, Bickers DR. Photosensitivity diseadses. Principles of diagnosis and treatment. Ontario: BC Decker Inc, 1989:160-202. 9. Kockevar IE. Phototoxicity of nonsteroidal inflammatory drugs. Coincidence or specific mechanism?. Arch Dermatol 1989;125:824-6.

48.- Ante la sospecha de sndrome nefrtico agudo, Cul de los siguientes hallazgos es el ms indicativo de sndrome nefrtico agudo, en el anlisis de orina?

a) Lipiduria. b) Cilindros hemticos. c) Cilindros granulosos. d) Cilindros hialinos.

El sndrome nefrtico (SN) es un conjunto de enfermedades caracterizadas por inflamacin de los glomrulos renales con el consecuente deterioro de su funcin. La inflamacin es por lo general inmune, aunque puede resultar ser de origen infeccioso.1 Como resultado aparece una prdida sbita de sangre (hematuria) y de protenas en la orina (proteinuria) y una cada rpida del ndice de filtrado glomerular (VFG).

El sndrome nefrtico tiene mltiples causas, entre ellas causas de origen infeccioso que afectan al glomrulo, cuyo origen puede ser propiamente renal tanto como sistmico. Adems hay causas que involucran la autoinmunidad, trastornos metablicos y traumatismos entre otros. En jvenes y adolescentes Las enfermedades causales que se observan con ms frecuencia en nios y adolescentes son, entre otras: Nefropata por IgA. Prpura de Schnlein-Henoch. Sndrome urmico hemoltico. Glomerulonefritis pos-estreptoccica, el prototipo de la enfermedad nefrtica aguda.

El motivo de consulta de un paciente con sndrome nefrtico, por lo general, es referir edema, orinas oscuras, micciones de bajo volumen y poco frecuentes y, tardiamente, aparecen dolor lumbar, dificultad respiratoria y convulsiones.2 El examen fsico puede mostrar signos de retencin nitrogenada e hipertensin arterial. Hematuria En el 75% de los pacientes con un sndrome nefrtico se presenta una hematuria macroscpica y es uno de los sellos clnicos de los pacientes con este sndrome.7 Caractersticas de la hematuria de origen glomerular: Color oscuro. Total (presente en el primer, segundo y tercer chorro miccional) Indolora. Sin cogulos. Al examen microscpicos, los hemates no se observan frescos. Presencia de Cilindros Hemticos, lo cual es casi patognomnico de la hematuria glomerular.

Estos datos son caractersticos de la hematuria glomerular a la vez que la hematuria glomerular es caracterstica del sndrome nefrtico: 1. 2. (Agosto de 2007). Sndrome nefrtico agudo (en espaol). Enciclopedia mdica

Tierney, Lawrence M., McPhee, Stephen J., and Papadakis, Maxine A. Current: Medical Diagnosis and Treatment, 2003 (en ingls). Publicado por McGraw-Hill Professional Publishing, 2002; edicin 42; pg 886. ISBN 0-07-139593-8.

en espaol. Consultado el 14 de febrero, 2008.

Alan Stevens, James Lowe ANATOMIA PATOLOGICA (en espaol). Publicado por Elsevier Espaa, 2001; pg 357. ISBN 84-8174-512-X. 4. Jimenez-Murillo, Luis; A. Berlango Jimnez y F. J. Montero Prez (2004). Medicina de urgencias, 3ra edicin (en espaol), Elsevier, Espaa, pp. 493 ISBN 848174672X. 5. Kumar, Vinay; Ramzi S. Cotran y Stanley L. Robbins (2003). Patologa humana, 7ma edicin (en espaol), Elsevier, Espaa, pp. 522 ISBN 8481746665. Segn Goic. Semiologa Mdica. Santiago de Chile: Editorial Mediterrneo. (1990) p. 255-256

3.

49.- Masculino de 22 aos presenta sndrome nefrtico, la presencia de anticuerpos anticitoplsmaticos de neutrfilos con patrn citoplasmtico (C-ANCA) positivos, nos orienta hacia el diagnstico de:

a) Granulomatosis de Wegener. b) Prpura de Schnlein-Henoch c) Sndrome de Goodpasture. d) Nefritis lpica. La granulomatosis de Wegener (GW) es una vasculitis sistmica primaria que compromete predominantemente aparato respiratorio y riones. Ms de 90% tiene compromiso respiratorio alto o bajo, o ambos, presentando enfermedades como sinusitis, otitis media, hipoacusia, ulceracin de la mucosa nasal, estenosis traqueo-bronquiales, ndulos pulmonares (con o sin cavitacin) o hemorragia alveolar. El compromiso renal aparece aproximadamente en 80% de los casos, manifestndose por proteinuria, hematuria e insuficiencia renal. La afeccin ocular y neurolgica perifrica (mononeuropata mltiple) seguida de alteraciones cutneas, como lceras y prpura, le siguen en frecuencia. Se caracteriza histolgicamente por la presencia de granulomas y vasculitis necrotizante en vasos de mediano y pequeo calibre. La presencia de anticuerpos anticitoplasma de neutrfilo (ANCA), habitualmente con patrn de tipo citoplasmtico (cANCA), es un elemento de principal importancia en la orientacin diagnstica.

Referencias:
1. Ros Blanco JJ, Gmez Cerezo J, Vzquez Muoz E, Surez Garca J, Lpez Rodrguez M, Ybenes GregorioL, et al. Estudio clnico-biolgico y radiolgico de la granulomatosis de Wegener en un hospital universitario. Rev Clin Esp 2005; 205(8): 367-73.

2. Bosch X, Guilabert A, Font J. Antineutrophil cytoplasmic antibodies. Lancet 2006; 368(9533): 404-18. 3. Purriel P, Muras O, Acosta-Ferreira N, Vignale R. Granulomatosis de Wegener (a propsito de tres observaciones).

4. Harris ED, Budd RC, Genovese MC, Firestein GS, Sargent JS, Sledge CB. Kelley's Textbook of Rheumatology. 7th ed. St. Louis, Mo: WB Saunders; 2005:1361-1366.

5. Calabrese LH, Molloy ES, Duna G. Antineutrophil Cytoplasmic Antibody-Associated Vasculitis. In: Firestein GS, Budd RC, Harris ED Jr., McInnes IB, Ruddy S, eds. Kelley's Textbook of Rheumatology. 8th ed. Philadelphia, Pa: Saunders Elsevier; 2008: Chap 82.

Lectura recomendada:

Granulomatosis de Wegener, abordaje diagnstico y teraputico


Olga Vera-Lastra,a* Arturo Olvera-Acevedo,a Alejando McDonal-Vera,b Manuel PachecoRuelasc y Jos Arturo Gayosso-Rivera. Departamento de Medicina Interna del Hospital de Especialidades y Hospital de infectologa, Centro Mdico Nacional La Raza, IMSS, Mxico D.F.

50.- Hombre de 54 aos, acude a consulta por descontrol glucmico. Tiene antecedente de DM tipo 2 de 13 aos de evolucin controlada con diferentes hipoglucemiantes orales. Desde hace un mes est bajando de peso y tiene polidipsia y poliuria a pesar de tomar su tratamiento con metformn 850 mg 3 veces al da y glibenclamida tab 5 mg, 4 tabletas diarias, adems de la dieta. EF: peso 68 kg, estatura 1.70, TA 140/80, FC 96x. Glucosa: 289 mg, hemoglobina glucosilada de 11%. La conducta teraputica ms apropiada es: a) b) c) d) Aumentar dosis de glibenclamida. Iniciar insulina de accin intermedia o prolongada. Iniciar insulina rpida por requerimientos. Aadir un tercer hipoglucemiante.

SELECCIN DEL TRATAMIENTO


DIETA + EJERCICIO Metas no obeso BIGUANIDA delgado SULFONIUREA Metas s continuar Metas s

Metas no

SELECCIN DEL TRATAMIENTO


MEZCLAS SU + BG

Metas no TX COMBINADO HO da Insulina noche

Metas s Metas no INSULINA

Continuar

Medical Management of Hyperglycemia in Type 2 Diabetes: A Consensus Algorithm for the Initiation and Adjustment of Therapy.

Dieta, ejercicio, educacin y automonitoreo


HbA1c > 9%

2 hipoglucemiantes Biguanida + secretagogo Biguanida + Tiazolidinediona HO + insulina

Insulina basal o preprandial o ambas

Cheng YY A. CMAJ 2005; 172(2):213-26.

51. - A 35-year-old man complains to a physician of chronic vague gastric pain of several years duration. The pain is sometimes relieved by food. Serum immunoglobulin studies for IgG and IgA antibodies directed against Helicobacter pylori are strongly positive. Endoscopy with gastric antral biopsy demonstrates gastrids but no ulcerative lesions. H. pylori organisms are seen with special stains on the biopsy fragments. The patient is treated with a 1-week course of omeprazole (20 mg bid), plus clarithromycin and metronidazole (500 mg bid each). Which of the following is the most appropriate test to noninvasively determine whether the H. pylori has been eradicated?

a) Culture of gastric biopsy. b) Repeat qualitative IgA and IgG anjtibodies against H. pylori . c) Repeat quantitative IgA and IgG antibodies against H. pylori. d) Urea breath test.

Lahaie RG, Gaudreau C. Helicobacter pylori antibiotic resistance: trends over time. Canadian Journal of Gastroenterology. 2000;14(10):895899.

52.- Un hombre de 55 aos se le diagnostica HPB, se niega a tomar tratamiento farmacolgico y elige una RTUP, Cul es la complicacin ms frecuente de este procedimiento? a) b) c) d) Contractura de cuello vesical Impotencia Incontinencia Eyaculacin retrgrada

Reseccin Transuretral (RTU): es el gold standard de las intervenciones. __ En general es una ciruja corta. __ 90% mejora importante de los sntomas. __ Morbimortalidad baja del 0,2%. __ El 20 % debe reoperarse a los 10 aos aproximadamente. __ Complicaciones: impotencia 10%, eyaculacin retrograda 50%, incontinencia 4%.

BIBLIOGRAFIA: 1. Medina JJ, Parra RO, Moore RG. Benign prostatic hyperplasia (the aging prostate). Med Clin North Am 1999 Sep;83(5):1213-29. 2. Oesterling JE. Benign prostatic hyperplasia. Medical and minimally invasive treatment options. N Engl J Med 1995 Jan 12;332(2):99-109.

53.- Masculino de un ao con sospecha de retinoblastoma, los datos que orientan a ste diagnstico son: a) Dolor, fotofobia y lagrimeo. b) Lagrimeo, fotofobia y aumento del dimetro corneal. c) Fotofobia y quemosis conjuntival. d) Estrabismo y leucoria.

RETINOBLASTOMA TUMOR OCULAR MS FRECUENTE EN INFANCIA. 1/20.000 RN 80% en < 3 aos Uni o bilaterales. Herencia 1. AD alta penetrancia (90-95%) 2. Espordicos

Diagnstico: Leucocoria Estrabismo Mala AV Ojo rojo y doloroso Celulitis orbitaria Examen de rutina

Annals d.Oftalmologia 2001;9(2):74-92 N. Martn, MD. Coll, J. Garca, J. Snchez de Toledo, E. Trivio, M. Guitart, JJ. Gil. 1. Unidad Oftalmologa Peditrica. Hospital Maternoinfantil Vall d.Hebron 2. Departamento de biologa-celular, fisiologa e inmunologa de la Universidad Autnoma de Barcelona. 3. Servicio oftalmologa Hospital General Vall d.Hebron.

54.- Masculino de 50 aos con dolor precordial relacionado al esfuerzo, de corta duracin, de 4 meses de evolucin y con ECG en reposo normal, el siguiente estudio de eleccin es:

a) Prueba de esfuerzo con protocolo de Bruce. b) Ecocardiograma de reposo. c) Prueba de Talio- Dipiridamol. d) Ecocardiograma con estrs farmacolgico.

El propsito fundamental de la prueba es el de demostrar la existencia de isquemia miocrdica en los subgrupos de poblacin con mayor prevalencia de cardiopata isqumica o bien en los subgrupos en donde la prueba se efecta a manera de evaluacin del tratamiento en pacientes ya conocidos con cardiopata isqumica. Existe otro subgrupo de poblacin en quienes este estudio ayuda a determinar la clase funcional en la que se encuentran e incluye a pacientes con valvulopatas o con insuficiencia cardiaca de cualquier etiologa.

Protocolos de esfuerzo. El protocolo ms empleado es el de Bruce sobre treadmill, aunque existen otros protocolos y su eleccin depender de las condiciones del individuo. Los protocolos discontinuos son los que alternan periodos de esfuerzo que se intercalan con periodos de reposo de duracin similar, se emplean en escasas circunstancias. Los protocolos continuos son los que no interrumpen el esfuerzo una vez iniciado hasta finalizada la prueba, permiten mejor adaptacin fsica y psicolgica y es posible adaptar la intensidad de forma individualizada para que la prueba tenga una duracin de 6 a 12 minutos. Los protocolos mximos son los que se suspenden debido a la sintomatologa del paciente, a los signos registrados durante la prueba o se alcanzan valores mximos de FC y VO2. Los protocolos submximos son los que se suspenden cuando el sujeto alcanza un nivel determinado de carga, habitualmente el 85% de la FC mxima terica (que se encuentra entorno a los 170 lpm). En la prctica diaria, el nivel de carga (VO2) se expresa en forma de trabajo externo (MET equivalentes metablicos) que corresponden a 3,5ml/kg/min de VO2, lo que permite comparar protocolos entre s (cada protocolo dispone de frmulas para realizar el clculo de los METS), el error que cometen en el clculo de los METS es mayor en protocolos discontinuos. Emplear la FC como nico criterio para determinar el esfuerzo mximo es errneo, por lo que deberan tenerse en cuenta otros criterios, como es la percepcin subjetiva por parte del paciente mediante la escala de Borg (tabla I). Esta dificultad en la prediccin del esfuerzo mximo es lo que limita la realizacin de pruebas submximas a la determinacin de la condicin fsica de sujetos aparentemente sanos.

Tabla I. Escala de Percepcin del esfuerzo de Borg. Escala de 15 grados Valor 6 7 8 9 10 11 12 13 14 15 16 17 18 19 20 Percepcin No se siente nada Muy muy leve Muy leve Considerablemente leve Moderadamente dura Dura Muy dura Muy muy dura Esfuerzo mximo Valor 0 0,5 1 2 3 4 5 6 7 8 9 10 Muy muy fuerte (submxima) Escala de 10 grados Percepcin Nada Muy muy leve Muy leve Leve Moderada Algo fuerte Fuerte o intensa Muy fuerte

* A la izquierda la escala original de esfuerzo percibido en 15 grados (de 6 a 20) y a la derecha la ms nueva de 10 categoras. Tabla II. Indicaciones clsicas de la ergometra. I. Fines diagnsticos A. Pacientes sintomticos. 1. Dolor torcico: a) Tpico b) Atpico 2. Clnica de equivalentes isqumicos. A. Pacientes asintomticos. 1. Con alteraciones en el ECG sugestivas de isquemia. 2. Con alta probabilidad de padecer Cardiopata Isqumica (paciente con mltiples factores de riesgo) 3. Cuando convenga descartar con cierta seguridad CI. 4. Con sospecha de CI silente. 5. Sedentarios que inician programa de actividad fsica. 6. Para estudio funcional de ciertas arritmias. II. Con fines valorativos y pronsticos. 1. Seguimiento de paciente con CI conocida. 2. Tras IAM.

3. En exmenes prelaborales o laborales. 4. De la eficacia del tratamiento: Mdico. Cateterismo y angioplastia. Quirrgico. 5. Respuesta de la Tensin Arterial. 6. En valvulopatas o miocardiopatas. 7. Estudio de arritmias y trastornos de la conduccin aurculo-ventricular. 8. En cardiopatas congnitas. Bibliografa 1. Gibbons RJ (Edit.). ACC/AHA 2002 Guideline Update of Exercise Testing. 2002 American College of Cardiology Foundation and American Heart Association ACC/AHA; 2002 [Acceso 1-4-06]. Guidelines for cardiac exercise testing. ESC Working Group on Exercise Physiology, Physiopathology and Electrocardiography Eur Heart J 1993; 14: 969-988. Fernando Ars Aros F, Boraita A, Alegria E, Alonso AM, Bardaji A, Lamiel R. Guas de prctica clnica de la Sociedad Espaola de Cardiologa en pruebas de esfuerzo. Rev Esp Cardiol 2000; 53 (8): 1063-94 Chaitman B. Las pruebas de esfuerzo. En: Braunwald E, editor. Tratado de Cardiologa. Medicina Cardiovascular. 4 ed. Madrid Mc-Graw-Hill-Interamericana de Espaa; 1993. p. 177-197. Schlant RC, Friesinger GC 2nd, Leonard JJ. Clinical competence in exercise testing: A statement for physicians from the ACP/ACC/AHA Task Force on Clinical Privileges in Cardiology. J Am Coll Cardiol 1990; 16: 1061-5. Reyes Lopez de los M, Iiguez Romo A, Goicolea de Oro A, Funes Lopez B, Castro Beiras A. El consentimiento informado en cardiologa. Rev Esp Cardiol 1998; 51: 782796. Fletcher GF, Flipse T, Malouf J, Kligfield P. Current status of ECG stress testing. Curr Probl Cardiol. 1998 Jul; 23(7): 353-423. Alegra Ezquerra E, Alijarde Guimer M, Cordo Mollar JC, Chorro Gasc FJ, Pajarn Lpez A. Utilidad de la prueba de esfuerzo y de otros mtodos basados en el electrocardiograma en la cardiopata isqumica crnica. Rev Esp Cardiol 1997; 50: 6-14 Wasserman K, Hansen JE, Sue DY, Whipp BJ, Casaburi R. Principles of exercise testing and interpretation . 2 ed. Philadelphia: Lea & Febiger; 1994. p. 95-111. American college of Sports Medicine. Guideliness for exercise testing and prescription. 5 ed. Baltimore: Williams & Wilkins; 1995. Borg GA. Psychophysical bases of perceived exertion. Med Sci Sports Exerc 1982; 14: 377-381. Froelicher VF, Umann TM. Exercise testing: clinical applications. En: Pollock ML, Schmidt DH, editors. Heart disease and rehabilitation. 3 ed. Champaign, IL: Human Kinetics, 1995; p.57-79. Myers J, Froelicher VF. Exercise testing. Procedures and implementation. Cardiol Clin. 1993; 11(2): 199-213.

2. 3.

4.

5.

6.

7. 8.

9. 10. 11. 12.

13.

14. Weiner DA, McCabe C, Hueter DC, Ryan TJ, Hood WB Jr. The predictive value of anginal chest pain as an indicator of coronary disease during exercise testing. Am Heart J 1978; 96: 458-462.

55.- Masculino de 44 aos diagnosticado por trastorno obsesivo-compulsivo. Los actos obsesivos ms frecuentes en este trastorno son: a) Recuentos mentales. b) Evitar pisar las cruces de las baldosas. c) Comprobaciones y rituales de limpieza. d) Acumulacin y coleccin de objetos. Sntomas del TOC Obsesiones

Las obsesiones son ideas o impulsos no deseados que aparecen repetidamente en la mente de la persona que padece TOC. Los pacientes suelen tener miedo a sufrir dao ellos mismos, o alguien al que quieren, se preocupan irracionalmente por no contaminarse, o tienen una necesidad excesiva de hacer las cosas correctamente o con perfeccin. Una y otra vez, la persona piensa algo que le inquieta, como por ejemplo, "Mis manos pueden estar contaminadas--debo lavarlas" o "Puedo haber dejado el gas abierto" o "Estoy hacindole dao a mi hijo." Estos pensamientos angustiosos se inmiscuyen con los otros pensamientos del paciente, y causan ansiedad. A veces, las obsesiones son de carcter violento o sexual, o tienen que ver con enfermedades. Compulsiones En respuesta a sus obsesiones, la mayora de las personas con TOC recurren a comportamientos repetitivos llamados compulsiones. Los ms frecuentes son los rituales de limpieza y las comprobaciones. Otros comportamientos compulsivos incluyen recuentos (a menudo al mismo tiempo que se realizan otras acciones compulsivas, tales como lavarse las manos), hacer repeticiones, acaparamiento, y recolocaciones sin fin de objetos en un esfuerzo para mantenerlos perfectamente alineados. Tambin son corrientes los problemas mentales, tales como repetir frases mentalmente y hacer listas. Estos comportamientos, en general, tienen por objeto proteger de peligros a la persona que padece TOC, o a los otros. Algunas personas con TOC tienen rituales establecidos; otros tienen rituales que son complejos y cambiantes.

El ejecutar estos rituales slo proporciona alivio temporal de la ansiedad, pero no hacerlos incrementa la ansiedad de la persona.

56.- Masculino de 17 aos, que presenta hemartrosis. Tiene TPT alargado, TP y TT normales, por lo que tiene una alteracin de la va: a) Del Complemento b) Intrnseca c) Extrnseca d) Colinrgica

Manual CTO 7 edicin, p. 723. La va intrnseca est constituida por la activacin secuencial de los factores XII, XI, IX,. VIII y V. el tiempo de tromboplastina parcial activada mide la actividad de la coagulacin intrnseca y sirve para monitorizar el tratamiento con la heparina no fraccionada.

57.- Su frecuencia es de 1/10,000, bajo peso al nacimiento, LPH (Labio paladar hendido) microftalmia, holoprosencefalia, aplasia cutis vertex, riones poliqusticos, malrotacin del colon, y en nias puede existir tero bicorne, son manifestaciones de:

a) b) c) d)

Trisoma 21 Trisoma 13 Trisoma 18 Trisoma 11

La trisoma 13 representa la tercera aneuploida autosmica viable ms frecuente en la especie humana, slo superada en frecuencia por la trisoma 21 y trisoma 181. Aunque su etiologa es an desconocida, estudios citogenticos en individuos afectados han podido determinar que aproximadamente un 75% de los casos corresponden a no-disyuncin meitica, un 20% a translocaciones y un 5% a mosaicismo debido a nodisyuncin postcigtica.

La expresin fenotpica de la trisoma 13 es caracterstica y consiste en anomalas faciales, esquelticas y del sistema nervioso central, siendo tambin frecuentes las malformaciones estructurales del aparato cardiovascular, genitourinario y gastrointestinal (Tabla I).1 La mayora de los defectos estructurales asociados a trisoma 13 pueden ser identificados prenatalmente por ultrasonografa.2-6 Esto permite ofrecer estudio citogentico prenatal en casos seleccionados, establecer el diagnstico definitivo y manejar el embarazo afectado en forma racional, ya que esta anomala se asocia a una mortalidad perinatal cercana al 100%.

58.- Un hombre de 47 aos de edad afebril con antecedente de pancreatitis crnica, presenta una masa abdominal palpable y amilasa srica persistentemente elevada. La mayor probabilidad de diagnstico es?

a) b) c) d)

Cistadenoma pancretico. Pseudoquiste pancretico. Carcinoma pancretico. Coleccin pancretica aguda

El pseudoquiste pancretico es una coleccin de jugo pancretico localizada, por lo general, en el interior o alrededor del parnquima pancretico. El pseudoquiste pancretico est confinado por una capa no epitelializada de tejido necrtico, fibrtico y de granulacin, que se desarrolla tras una lesin pancretica. Para su formacin requiere un mnimo de cuatro semanas desde que ese dao se produce. El pseudoquiste pancretico es una complicacin tanto de la pancreatitis aguda como de la crnica. Si bien la mayora de los pseudoquistes pancreticos se localizan en la cabeza y el cuerpo del pncreas, hasta un 20% de los mismos son extrapancreticos (1). Se han descrito pseudoquistes pancreticos en mltiples localizaciones, como cavidad pleural, mediastino y pelvis (2). Se presenta el caso de un pseudoquiste pancretico de localizacin heptica que apareci en el curso de una agudizacin de una pancreatitis crnica, y que se resolvi sin necesidad de drenaje. Diagnstico: Manifestaciones Clnicas:

1. 2.

Pacientes en la cuarta o quinta dcada de vida, antecedentes etiolgicos. Pacientes con pancreatitis aguda que no resuelve luego de 5 a 7 das de tratamiento o luego de mejora recae. 3. Sensacin de cuerpo extrao y pesadez en la mitad superior del abdomen. 4. Si pancreatitis crnica, dolor abdominal o sntomas por compresin de vscera. 5. Nauseas, vmitos y prdida de peso por obstruccin duodenal.

6.

Ictero, si compresin del coldoco.

7. 8.

Masa en abdomen superior, lisa y dura, muchas veces insensible. Ms raramente ascitis y derrame pleural.

Complementarios Diagnsticos:

1. 2.

Ultrasonografa Abdominal: Muchas veces diagnstico, preferido para vigilancia. TAC: Ideal para diagnstico.

Bibliografa
1. Hamm VB, Franzen N. Atypically located pancreatic pseudocyst in liver, spleen, stomach wall and mediastinum: their CT diagnosis. Rofo 1993; 159 (6): 522-7. 2. Vitas GJ, Sarr MG. Selected management of pancreatic pseudocyst: Operative versus expectant management. Surgery 1992; 111 (2): 123-30. 3. Mofredj A, Cadranel JF, Dautreaux M, Kazerouni F, Hadj-Nacer K, Deplaix P, et al. Pancreatic pseudocyst located in the liver: A case report and literature review. J Clin Gastroenterol 2000; 30 (1): 81-3. 4. Balzan S, Kianmanesh R, Farges O, Sauvanet A, O'toole D, Levy P, et al. Right intrahepatic pseudocyst following acute pancreatitis: an unusual location after acute pancreatitis. J Hepatobiliary Pancreat Surg 2005; 12 (2): 135-7.

59.- Masculino de 70 aos, diagnosticado de carcinoma de prstata, en este momento acude al servicio de urgencias por presentar confusin mental, nuseas, vmitos y estreimiento. Se realizan pruebas de laboratorio y destaca una calcemia de 16mg/Dl. Cul es la primera medida teraputica que debera tomar ante ste paciente? a) Hormonoterapia (leuprolide y estrgenos). b) Difosfonatos por va oral. c) Glucocorticoides por va intravenosa. d) Administracin de Solucin salina y furosemida por va intravenosa.

Los sntomas que provoca la hipercalcemia estn en relacin con sus niveles en sangre. Valores comprendidos entre 10,5 y 12 gr/dl no suelen provocar sntomas; a partir de estos valores la sintomatologa es progresiva, afectando con ms intensidad y gravedad a todos los sistemas del organismo. No est establecido un orden de aparicin de la sintomatologa, ni su correspondencia con los valores de calcemia. Sntomas de la hipercalcemia Sistema nervioso central Desrdenes mentales Dificultades cognitivas Ansiedad Depresin Confusin, estupor y coma Calcificacin corneal Suicidios (descritos aisladamente) Sistema neuro-muscular Fatiga o cansancio muscular Mialgias Descenso de la funcin de msculos respiratorios Laxitud articular Sistema renal Nefrolitiasis Diabetes inspida nefrognica (poliuria y polidipsia) Deshidratacin Nefrocalcinosis Sistema gastrointestinal Nuseas y vmitos Anorexia Estreimiento Dolor abdominal Pancreatitis lcera pptica Sistema esqueltico Dolor seo Artritis Osteoporosis Ostetis fibrosa qustica Resorcin subperistica Quistes seos Embarazo Hipoparatiroidismo neonatal Tetania neonatal Bajo peso al nacer

Retraso crecimiento intrauterino Hiperemesis gravdica

Alta morbilidad neonatal y materna Partos pretrmino Sistema cardiovascular Hipertensin arterial Calcificacin vascular Calcificacin miocrdica Hipertrofia miocrdica Acortamiento intervalo QT Arritmias cardacas Otros Queratitis Conjuntivitis Anemia normoctica normocrmica Gota o pseudogota

El tratamiento de la hipercalcemia est orientado fundamentalmente a eliminar la causa que la produce. En los casos sintomticos se requiere de un tratamiento inicial especfico. Debe ser tratada la que presenta sntomas o supera los 14 mg/dl. Los mecanismos para actuar sobre la hipercalcemia son: Aumento de la eliminacin renal de calcio. Disminucin de la absorcin intestinal de calcio. Disminucin de la resorcin sea de calcio. El tratamiento siempre debe iniciarse con la rehidratacin del paciente, que produce un aumento del volumen extracelular, alcanzando una ganancia de volumen de 1,5- 2,5 litros en las primeras 24 horas. A continuacin, se utilizar un diurtico de asa, tipo furosemida, que aumenta la excrecin renal de sodio y calcio. La dosis de diurtico se ajustar en funcin de las cifras de calcio a lo largo del tratamiento. Cuando la funcin renal est comprometida puede recurrirse a la hemodilisis con calcio bajo en el lquido de dilisis. En los casos en los que est implicado un aumento de la resorcin sea como causa de hipercalcemia, deberemos controlarla con: Bisfosfonatos: Disponemos de clodronato y pamidronato. El ms empleado es el primero, porque la dosificacin es ms fcil. Su administracin en el caso de la hipercalcemia es intravenosa, inicia el efecto a las 72 horas y alcanza el mximo a la semana.

Calcitonina: Acta ms rpido que los bisfosfonatos, pero slo mantiene el efecto durante 72 horas. Otros frmacos antirresortivos: Mitramicina y el nitrato de galio, con menor experiencia, pueden usarse en caso de fracaso de los anteriores.

En aquellos casos en los que est aumentada la absorcin intestinal de calcio, como en la produccin endgena de vitamina D (enfermedades granulomatosas o linfomas) deben tratarse con glucocorticoides. LECTURAS RECOMENDADAS: 1. Achogue HJ, Madias EN. Changes in plasma potassium concentration during acute acid base disturbances. Am J Med 71:456, 1981.

2. Humphreys MH. Urgencias por trastornos en los lquidos, electrolitos y equilibrio cido bsico. En: Diagnstico y Tratamiento de Urgencias. Editado por MT Ho y CE Saunders. Editorial El Manual Moderno. Mxico DF, 1991.

3. Mora JM, Delgado VA, D'Achiardi R. Trastornos del potasio. En: Manual de Urgencias en Medicina Interna. Captulo Central. Ediciones Act. Med. Colomb. Santaf de Bogot, 1994. 4. Silva E, Lpez C, Ramrez J. Trastornos del potasio. En: Compendio de Teraputica. Segunda edicin. Asociacin Colombiana de Medicina Interna. Captulo Central. Ediciones Act Med Colomb. Santaf de Bogot, 1992.

60.- Una mujer de 43 aos inicia recientemente con fatiga, somnolencia, piel seca, estreimiento y aumento de peso de 5 kg. Su tiroides est firme y tiene el doble del tamao normal. Cul de las siguientes pruebas de laboratorio confirma el presunto diagnstico de hipotiroidismo?

a) b) c) d)

Tiroxina srica (T4) Triyodotironina srica (T3) Captacin de resina T3 Hormona estimulante de tiroides (TSH) en suero

Hipotiroidismo La instauracin es habitualmente lenta y progresiva. Los sntomas se relacionan con una disminucin en la actividad funcional de todos los sistemas del organismo. Los ms clsicos son cansancio, intolerancia al fro (carcter muy friolero), apata e indiferencia, depresin, disminucin de memoria y de la capacidad de concentracin mental, piel seca, cabello seco y quebradizo, fragilidad de uas, palidez de piel, aumento de peso, estreimiento pertinaz y somnolencia excesiva. En situaciones extremas puede evolucionar hacia la insuficiencia

cardiaca, la hinchazn generalizada (mixedema), insuficiencia respiratoria y abocar al coma mixedematoso con prdida de conocimiento que conlleva un alto grado de mortalidad.

Al igual que el resto de enfermedades del tiroides, el hipotiroidismo es ms frecuente en el sexo femenino. Es a partir de los 40-50 aos cuando las mujeres tienden a desarrollar con ms frecuencia hipotiroidismo de causa autoinmune (tiroiditis de Hashimoto). El periodo postparto es igualmente propenso a la aparicin de este problema. La ciruga de tiroides y la aplicacin de yodo radioactivo representan situaciones de riesgo para el desarrollo de hipotiroidismo, lo que obliga a controlar evolutivamente la funcin tiroidea en estos casos. Los recin nacidos de madres hipertiroideas, hayan recibido o no tratamiento antitiroideo durante la gestacin, deben ser evaluados en este sentido. Las personas en las que se detectan anticuerpos antitiroideos (antimicrosomales, antitiroglobulina) tienden a desarrollar con el tiempo alteraciones de la funcin tiroidea, por lo que deben ser evaluados crnicamente de forma peridica. La determinacin de TSH es el parmetro ms sensible para el diagnstico del hipotiroidismo. Su elevacin es indicativa de que la funcin del tiroides es insuficiente. Este fenmeno se produce antes de que comiencen a descender en la sangre las concentraciones de hormonas tiroideas. Generalmente, en el hipotiroidismo establecido, adems de la elevacin de TSH, se produce un descenso de T4. El nivel de T3 con frecuencia se encuentra dentro de la normalidad. As pues, cuando aparecen sntomas sugestivos, el mdico solicitar una determinacin de TSH que es el mejor mtodo para descartar que exista hipotiroidismo. Puede acompaarse de una determinacin de T4 y de anticuerpos antitiroideos si se desea conocer si la causa se debe a fenmenos de autoinmunidad. En los casos de hipotiroidismo secundario debido a disminucin de la secrecin de TSH por parte de la hipfisis, el diagnstico se basa en confirmar concentraciones disminuidas de T4 y TSH en la sangre. Cuando la elevacin de TSH se acompaa de niveles normales de T4 la condicin es conocida con el nombre de hipotiroidismo subclnico. Si existe bocio puede ser conveniente realizar una ecografa tiroidea. Cuando existe sospecha de alteraciones en el desarrollo de la glndula o de deficiencia enzimtica, puede ser til obtener una gammagrafa tiroidea. Si se confirma un diagnstico de hipotiroidismo de causa autoinmune, es habitual evaluar la asociacin de alteraciones en otras glndulas como las suprarrenales, paratiroides o gnadas. REFERENCIAS: 1. Anderson R, Harnes J. 1975. Thyroid hormones secretion rates in growing and mature goats. J Anim Sci 40: 11301135.

2. Anke M, Henning A, Grun M, Partschefeld M, Groppel B. 1977. Der einluss des mangan, zink, kupfer, jod, selen, molybdan und nickelmangels aauf die fortpflanzuggsleistung des wiederkauers. Mathem Natur Reihe (Leipzig) 26: 283-292.

3. Balbuena O. 2003. Nutricin Mineral del Ganado. Sitio Argentino de Produccin Animal: 1-5, www.produccionanimal. com.ar.

4. Beckett GJ, Beddows SE, Morrice PC, Nicol F, Arthur JR. 1987. Inhibition of hepatic deiodination of thyroxine is caused by selenium deficiency in rats. Biochem J 248: 443447. 5. Brem JJ, Pochon DO, Roux JP, Trulls H. 1998. Exploracin diagnstica de la funcin tiroidea en ovinos. Rev Vet 8/9: 23-26.

6. Castillo V. 2001. Cambios de la funcin tiroidea en cachorros alimentados con dietas comerciales con alto contenido de yodo. On line: http://www.idealibrary.com.

61.- Un nio de 5 aos no inmunizado acude a consulta con una historia de 2 semanas de evolucin de tos paroxstica, fiebre de bajo grado, emesis post-tos y descarga nasal viscosa. El EF revela otitis media bilateral y conjuntivitis hemorrgica. Se auscultan estertores inspiratorios bilateralmente. BH con LT 45 000, con 95% de linfocitos. Cul de los siguientes es el diagnstico ms probable?

a) b) c) d)

Neumona por Chlamydia Tosferina Bronquiolitis Neumonitis por VSR

La tos ferina es una enfermedad infecciosa altamente contagiosa causada por un bacilo gram (-) llamado Bordetella pertussis. El comienzo suele ser insidioso con una fase catarral, con tos irritante que poco a poco se vuelve paroxstica, por lo regular en el trmino de una a dos semanas, y que dura de uno a dos meses o ms. Los paroxismos se caracterizan por accesos repetidos y violentos de tos; cada serie de ellos comprende innumerables toses sin inspiracin intermedia y puede ser seguida por un estridor respiratorio de tono alto caracterstico. Los paroxismos con frecuencia culminan con la expulsin de mucosidades claras y adherentes, a menudo seguida de vmito. Los lactantes menores de 6 meses de edad, los adolescentes y los adultos frecuentemente no tienen el cuadro tpico de estridores o tos paroxstica. Resumen: Etiologa: Bordetella pertussis.

Otros: Bordetella parapertussis, Bordetella bronchiseptica, Mycoplasma pneumoniae, Chlamydia trachomatis, Chlamydia pnuemoniae y adenovirus. Fuentes de contagio adolescentes y adultos. Infeccin intradocimiciliaria en el 80 % de los no vacunados. Contagio: Fase catarral y hasta 2 semanas de iniciada la tos. Periodo de incubacin de 10 a 21 das.

Cultivo nasofarngeo Bordet-Gengou (Dacrn o alginato de calcio). Negativo en fase temprana, > 4 semanas sin estaba vacunada y > 5 das si recibi tratamiento. Leucocitosis con linfocitosis absoluta. Tratamiento: Apoyo: Apnea, hipoxia y otras complicaciones. Antibitico en fase catarral es efectivo. Frenar la propagacin del germen. Eritromicina 40-50 mgkgd cada 6 hrs. x 14 das. Claritromicina 15-20 mgkgd cada 12hrs. x 7 das. Azitromicina 10-12 mgkgd cada 24 hrs. x 5 das. TMP SMZ 8 mgkgd cada 12 hrs. x 14 das.

Bibliografa:
Abul K. Abbas, Andrew H. Lichtman, Jordan S. Pober. Inmunologa Celular y Molecular. McGrall-Hill Interamericana. Cuarta edicin 2001. Napolen Gonzlez Saldaa y Mercedes Macas Parra. Vacunas en Pediatra. McGrall-Hill. Interamericana. Primera edicin 1999. Report of the Committee on Infectious Diseases. American Academy of Pediatrics. Red Book 2000. 25 edicin 2000. Stanley A. Plotkin, Walter A. Orenstein. Vaccines. W. S. Saunders Company. Tercera edicin 1999.

62.- Cual de las medidas siguientes es ms adecuada para combatir la hiperbilirrubinemia (l0mg/dl) de un lactante de 3 semanas, con un desarrollo y crecimiento normales, que recibe lactancia materna?

a) b) c) d)

Fototerapia Exanguineotransfusin Fenobarbital Esperar un par de das y repetir la prueba.

La fototerapia es el empleo de luz visible para el tratamiento de hyperbilirubinemia en el recin nacido (RN). Esta terapia relativamente comn baja el nivel de bilirrubina en el suero por transformacin de la bilirrubina en ismeros solubles en agua que pueden ser eliminados sin la conjugacin en el hgado.

La fototerapia convierte la bilirrubina que est presente en los capilares superficiales y espacio intersticial a ismeros solubles en agua que son excretables sin pasar por el metabolismo del hgado (Fig. 4). Maisels, un notable experto en bilirrubina, sugiere que la fototerapia se parece mucho a una droga percutanea. Cuando la fototerapia ilumina la piel, una infusin de fotones de energa, como molculas de una medicina, es absorbida por la bilirrubina de la misma manera que una molcula de medicina se une a un receptor. Las molculas de bilirrubina en la piel expuestas a la luz sufren las reacciones fotoqumicas relativamente rpido, configurational isomerization, isomerizacin

estructural, y la forma de fotooxidacin no txica, ismeros excretables. Estos ismeros de bilirrubina tienen formas diferentes del ismero natal, son ms polares, y pueden ser excretados del hgado en la bilis sin sufrir la conjugacin o requerir transporte especial para su excrecin. La eliminacin urinaria y gastrointestinal son ambas importantes en reducir la carga de bilirrubina.

Figura 4. El mecanismo de fototerapia. Cuando las molculas de bilirrubina absorben la luz, 2 reacciones fotoqumicas principales ocurren: el natural 4Z, 15Z-bilirubin se

convierte a 4Z, 15E bilirubin (tambin conocido como photobilirrubina) y a lumirrubina. A diferencia de 4Z, 15Z la bilirrubina, photobilirrubina puede ser excretado va heptica sin la conjugacin, pero su clearance es muy lento, y su conversin es reversible. En el intestino (lejos de la luz), photobilirrubina es convertida atrs a bilirubina natal. La lumirrubina no es reversible. Aunque mucho menos lumirrubina que photobilirrubina es formado, lumirrubina es eliminado del suero mucho ms rpidamente, y es probable que la formacin de lumirrubina es principalmente responsable de la disminucin en el suero de la bilirrubina. Las pequeas cantidades de bilirrubina natal tambin son oxidadas a monopyrroles y dipyrroles que pueden ser excretados en la orina. Esto es un proceso lento y slo un contribuidor menor a la eliminacin de bilirrubina durante la fototerapia. Cortesa de diagrama de Mara Puchalski.

El objetivo de la fototerapia es disminuir la bilirrubina srica y prevenir su acumulacin txica en el cerebro, donde puede causar serias complicaciones neurolgicas

permanente conocido como kernicterus. La fototerapia ha reducido enormemente la necesidad de exanguneo transfusin para tratar la hiperbilirrubinemia.

La fototerapia es usada de 2 modos principales: profilctica y teraputicamente. En RN prematuros o aquellos con un conocido proceso hemoltico, a menudo es usado profilacticamente, para prevenir un rpido aumento de la bilirrubina srica. En pretrminos pequeos o RN de trmino, es administrada en dosis teraputicas para reducir niveles de bilirrubina excesivos y evitar el desarrollo de kernicterus. La fotoisomerizacin de bilirrubina comienza casi al instante cuando la piel es expuesta a la luz. A diferencia de la bilirrubina no conjugada, los fotoproductos de estos procesos no son neurotxicos. Por lo tanto, ante una hiperbilirrubinemia severa del RN, es importante comenzar la fototerapia sin retraso.

Referencias:
1. Stokowski LA. Early recognition of jaundice and kernicterus. Adv Neonatal Care 2002;2:101-114. 2. Maisels MJ. A primer on phototherapy for the jaundiced newborn. Contemp

Pediatr. 2005; 22(6): passim. (OR, 38, 40, 44, 47, 48, 53, 54,57).
3. McDonagh AF. Phototherapy: from ancient Egypt to the new millenium. J

Perinatol 2001;21:S7-S12.
4. Maisels MJ. Phototherapy-traditional and nontraditional. J Perinatol 2001; 21(Suppl 1):S93-S97.

63.- Femenino de 34 aos que inicia tratamiento con sulfato de magnesio por presentar eclampsia, se presentan signos de sobre dosificacin Qu antdoto se debe emplear?

a) Gluconato clcico b) Nitroprusiato. c) Simpaticomimticos. d) Carbonato sdico.

NIVEL DE PRIMER CONTACTO (ATENCION PRIMARIA) Se debe instruir a todas las embarazadas que deben acudir inmediatamente a un centro de salud en cualquiera de los siguientes casos: Edema que se desarrolla rpidamente (en pocos das) Cefalea severa y persistente Dolor en la regin abdominal superior Visin borrosa

Se debe realizar la medicin de la presin arterial y un anlisis de orina para la deteccin de proteinuria a las mujeres que acudan a centros de salud presentando estos sntomas.

Convulsiones
Si se asiste a una mujer con eclampsia en un centro de atencin primaria: 1. Deben mantenerse las vas respiratorias permeables. 2. Se debe colocar a la mujer de costado (posicin decbito lateral izquierda) para evitar la aspiracin del vmito u otras secreciones. 3. Si es posible, se debe establecer una va intravenosa. 4. Se debe administrar sulfato de magnesio. Monitoreo de la administracin de sulfato de magnesio: Durante el tratamiento con sulfato de magnesio, se recomienda realizar un control cada 4 horas, como mnimo, para detectar la presencia de: Reflejo rotuliano, frecuencia respiratoria superior a 16 por minuto, volumen de orina >100 ml en las 4 horas previas. Sobredosis de sulfato de magnesio: Todo centro de salud que utilice sulfato de magnesio debe disponer de ampolletas de gluconato de calcio (1 g) como antdoto para la sobredosis de dicho frmaco.

Se sugiere medir la presin arterial y administrar antihipertensivos segn corresponda. Convulsiones recurrentes: en caso de convulsiones recurrentes, se administran otros 2 a 4 g de sulfato de magnesio por va IV en el lapso de 5 minutos, tanto para el rgimen IM como el IV; la dosis se determina en funcin del peso de la paciente.

El sulfato de magnesio es un frmaco usado en el control de las convulsiones eclmpticas, para suprimir o controlar las contracciones uterinas sean estas espontneas o inducidas, y como broncodilatador luego del uso de beta agonistas y agentes anticolinergicos. Tambin tiene indicacin como terapia de reemplazo en la deficiencia de magnesio, como laxante para reducir la absorcin de txicos del tracto gastrointestinal. El sulfato de magnesio esta ganando popularidad como tratamiento de inicio en el manejo de algunas arritmias, particularmente en arritmias secundarias a sobredosis de antidepresivos tricclicos o toxicidad digitlica. Esta tambin considerado clase Ila (probable beneficio) para la fibrilacin ventricular refractaria y la taquicardia ventricular, luego de la administracin de dosis de lidocaina y bretilio.

FARMACODINAMIA
El sulfato de magnesio tiene la capacidad de alterar la excitabilidad de la fibra miometrial, afecta el acoplamiento excitacin contraccin y el proceso mismo de contraccin, inhibe la entrada de calcio al sarcoplasma y reduce la frecuencia de los potenciales de accin. Inhibe tambin la liberacin de acetilcolina. Por ser estas acciones comunes en las fibras musculares se pueden ver afectadas tambin la musculatura voluntaria e incluso las fibras miocrdicas.(1)

Bibliografa:
Graves C. Frmacos que contraen o relajan el tero. En: Hardman J, Limbird L, Molinoff P, Ruddon R, Goodman A, eds. Goodman & Gilman. Las Bases Farmacolgicas de la Teraputica. 9 ed. Mxico DF: McGraw-Hill Interamericana; 1996. pp. 1012-3.

64.- Femenino de 43 aos, gesta-5, partos-3, abortos-1, con diagnstico de anemia ferropnica, de 9.5 g/dl, refiere ciclos menstruales de 31,32 x 8,9 das de duracin, acompaados de cogulos, los cuales aparecieron despus del nacimiento de su segundo hijo hace 13 aos. E.F.: Buen estado general, TA 130/80, genitales con evidencia de sangrado activo, al tacto vaginal se detecta tero de consistencia firme voluminosa, irregular, aproximadamente de 12 cm. anexos libres. El diagnstico ms probable es:

a) Adenomiosis uterina. b) Cncer cervicouterino. c) Miomatosis uterina. d) Hiperpalsia adenomatosa de endometrio.

MIOMATOSIS UTERINA Definicin: Tumor benigno que se origina en el miometrio, por lo que su componente histolgico predominante es el tejido muscular y, en menor medida, el conectivo y fibroso. El nico tratamiento efectivo es el quirrgico; sin embargo, slo requieren ser tratados aquellos que producen sntomas.

Evaluacin y Diagnstico: Historia: 1. 2. 3. 4. El sntoma ms frecuente suele ser la hemorragia uterina. Los sntomas principales estn relacionados con el crecimiento del tumor. La paciente puede notar una masa en hipogastrio o abdomen inferior. La masa se puede asociar a dolor plvico, o manifestaciones por compresin de rganos o estructuras vecinas. 5. Puede haber alteracin de la fertilidad.

Examen Fsico: 1. Se debe realizar con la vejiga y el recto vacuo. 2. El hallazgo primordial es el aumento de volumen y consistencia del tero, el cual puede ser simtrico (ndulos submucosos) o irregular (ndulos intramurales o subserosos). Exmenes Auxiliares: 1. Papanicolau crvicovaginal: Indicado siempre; permite descartar neoplasia epitelial cervical o cncer infiltrante de crvix.

2. Ultrasonido transabdominal y transvaginal: Indicado siempre; permite evaluar la localizacin, tamao y nmero aproximado de miomas. 3. Hemoglobina, hematocrito: Indicado cuando hay historia de sangrado; orienta en la severidad del sangrado y anemia. 4. Hemograma y VSG: Indicado cuando hay historia de fiebre; si es anormal sugiere infeccin o necrosis del mioma (puede ser apropiado descartar infeccin de otro rgano o sistema). 5. Grupo sanguneo y factor Rh: Si hay anemia severa o en el preoperatorio. 6. Perfil de coagulacin (tiempo de protrombina y de tromboplastina parcial, recuento de plaquetas): Si hay historia de sangrado exagerado. 7. Gonadotrofina corinica (subunidad sr ica): Per mit en casos de ciclos irregulares, retraso menstrual o tero de consistencia blanda. Diagnstico Diferencial: 1. 2. 3. 4. 5. 6. Embarazo. Tumor de ovario. Enfermedad inflamatoria plvica, complejo inflamatorio anexial plvico. Endometriosis. Adenomiosis. Tumor extragenital: Colon, retroperitoneo.

Referencias Bibliogrficas: Hillard PA. Benign Diseases of the Female Reproductive Tract: Symptoms and Signs. En: Berek JS, Adashi EY, Hillard PA, eds. Novak's Gynecology. Baltimore: Williams and Wilkins, 1996:331-97. Hutchins FL, Greenber MD. Miomas Uterinos: Diagnstico e Indicaciones de Tratamiento. Clinicas de Ginecologa y Obstetricia. Temas Actuales. 1995;5:609-14. Davis KM, Sclass WD. Tratamiento Mdico para Miomatosis Uterina. Clinicas de Ginecologa y Obstetricia. Temas Actuales. 1995;5:671-81. Selwyn P, Oskowitz MB. Leiomyomata Uteri. En: Friedman EA, ed. Gynecological Decision Making. St. Louis: Mosby, 1983:148-9. Diaz Huamn V. Tumores Benignos del Aparato Reproductor Femenino. En: Ludmir A, Cervantes R, Castellano C, eds. Ginecologa y Obstetricia, Prevencin Diagnstico - Tratamiento. Lima: Concytec, 1996:907-25.

65.- Para valorar que tan fuerte es la asociacin entre una exposicin y una enfermedad, Cul sera la medida de asociacin a calcular? a) b) c) d) Riesgo relativo Tasa de mortalidad Incidencia Prevalencia

El Riesgo Relativo mide la fortaleza de una asociacin entre un factor y un cierto resultado final; de este modo, un Riesgo Relativo orienta hacia causalidad y es util para investigar el origen de una enfermedad.

Morton R. F. Bioestadstica y Epidemiologa, Interamericana, 3. Ed. 1 993; pg: 36.

66.- Masculino de 25 aos, que 10 das despus de acudir a una despedida de soltero, comienza con inflamacin de rodilla derecha y de ambos tobillos, conjuntivitis bilateral, aftas orales y erosiones superficiales no dolorosas en el glande. El diagnstico ms probable es:

a) b) c) d)

Enfermedad de Still. Infeccin gonoccica. Infeccin por Staphylococcus Aureus. Enfermedad de Reiter.

Proceso inflamatorio estril de la membrana sinovial, precedido o precipitado por una infeccin que ocurre fuera de la articulacin.

DIAGNOSTICO: HISTORIA SNTOMAS GENERALES MANIFESTACIONES MSCULO-ESQUELTICAS Artralgias, artritis aditiva o migratoria Monoartritis u oligoartritis asimtrica Articulaciones grandes que sostienen peso: rodillas, tobillos y caderas Dactilitis o dedos en salchicha Afeccin axial: articulaciones S-I y columna lumbar Entesopata, tenosinovitis MANIFESTACIONES GENITO-URINARIAS Uretritis, balanitis circinada (es importante sealar que las lesiones son indoloras) , prostatitis Cervicitis, cistitis, enfermedad plvica inflamatoria MANIFESTACIONES CUTNEAS Y DE MEMBRANAS MUCOSAS: Queratodermia blenorrgica Eritema nodoso Distrofia ungueal lceras orales

MANIFESTACIONES OCULARES Conjuntivitis y uvetis. MANIFESTACIONES G-I

BIBLIOGRAFA RECOMENDADA

Klippel JH, Stone JH, Crofford LJ, White PH, editors. Primer on the rheumatic diseases. 13th ed. New York: Springer-The Arthritis Foundation; 2008. Martnez-Elizondo P, editor. Introduccin a la Reumatologa. 4a ed. Mxico: Colegio Mexicano de Reumatologa A.C./Intersistemas S.A. de C.V.; 2008. Firestein GS, Budd RC, Harris ED Jr, McInnes IB, Ruddy S, Sergent JS, editors. Kelleys Textbook of Rheumatology. 8th ed. Philadelphia: Saunders Elsevier; 2009.

67.- Un hombre de 29 aos con historia de actividad homosexual hace 5 aos y HIV-1 positivo con tratamiento con HAART , el cual hace 8 meses suspende el medicamento por sentirse bien y que ha presentado hace dos meses una neumona por neumocictis carinii , en caso de presentar una infeccin del SNC por papovavirus tiene un riesgo de producir:

a) b) c) d)

Adrenoleucodistrofia. Esclerosis mltiple. Panencefalitis subaguda esclerosante. (SSPE) Leucoencefalopatia multifocal progresiva. (PML)

La leucoencefalopata multifocal progresiva (LMP) es una enfermedad de etiologa viral (virus JC, Papovavirus), que afecta de forma subaguda o crnica la sustancia enceflica provocando una desmielinizacin progresiva de sta. Se observa en pacientes jvenes, portadores de enfermedades inmunosupresoras (infeccin por VIH etapas finales, enfermedades linfoproliferativas, tratamientos inmunosupresores, etctera). El diagnstico de LMP se plantea ante un paciente inmunodeprimido, que presenta un cuadro de deterioro cognitivo o dficit focales (hemiparesia, alteraciones campimtricas, alteraciones de la coordinacin y del equilibrio, etctera), o ambos, de curso progresivo, en

cuestin de semanas o pocos meses, infrecuentemente acompaado de cefaleas y sin fiebre. Este cuadro lleva inexorablemente a la muerte, aunque se han reportado casos anecdticos de detencin y reversibilidad de la enfermedad (7). La tomografa computarizada (TC) enceflica puede ser normal al inicio de la enfermedad, o mostrar lesiones hipodensas de la sustancia blanca, a menudo confluentes, ms frecuentemente ubicadas en las regiones frontales y parietooccipitales que no se realzan con el contraste, no producen efecto de masa y respetan la sustancia gris cortical. Leucoencefalopata multifocal progresiva Rev. Med. Uruguay 2003; 19: 78-82.

Bibliografa:
1 . Corradi H. Leucoencefalopata multifocal progresiva. In: Salamano R (coord). Temas de Neuroinfectologa. Montevideo: Oficina del Libro-AEM, 1998: 105-8. 2 . Astrom KE, Mancall EL, Richardson EP Jr. Progressive Multifocal Encephalopathy. Brain 1958; 81: 930.

3 . Cavanagh JB, Greenbaum D, Marshall AHE, et al. Cerebral Demyelination associated with disorders of reticuloendothelial system. Lancet 1959; 2: 525. 4 . Richardson EP Jr. Progressive Multifocal Leukoencephalopathy. N Engl J Med 1961; 265: 815. 5 . Padgett BL, Walker DL, Zu Rheim GM, Eckroade RJ, Dessel BH. Cultivation of papova-like virus from human brain with progressive multifocal leucoencephalopathy. Lancet 1971; 1: 1257-60.

68.- Una nia de 2 aos de edad presenta lesiones rojas, con costras melicricas en la cara, cuero cabelludo, rea de paal y extremidades, desde aproximadamente los 2 meses de edad. El evitar sustancias irritantes no ha funcionado en el tratamiento. La nia se rasca constantemente las reas afectadas. Hay una historia familiar positiva para asma y fiebre. Cul es el diagnstico ms probable? a) b) c) d) Dermatitis atpica Celulitis Dermatitis de contacto Liquen cimple crnico

La dermatitis atopica (DA) (Wise-Sulzberger, 1993), llamada neurodermatitis diseminada, por las escuelas Europea, es un estado reaccional de la piel, intensamente pruriginosa, ms frecuente en los nios, multifactorial, en la que combinan factores constitucionales y factores ambientales, por lo tanto de difcil tratamiento, muy frecuente en la consulta diaria del pediatra y del dermatlogo.

DATOS EPIDEMIOLOGICOS La enfermedad originalmente conocida como prrigo de Besnier y eccema constitucional, es ubicua, afecta a todas las razas y existe en todos los pases. Se seala su existencia hasta en el 2% de la poblacin general y en el 14% de los nios. La enfermedad se inicia antes del primer ao de la vida en ms del 60% de los casos, la curva desciende hacia los 12 aos de la cifra de inicio es apenas del 5%- y es excepcional que la enfermedad se inicie en la edad adulta. La enfermedad sufre exacerbaciones en las temporadas de calor o de fro cuando hay sequedad de la atmsfera. CUADRO CLNICO Tradicionalmente se han descrito tres etapas que en la actualidad ya no se presentan pues se suman unas a otras debido a los tratamientos que reciben los pacientes desde su inicio. Etapa de lactante. La enfermedad suele iniciarse en los primeros meses de vida, a veces casi desde el nacimiento. Las lesiones afectan la cara: mejillas, frente (respetando su centro); afectan tambin la piel cabelluda, los pliegues, las nalgas y el dorso de los pies. Las lesiones son de tipo eccematosos (piel llorosa): eritema, vescular y costras melicricas con costras hemticas como signo del rascado. Estas lesiones altamente pruriginosas evolucionan por brotes y en general si el paciente no es yatrognicamente tratado, tienen tendencia a involucionar al ao de edad.

Fase del escolar. Se inicia hacia los 3 a 7 aos, cuando el nio empieza a ir a la escuela. Las lesiones son preferentemente flexurales: cuello, pliegues de codo y huecos poplteos y la morfologa corresponde a la de una dermatitis crnica: Zonas de eritema y liquenificacin (ndice de rascado crnico) y costras hemticas peridicamente sufren un proceso de eccematizacin sobre todo por los tratamientos indebidos. La enfermedad evoluciona por brotes, hay temporadas en que no existen lesiones aun cuando persiste a veces una piel xertica (seca) y pigmentada con intenso prurito. Etapa del adulto. Se vea con poca frecuencia y ahora es habitual en las consultas diarias debido a los malos tratamientos. En este caso adems de las zonas flexurales, se presenta lesiones periorbitarias; y peribucales y lesiones vesiculosas en las manos. Son lesiones tanto liquenificadas como eccematosas, muy pruriginosas, que alternan con periodos asintomticos. Con el uso inmoderado de los corticoesteroides no hay diferenciacin entre las etapas de la enfermedad y se ven casos que arrastran su enfermedad casi desde que nacen hasta la edad adulta.

DIAGNSTICO Es clnico y relativamente sencillo, aunque no toda dermatitis flexural es necesariamente de origen atpico. Deben tomarse en cuenta los antecedentes del propio enfermo y los familiares. En los lactantes debe diferenciarse de la dermatitis seborreica que afecta sobre todo la piel cabelluda y el centro de la cara. En ocasiones hay mezcla de las dos dermatitis: la atpica y la seborreica y es difcil diferenciarlas. La dermatitis del paal predomina en los lactante; en las zonas glteas y genitales, se presenta con eritema y lesiones vesculopapulosas muy pruriginosas. La pitiriasis alba corresponde a los clsicos jiotes de los nios: Manchas hipocrmicas cubiertas de fina escama en la cara. Estos procesos pueden coincidir con manifestaciones de DA, pero nunca se ha demostrado la relacin de causa efecto. TRATAMIENTO Medidas generales. Es conveniente una explicacin amplia al paciente y su familia sealando la naturaleza y evolucin de la enfermedad y lo que se espera del tratamiento que vamos a indicar. Evitar el sol excesivo, el agua clorada de las albercas, el uso de jabn (indicar jabones neutros o sustitutos de jabn), prohibir el uso de pomadas y remedios caseros as como los cosmticos que irritan a la piel.

Las dietas restrictivas han sido y siguen siendo un tema muy controversial, pues mientras los alergistras siguen insistiendo sin muchas bases en el beneficio de estas dietas, los ms hemos comprobado su inutilidad. Existe un grupo muy limitado en que se puede demostrar que una dieta restrictiva de huevo, leche, fresas, etc. puede mejorar los brotes de DA y en tales casos (slo en esos casos) se aconsejara la supresin de tales medicamentos; en lo general se permite al paciente que coma de todo. En la actualidad se estn limitando por los pediatrias ciertos alimentos como el huevo, las fresas, el pltano y la leche de vaca en el primer ao de vida por ser muy alergnicos y se discute la utilidad de la leche materna en estos nios. Es necesario hacer ver que estos pacientes requieren de una atencin ms personal, que sienten la necesidad de cario por lo que es de recomendarse que el nio sea atendido personalemente por la madre. Tratamiento tpico. Depende del estado de la piel; si est eccematosa debe secarse antes de aplicar cualquier pomada que ser rechazada por la piel llorosa. El uso de fomentos con agua de manzanilla o suero fisiolgico es til. Los fometos con agua de vgeto (subacetato de plomo) al 20% son potentes antiexudativos, pero deben limitarse a reas restringidas y no usarse en nios. Estando la piel ya seca, se usan pastas inertes que llevan xido de zinc y calamina en una base de vaselina y lanolina que son protectoras a la vez que antiprutiginosas. Si la piel est muy seca y liquenificada, los fomentos y baos sern emolientes, con almidn y aceites seguidos de cremas o pomadas ms grasosas que llevan vaselina, coldcream y xido de zinc.

En el comercio existen numerosos preparados humectantes que ayudan a mantener el manto cido grasa-agua, que se pierde en la DA. Si hay dermatitis por contacto o imptigo hay que tratar primero estas complicaciones con sulfato de cobre al 1: 1000 y pomadas con vioformo o mupirocn. En el caso de eritrodermia, los baos emolientes y el uso del petrolato (vaselina) ser lo indicado. Cuando hay mucha liquenificacin, pueden usarse cremas con alquitrn de hulla al 3% en base de coldcream por tiempo y zonas limitadas. Los corticoesteroides tpicos son los medicamentos ms usados en esta enfermedad y muchas veces causa de las complicaciones que se presentan. Estos medicamentos no curan nada, solo engaan al paciente y al mdico hacindoles creer que la enfermedad va curando cuando slo se oculta y modifica. Al pasar el efecto de estos medicamentos, invariablemente se presenta el rebote y ms tarde la corticodependencia, haciendo a la enfermedad incontrolable. Nunca deben usarse los corticoesteroides fluorinados en nios y en la cara y zonas genitales o en los pliegues por su posibilidad de absorcin. La hidrocortisona es de baja potencia, hace menos dao, pero tambin es menos efectiva; algunos la recomiendan. Tratamiento sistmico. El uso de antihistamnicos sobre todo de la primera generacin que son sedantes como la clorfeniramina y la hidroxicina ayudan a mejorar el prurito; el ketotifeno y la oxotamida por su accin dual: inhiben la produccin de histamina por los mastocitos y bloquean los receptores Hl, tambin son de ayuda, al igual que los sedantes suaves tanto para el paciente como para la madre que est en perenne angustia que

transmite al pequeo paciente. La talidomida ha mostrados ser de ayuda en casos de DA conticoestropeada, en su fase eritrodrmica, no tanto en los nios, a dosis de 100 mg al da. Los antibiticos tipo dicloxacilina, sern necesarios cuando haya infeccin o simplemente eccematizacin por el papel que tiene el estafilococo dorado.

1. Criterios Diagnstico para Dermatitis Atpica (DA), MedicalCriteria.com. Dr. Luciano Domnguez-Soto, Dr. Amado Sal Cano, Dermatitis atpica neurodermatitis diseminada, Dermatologa, Parte C, Libro ) "

69.- Cuando los resultados en la manometra esofgica muestran un aumento de la presin basal del esfnter esofgico inferior (EEI) junto a una disminucin o ausencia de su relajacin con la deglucin, estamos frente a un cuadro de: a) Esclerodermia con afectacin esofgica. b) Enfermedad por reflujo gastroesofgico c) Espasmo esofgico difuso. d) Acalasia.

Diagnstico de acalasia Aspecto radiogrfico Un esofagograma puede mostrar la disminucin de peristalsis, la dilatacin del esfago proximal y el estrechamiento del esfago en su parte inferior. El paciente se traga una solucin de bario, con fluoroscopia continua que son grabaciones de rayos X para observar el flujo del fluido a lo largo del esfago, sin que se observe el movimiento peristltico normal del esfago. Hay un agudo estrechamiento en el esfnter esofgico inferior y reduccin del dimetro en la unin gastro-esofgica. La imagen que proyecta se denomina clsicamente en pico de loro o en cola de ratn. Por encima de la reduccin, el esfago a menudo se observa con una dilatacin de diversos grados a medida que poco a poco se va estirando en el tiempo. Por la falta de movimientos peristlticos, se suele observar en la radiografa un margen entre aire y lquido. Manometra esofgica Debido a su sensibilidad, el diagnstico es confirmado por medio de una manometra esofgica, que mide las presiones del esfago mediante una sonda nasoesofgica y permite comparar las presiones en situacin basal y durante la deglucin.4 Se inserta un tubo delgado a travs de la nariz, y se le instruye al paciente a deglutir varias veces. La sonda mide las contracciones musculares en diferentes partes del esfago durante el acto de la

deglucin. La manometra revela la falla del EEI para relajarse con cada deglucin y la falta de peristaltismo funcional del msculo liso en el esfago. Para descartar complicaciones se suele acudir a una endoscopia digestiva alta.

Esquema manomtrico de acalasia demostrando contracciones aperistlticas, un aumento de la presin intraesofgica y el fallo de la relajacin del esfnter esofgico inferior. Debido a la similitud en los sntomas, la acalasia se puede confundir con trastornos ms comunes, tales como la enfermedad de reflujo gastroesofgico, la hernia de hiato, e incluso trastornos psicosomticos.

REFERENCIAS BIBLIOGRFICAS: 1. Garca Gutirrez A. Acalasia de esfago. Disponible en: http://www.sld.cu/galerias/pdf/uvs/cirured/acalasia..pdf Consultado Mayo 27, 2006. 2. Fareras Rozman, et al. Acalasia esofgica.Tratado de Medicina Interna. 15 Edicin. Espaa: Ediciones Harcourt; 2003. Seccin 2. Cap 20.p.354-9.

70. - The following conditon is commonly seen in the magnesium-amonium-phosphate (struvite) stones: a) b) c) d) Recurrent P. mirabilis infection Double J stent placement Resorptive hypercalciuria Renal azotemia

Esta bacteria de colonias redondeadas tiene la habilidad de producir grandes niveles de ureasa. La ureasa hidroliza urea a amonaco, (NH3) y eso hace a la orina ms alcalina. Y al subir la alcalinidad puede liderar la formacin de cristales de estruvita, carbonato de calcio, y/o apatita. Esta bacteria puede encontrarse en clculos, y esas bacterias escondidas all, pueden reiniciar una infeccin post tratamientos antibiticos. Al desarrollarse los clculos, despus de un tiempo pueden seguir creciendo ms y causar obstruccin dando fallas renales.

Proteus tambin puede producir infecciones de heridas, septicemia y neumonas, sobre


todo en pacientes hospitalizados.

Referencias Bibliogrficas:

Esipov, Sergei E. and J. A. Shapiro (1998). Kinetic model of Proteus mirabilis swarm colony development. Journal of Mathematical Biology 36 (3). doi 10.1007/s002850050100. Frnod, Emmanuel (2006). Existence result for a model of Proteus mirabilis Differential and integral equations 19 (6): pp. 697-720. swarm. http://arxiv.org/abs/math.FA/0702761.

71.- Un sujeto de 30 aos de edad adicto a las drogas intravenosas presenta debilidad del hemicuerpo derecho y cefalea en un periodo de dos das. La exploracin revela un individuo mal nutrido y afebril con hemiparesia derecha leve. Cul de los siguientes es el diagnstico ms probable? a) b) c) d) Endocarditis bacteriana Meningitis por virus de la inmunodeficiencia humana (VIH) Absceso cerebral. Meningitis criptoccica.

Los adictos a drogas intravenosas estn propensos a sufrir bacteriemia, que a su vez puede producir absceso cerebral y disfuncin neurolgica progresiva. Los pacientes con aqul, por lo comn se encuentran afebriles a no ser que haya endocarditis acompaante u otro origen endovascular de infeccin. Los adictos a drogas intravenosas tienden a presentar endocarditis bacteriana y pueden cursar con dficit neurolgicos en forma apopltica debido a embolia sptica del cerebro. Sin embargo, por lo regular tienen fiebre. La meningitis por VIH produce cefalea y datos de irritacin menngea, pero no se presenta dficit neurolgico focal. La meningitis criptoccica se manifiesta con conducta alterada y cefalea y los pacientes estn afebriles. Sin embargo, es raro que haya datos de seudoapopleja. Por ltimo, el uso de drogas intravenosas puede provocar embolia de cuerpo extrao pero con problemas neurolgicos apoplticos. Un mbolo puede llegar al cerebro por un cortocircuito cardaco de derecha a izquierda o bien por una malformacin arteriovenosa pulmonar si la inyeccin es venosa. El mbolo puede entrar a la circulacin cerebral de manera directa en caso de inyeccin intracarotdea.

Referencias Bibliogrficas: Nath A. Brain abscess and parameningeal infections. In: Goldman L, Ausiello D, eds. Cecil Medicine. 23rd ed. Philadelphia, Pa: Saunders Elsevier; 2007: chap 438 Allen R. M. MMS Medicina Interna. 5. Edicin. National Medical Series. Mc. Graw Hill. 2006. (Captulo 11 XVI B 1-2).

72.- Femenino de 62 aos con diagnstico de miastenia gravis. Cul de los siguientes medicamentos est dirigido al manejo de esta patologa? a) b) c) d) Neostigmina Quinidina Sumatriptn Succinilcolina

Manejo farmacolgico: Existen diferentes pautas teraputicas dirigidas a contrarrestar


los sntomas de la enfermedad o el mecanismo inmunolgico. Los frmacos utilizados son:

- Inhibidores de la Acetilcolinesterasa (Neostigmina, Piridostigmina). Dirigidos al manejo sintomtico de la MG, mejorando la fuerza motora pero no la progresin de la enfermedad. Su mecanismo de accin es la inhibicin reversible de la acetilcolinesterasa, lo cual genera un aumento de ACh en la placa motora. La dosis a utilizar es variable y debe modificarse en distintas etapas de la enfermedad, incluso siendo frecuente no lograr un efecto uniforme en los diferentes grupos musculares en un mismo paciente. El objetivo por lo tanto ser utilizar la dosis mnima con la que se genere la mejor respuesta clnica. El efecto se obtiene de 30 minutos a 2 horas de la administracin y tiene una duracin de hasta 6 horas. Las dosis recomendadas de Piridostigmina son de 15-60 mg cada 4-6 horas va oral y de Neostigmina 0,5-2 mg/kg cada 4-6 horas intramuscular. Las reacciones adversas asociadas son: dolor abdominal, hipersalivacin, aumento de las secreciones respiratorias y bradicardia y se relacionan con el efecto colinrgico generado, por lo que es necesario administrar concomitantemente atropina2,3.

Referencia bibliogrfica:
1995.

1.-Ponsetia JM: Miastenia Gravis. Manual Teraputico. Barcelona; Springer Verlag Ibrica,

2.- Ponsetia JM, Espina E, Armengola M: Diagnstico y Tratamiento de la Miastenia grave. Med Clin (Barc) 2000; 115: 264-70.

3.- Drachman DB: Myasthenia gravis. N Engl J Med 1994; 330: 1797-810. 4.- Andrews PI: Autoimmune myasthenia gravis in childhood. Semin Neurol 2004; 24: 10110 5. - Anlar B: Juvenile myasthenia: diagnosis and treatment. Paediatr Drugs 2000; 2: 161-9.

6. - Gajdos P: Myasthenic syndrome. Diagnosis trends. Rev. Prat 2000; 50: 419-23 7.

73.- Varn de 28 aos de edad con AV en los dos ojos de 20/30. Al explorarlo el paciente entrecierra los ojos. De cerca ve bien, pero se cansa al estar leyendo despus de un tiempo. El resto de la exploracin es normal. Cul es el diagnstico ms probable? a) b) c) d) Miopa Astigmatismo Hipermetropa Miopa y presbiopa

AMETROPAS Cuando los rayos de luz paralelos que inciden en un globo ocular enfocan a nivel de la retina, dicha condicin se conoce como emetropa. Si, por el contrario, estos rayos de luz no enfocan a nivel de la retina, se habla entonces de ametropa. En esta ltima el foco principal se encuentra por delante (MIOPA) o por detras de la retina (HIPERMETROPA), tanto ms retirado de ella cuanto mayor sea la ametropa, o bien pierde su caracterstica puntiforme para formar dos lneas focales principales perpendiculares entre s (ASTIGMATISMO). El astigmatismo es un defecto refractivo que hace que todo se vea deformado o desenfocado, tanto en visin cercana como en visin lejana. En el ojo astgmata, la crnea tiene forma de elipse, esto hace que las imgenes no enfoquen en un foco nico como en el ojo normal.

El astigmatismo puede combinarse con miopa, dando lugar a un astigmatismo mipico donde adems de ver desenfocado de lejos, las imgenes tanto cercanas como lejanas se perciben deformes. El astigmatismo puede combinarse con hipermetropa, dando lugar a un astigmatismo hipermetrpico donde adems de ver desenfocado de cerca, las imgenes tanto cercanas como lejanas se perciben deformes. El sntoma ms importante del astigmatismo es la percepcin de imgenes distorsionadas. As mismo son habituales los dolores de cabeza frontales y en algunas ocasiones inclinaciones laterales de la cabeza compensatorias. El astigmatismo distorsiona o borra la visin a una cierta distancia, tanto de lejos como de cerca. La visin es similar al efecto de los espejos deformados, los cuales reproducen objetos demasiado altos, demasiado anchos o demasiado delgados.

PAC Oftalmologia Parte B Libro 4 Errores Refractivos y Ciruga Refractiva GENERALIDADES PTICAS.

74.- Un hombre de 55 aos presenta dolor precordial que le apareci cuando estaban en reposo; se irradio al cuello y al hombro izquierdo, refiere que tuvo 20 minutos de duracin y cedi , tiene antecedentes de obesidad, Diabetes Mellitus, hipertensin y sedentarismo, el diagnostico mas probable es: a) b) c) d) Angina inestable. Infarto agudo al miocardio. Angina estable. Pericarditis.

Entendemos por angina de pecho, o angor, un dolor torcico, generalmente retrosternal, que puede ser descrito como opresivo, quemazn o simplemente una leve pesadez, y que est motivado por la isquemia del miocardio. Este dolor o molestia puede irradiar o presentarse nicamente en el cuello, mandbula, hombros, brazos, antebrazos, manos, espalda o epigastrio. Rara vez se presenta por encima de la mandbula o debajo del epigastrio. La isquemia miocrdica se presenta en ocasiones con sntomas distintos de la angina como disnea, debilidad, fatiga o eructos. Estos sntomas son equivalentes anginosos y suelen ser mas frecuentes en personas ancianas. La situacin en que se presenta la angina inestable no parece relacionada con un mayor trabajo cardaco. Es decir, la isquemia miocrdica no parece justificarse por un mayor consumo miocrdico de oxgeno y, por lo tanto, la causa es una disminucin aguda del flujo sanguneo coronario. Clsicamente se denomina angina inestable a la que se presenta en las siguientes circunstancias:

1. Angina de reposo: Ocurre en reposo o con un mnimo esfuerzo. 2. Angina de comienzo reciente: Inicio de los sntomas en el ltimo mes, en un paciente previamente asintomtico, y de aparicin con esfuerzos mnimos. 3. Angina progresiva: En un paciente con angina estable previa, los sntomas se presentan con esfuerzos menores, son cada vez mas frecuentes o su duracin es ms prolongada. La presentacin clnica de la angina inestable puede ser idntica a la de un infarto agudo de miocardio (IAM), la diferencia es conceptual: Si hay necrosis miocrdica hablamos de infarto agudo de miocardio. Los datos que podemos obtener de la historia clnica, la exploracin fsica y el electrocardiograma, con frecuencia no permiten diferenciar entre angina inestable e infarto agudo de miocardio. La elevacin de marcadores de dao miocrdico como la creatnfosfoquinasa (CPK) o la troponina T o I, identifican una necrosis miocrdica y, por lo tanto un IAM. Es por este motivo que actualmente tanto la angina inestable como el IAM se agrupan bajo el trmino de sndrome coronario agudo. Segn la presentacin electrocardiogrfica, el sndrome coronario agudo (SCA) se divide en SCA con elevacin del segmento ST (frecuentemente evoluciona a un infarto con onda Q) y SCA sin elevacin del segmento ST, que incluye a la angina inestable y la mayor parte de los casos de IAM sin onda Q. Esta terminologa es la que actualmente se utiliza porque tiene la ventaja de clasificar el cuadro clnico del paciente a partir de datos clnicos y electrocardiogrficos que pueden obtenerse de modo rpido y sencillo.

LECTURA RECOMENDADA
Guas clnicas para el manejo de la angina inestable e infartosin elevacin del ST. Estratificacin del riesgo. Bibliografa Internacional: R. Marrn Tundidor*, P. Palazn Saura*, L. M. Claraco Vega*, C. Ascaso Martorell*, J. Povar Marco*, J. M. Franco Sorolla*, I. Calvo Cebollero** *SERVICIO DE URGENCIAS Y **SERVICIO DE CARDIOLOGA-UNIDAD DE HEMODINMICA Y CARDIOLOGA INTERVENCIONISTA.

75.- Se trata de masculino de 55 aos alcohlico, que ingresa a la sala de urgencias por sospecha de sndrome de Boerhaave o rotura espontnea del esfago. La triada clnica clsica de sta patologa consiste en? a) Nuseas, vmitos y fiebre b) Vmitos, dolor torcico y enfisema subcutneo c) Fiebre, dolor abdominal y hematemesis d) Dolor torcico, dolor abdominal y vmito

Definicin: Ruptura espontnea del esfago que ocurre sobre todo como consecuencia de
un violento vmito Este sndrome fue descrito por primera vez en 1724 por Herman Boerhaave. Se diferencia del sndrome de Mallory-Weiss en que en este caso se tratada de una perforacin transmural mientas que el sndrome de Mallory-Weiss es tan solo una laceracin. Dado que frecuentemente est asociado al vmito, no puede decirse que en realidad se trate de una ruptura espontnea. Hay que distinguirlo de la perforacin yatrognica que supone el 85-90% de los casos de rupturas esofgicas. Casi siempre en el lado izquierdo del tercio inferior esofgico a 2 o 3 cm de la unin gastroesofgica. A igual que el sndrome de Mallory-Weiss, se ha relacionado con los aumentos bruscos de la presin intrabdominal y con la hernia de hiato. Es ms frecuentemente observado en pacientes alcohlicos o que abusan de la comida. Se trata de un sndrome relativamente raro pero con un alto ndice de mortalidad (35%). De hecho, es considerada como la ms letal de todas las perforaciones del tracto digestivo. Los sntomas son vmitos, dolor torcico y enfisema subcutneo cervical.

REFERENCIAS BIBLIOGRFICAS: C. Yeo. Shackelford' surgery of the Alimentary Tract, 2007. Saunders, 2700 pginas. Khan, Aamir Z.; Forshaw, Mathew J.; Davies, Andrew R.; Youngstein, Taryn; Mason, Robert C.; Botha, Abraham J. Transabdominal Approach for Management of Boerhaave's Syndrome . American Surgeon , May2007, Vol. 73 Issue 5, p511-513. Hill, Andrew G.; Tiu, Albert T.; Martin, Iain G. Boerhaave's syndrome : 10 years experience and review of the literature: H. ANZ Journal of Surgery , Dec2003, Vol. 73 Issue 12, p1008-1010. Ochiai, T.; Hiranuma, S.; Takiguchi, N.; Ito, K.; Maruyama, M.; Nagahama, T.; Kawano, T.; Nagai, K.; Nishikage, T.; Noguchi, N.; Takamatsu, S.; Kawamura, T.; Teramoto, K.; Iwai, T.; Arii, S. Treatment strategy for Boerhaave's syndrome. Diseases of the Esophagus , May2004, Vol. 17 Issue 1, p98-103.

76.- El signo clnico que indica irritacin del nervio citico como componente de una lumbalgia se denomina: a) Signo de Lassegue b) Signo de Galeazzi c) Signo de McMurray d) Signo de Filkestein

La lumbalgia es una patologa muy frecuente en nuestro medio, con una enorme repercusin sanitaria y sociolaboral. Con la edad, se inicia una cascada degenerativa en la columna lumbar, que comienza en el disco intervertebral, continuando por las facetas articulares y dems elementos vertebrales. Esta degeneracin del raquis forma parte del envejecimiento normal del individuo, aunque en ocasiones puede causar dolor y/o alteraciones neurolgicas. Para comprender la fisiopatologa del dolor lumbar, ser preciso conocer que la inervacin de la columna lumbar se hace fundamentalmente por tres ramos nerviosos: ramo dorsal de los nervios espinales lumbares (o rami dorsal), nervio sinuvertebral de Luschka y ramos ventrales de la cadena simptica. Existen dos tipos de patrones de dolor en la columna lumbar: El dolor irradiado y el dolor referido. La cascada de la degeneracin consta en tres estadios. El primer estadio sera la disfuncin. El anillo fibroso se fisura y pierde la capacidad de contener al ncleo pulposo. Esto ocasiona primero el sndrome de disrupcin discal y, si el ncleo supera el contorno del annulus, las hernias discales. El segundo estadio de Kirkaldy-Willis es el de inestabilidad. En este estadio la movilidad en el segmento mvil aumenta de forma patolgica.

Maniobra de Lassgue: Es la ms importante, es casi patognomnica. Con el paciente en decbito supino se levanta la pierna extendida. Se considera positiva cuando aparece un dolor intenso a nivel lumbar, en la pierna o en ambos niveles, entre los 30-75 de flexin en la cadera, debido al estiramiento del nervio citico. Pasados los 70 puede aparecer un falso positivo (dolor tambin en sujetos sanos) por la distensin de los isquiotibales. Para descartarlo realizaremos otras maniobras diagnsticas.

Referencias Bibliogrficas:
1. Andersson GBJ. Epidemiologic features of chronic low-back pain. Lancet. 1999;354:5815. 2. Estudio EPISER. Sociedad Espaola de Reumatologa. 2000. 3. Herrera Rodrguez A, Rodrguez Vela J. Estenosis de canal lumbar. Rev Ortop Traumatol.2002;4:351-72. 4. MacGregor AJ, Andrew T, Sambrook PN, Spector TD. Structural, psychological, and genetic influences on low back and neck pain: a study of adult female twins. Arthritis Rheum.2004;51:160-7. 5. Kim KS, Yoon ST, Park JS, Li J, Park MS, Hutton WC. Inhibition of proteoglycan and type II collagen synthesis of disc nucleus cells by nicotine. J Neurosurg Spine. 2003;99:291-7. 6. Fujiwara A, Tamai K, Yamato M, An HS, Yoshida H, Saotome K, et al. The relationship between facet joint osteoarthritis and disc degeneration of the lumbar spine: an MRI study. Eur Spine J. 1999;8:396-401.

7. Eyre DR, Muir H. Types I and II collagens in intervertebral disc. Interchanging radialdistributions in annulus fibrosus. Biochem J. 1976;157:267-70.56.-5.

77.- Un lactante menor tiene el antecedente de traumatismos leves por cadas de su propia altura, lo que le ha ocasionado la formacin de hematomas en las rodillas, los tobillos y los codos. Se solicitaron estudios de laboratorio que han revelado tiempo parcial de tromboplastina prolongado y nmero normal de plaquetas. El diagnstico ms probable es:

a) b) c) d)

Hipoprotrombinemia. Afibrinogenemia. Disfibrinogenemia. Hemofilia.

La hemofilia es una enfermedad gentica ligada al cromosoma X que se expresa por una disminucin de Factor de coagulacin VIII o IX, segn corresponda a Hemofilia A o B respectivamente. Sospecha diagnstica: La enfermedad debe sospecharse en los varones con antecedentes familiares por lnea materna de hemofilia. La edad de inicio de la sintomatologa esta relacionada con el nivel de factor presente. Hemofilias graves: Pueden presentar sntomas desde el periodo de recin nacido manifestndose como un simple hematoma en la regin de vacuna BCG hasta una severa hemorragia intracraneana. En periodo de lactante aparecen equimosis en las zonas de presin y al iniciar la marcha comienzan la aparicin de hematomas de magnitud exagerada para traumatismos leves y las hemartrosis. Hemofilias moderadas: Puede iniciarse a temprana edad, incluso con presencia de hemartrosis y hematomas, pero en general las manifestaciones son evidentes frente a cirugas y traumas. Hemofilias leves: La clnica es muy escasa, puede haber antecedentes de epistaxis, equimosis fciles o simplemente asintomtico. Generalmente la sospecha diagnstica es por exmenes preoperatorios de rutina, o bien cuando han sufrido traumas de cuanta o en relacin a ciruga. Escenarios de sospecha Paciente de cualquier edad con hemorragias espontneas o desproporcionadas al evento traumtico.

Inicialmente el manejo de estos pacientes es realizado por el mdico no especialista. Exmenes recomendados: TTPA, TP, Hemograma con recuento de plaquetas. Si el TTPA y/o el TP es prolongado con hemograma normal en paciente sintomtico, se plantea la sospecha de un dficit de coagulacin congnito. La decisin de hospitalizar al enfermo depender de la gravedad del cuadro. En general, las hemartrosis y hematomas de baja cuanta no requieren hospitalizacin. GRADO DE RECOMENDACIN C: Se recomienda el uso de antifibrinolticos en sangrados de mucosa oral, nasal y digestiva. GRADO DE RECOMENDACIN B: El uso de plasma est reservado para aquellas situaciones de sangrado grave en un paciente con alta sospecha de hemofilia an no clasificada. GRADO DE Ministerio de Salud Subsecretara de Salud Pblica 13.

RECOMENDACIN C. Se sugiere, antes de iniciar tratamiento, tomar una muestra de sangre en tubo citrato centrifugar y congelar el plasma a -70 C para el estudio inicial del caso. Intervenciones no recomendadas: - Realizar tiempo de sangra. - Estudios de imgenes antes de tratar al paciente. - Trasladar pacientes sin haber sido estabilizados previamente. - Uso de antifibrinolticos en hematuria. GRADO DE RECOMENDACIN C En hemorragias de riesgo vital (ver ms adelante), el traslado a un centro de atencin de hemofilia debe ser realizado una vez estabilizado el paciente con la administracin de plasma. REFERENCIAS: 1 Stobart K, Iorio A, Wu JK. Clotting factor concentrates given to prevent bleeding and bleeding-related complications in people with hemophilia A or B. Cochrane Database Syst Rev. 2006 Apr 19;(2):CD003429. 2 Manco-Johnson MJ, Abshire TC, Brown D, Buchanan R, Cohen AR, Di Michele D, Hoots WK, Leissinger CA, McRedmond K, Nugent D, Shapiro AD, Thomas GA, Valentino LA, Riske B. Initial results of a randomized prospective trial of prophylaxis to prevent joint disease in young children with factor VIII (FVIII) deficiency. Blood 2005; 106: 6a (abstract). 3 Panicker J, Warrier I, Thomas R, Lusher JM. The overall effectiveness of prophylaxis in severe haemophilia. Haemophilia. 2003 May;9(3):272-8. 4 Fischer K, Van Den Berg M. Prophylaxis for severe haemophilia: clinical and economical issues. Haemophilia. 2003 Jul;9(4):376-81. 5 Royal S, Schramm W, Berntorp E, Giangrande P, Gringeri A, Ludlam C, Kroner B, Szucs T; for the European haemophilia economics study group. Quality-of-life differences between prophylactic and on-demand factor replacement therapy in European haemophilia patients. Haemophilia. 2002 Jan;8(1):44-50. 6 Steen Carlsson K, Hjgrd S, Glomstein A, Lethagen S, Schulman S, Tengborn L, Lindgren A, Berntorp E, Lindgren B. On-demand vs. prophylactic treatment for severe haemophilia in Norway and Sweden: differences in treatment characteristics and outcome. Haemophilia. 2003 Sep;9 (5):555-66. 7 Fischer K, van der Bom JG, Molho P, Negrier C, Mauser-Bunschoten EP, Roosendaal G, De Kleijn P, Grobbee DE, van den Berg HM. Prophylactic versus on-demand treatment strategies for severe haemophilia: a comparison of costs and long-term outcome. Haemophilia. 2002 Nov;8(6):745-52.

78.- Se trata de paciente masculino de 33 aos el cual presenta los siguientes datos clnicos, lgia facial, rinorrea posterior , obstruccin nasal, stos son criterios mayores para el diagnstico de:

a) b) c) d)

Sinusitis aguda Faringoamigdalitis estreptococica Rinitis vasomotora Rinitis estacional

CRITERIOS CLNICOS El diagnostico de la sinusitis aguda es clnica, depende en la presencia de por lo menos dos sntomas mayores, o un sntoma mayor y dos menores. Sntomas mayores: Dolor o presin facial. Obstruccin nasal. Rinorrea purulenta. Hiposmia o anosmia.

Sntomas menores: Cefalea. Halitosis. Dolor dental superior. Tos, especialmente en nios. Otalgia o presin en odos.

La combinacin de tres de cada cuatro criterios ha dado una especificidad del 81% y una sensibilidad del 66%. En estudios realizados por Williams y cols. encontraron en hombres adultos, que la congestin nasal, tos, estornudos tienen sensibilidad del 70% al 72%. El sntoma ms especfico (93%) es el dolor en senos maxilares. En un estudio en Europa, el indicador ms sensible fue el dolor en los dientes (83%). El estndar de oro para el diagnstico de la sinusitis maxilar es el hallazgo de material purulento a travs de la aspiracin del seno maxilar. Williams Jr JW, Aguilar C, Cornell J, Chiquette E. Dolor RJ, Makela M, Holleman DR, Simel DL. Antibiotics for acute maxillary sinusitis. Cochrane Database of Systematic Reviews 2003, Issue.

79.- Una mujer de 28 Aos llega al servicio de urgencias con disnea y dolor torcico pleurtico. Tambin refiere que en los cuatro das previos tuvo sntomas muy marcados de tumefaccin e hipersensibilidad en la pantorrilla y el muslo derechos. Con base en la presentacin clnica se sospecha una trombosis venosa profunda que podra haber causado embolia pulmonar. Cul de los siguientes fragmentos de informacin de los antecedentes de la enferma apoyan mejor este diagnstico? a) b) c) d) Antecedente de tabaquismo Antecedente de diabetes M. en la familia de la paciente Antecedente de lesin en la extremidad inferior Antecedente de hipertensin

La lesin de las extremidades inferiores puede producir formacin de cogulos sanguneos y desarrollo de tromboflebitis. Otros factores que contribuyen al desarrollo de trombosis venosa profunda incluyen uso de compuestos con estrgenos (p. ej., anticonceptivos orales) o inmovilizacin de la extremidad inferior (p. ej., durante ciruga o reposo en cama prolongado) que da lugar a estasis venosa. La hipertensin, la diabetes cademia y el abuso de drogas intravenosas no tienen relacin con la trombosis venosa profunda. La disnea y el dolor pleurtico sugieren que la trombosis venosa profunda ha causado embolia pulmonar, que es provocada por desplazamiento de un trombo de las venas de las extremidades inferiores o la pelvis a la arteria pulmonar.

Factores predisponentes para trombosis venosa profunda


A mediados del siglo 19 Virchow enunci los tres factores fundamentales en la patognesis de la trombosis intravascular: dao de la pared vascular, estasis del flujo sanguneo y cambios en la coagulabilidad de la sangre. Hoy, ya prximos al siglo 21 dicha definicin contina vigente. Factores predisponentes generales. En estudios epidemiolgicos, la edad sobre 50 aos, la obesidad, la hipertensin arterial y el tabaquismo son factores que se asocian a riesgo de flebotrombosis. Dao de la Pared Vascular. El trauma directo del endotelio vascular puede ocurrir durante algunos procedimientos diagnsticos y teraputicos efectuados por va venosa femoral (por ejemplo: cateterismo venoso para dilisis, estudios cardiolgicos, etc.). El cateterismo venoso prolongado para quimioterapia, hiperalimentacin parenteral o monitorizacin, puede dar origen a flebotrombosis, a pesar del uso de catteres de material no trombognico. El trauma indirecto de la pared venosa puede ocurrir en contusiones y fracturas. Estudios recientes han demostrado flebotrombosis entre el 50 y 70% de los pacientes con trauma mayor de tronco o extremidades inferiores, siendo la lesin venosa endotelial un elemento predisponente inicial.

La trombosis tambin puede ser inducida por activacin de las clulas del endotelio por citoquinas provenientes de procesos traumticos o inflamatorios a distancia. Los procedimientos quirrgicos, especialmente los ortopdicos (por ejemplo artroplastas de cadera o rodilla), u operaciones de la cavidad pelviana (por ejemplo ginecolgicas y urolgicas) se asocian a un elevado riesgo de flebotrombosis, el cual es atribuido al dao sufrido por estructuras venosas vecinas. El dao endotelial, secuela de una flebotrombosis previa, tambin es un factor predisponente para una nueva flebotrombosis. Estasis sangunea. La disminucin de la velocidad del flujo venoso es un factor que favorece la flebotrombosis. El enlentecimiento es normal en el reposo e inmovilidad muscular, por lo que la sola permanencia prolongada en cama puede ser una causa predisponente. El enlentecimiento del retorno venoso tambin puede tener su origen en un trastorno central, con disminucin del gasto cardaco, tal como ocurre en la insuficiencia cardaca. La inmovilidad "forzada", con ausencia de la funcin de bomba muscular, propia de viajes prolongados sin posibilidad de deambular, la inmovilizacin de una extremidad por esguince o fractura, la inmovilidad de ambas extremidades por lesin neurolgica enceflica o espinal, la inmovilidad por administracin de anestesia general o regional, son todas condiciones que comparten el mismo factor comn predisponente: la menor velocidad del flujo de retorno venoso. El riesgo de flebotrombosis por enlentecimiento del flujo sanguneo ocurre si hay compresin venosa extrnseca, situacin frecuente en el embarazo y menos frecuente en tumores o masas pelvianas o retroperitoneales. La dilatacin venosa, sea del territorio superficial (vrices) o profundo (secuelas postrombticas, insuficiencia venosa y aneurismas venosos) tambin es un elemento asociado a trombosis por estasis venosa. Trastornos de la coagulacin o trombofilias. La sangre se mantiene en estado lquido por el equilibrio entre factores procoagulantes y factores anticoagulantes. Dicho equilibrio puede ser alterado en forma transitoria por condiciones como el embarazo o durante el uso de anticonceptivos orales, favoreciendo la coagulacin espontnea. El desarrollo de algunos tumores, en especial en el cncer de pncreas, ovario, prstata, pulmn, mama, as como en el linfoma no Hodking, predispone a la enfermedad tromboemblica venosa por mecanismos de hipercoagulabilidad poco conocidos, probablemente relacionados a protenas anormales de origen tumoral. La mayor viscosidad sangunea por aumento del hematocrito, como ocurre en individuos que viven en altura o en las policitemias, tambin predispone a la flebotrombosis.

El dficit de protenas anticoagulantes naturales (por ejemplo antitrombina III, protenas C y S), la mutacin en algunos factores (ejemplo: resistencia a protena C activada) o la aparicin o acumulacin anormal de algunas sustancias circulantes como anticuerpos antifosfolpidos u homocistena, son condiciones reconocidas de mayor riesgo de flebotrombosis (ver artculo "Hipercoagulabilidad: estudio y tratamiento" en esta monografa).

Reerencias Bibliogrficas:
1. Hirsh J., Hoak J. Management of deep vein thrombosis and pulmonary embolism. A statement for healthcare professionals. American Heart Association, Circulation 1996; 93: 2212-45. 2. Schulman S, Rhedin A.S, Lindmarker P. et al. A comparison of six week with six month of oral anticoagulant therapy after a first episode of venous thromboembolism. N Engl J Med 1995; 332: 1661-5. 3. The Columbus Investigators. Low molecular weight heparin in the treatment of patients with venous thromboembolism. N Engl J Med 1997; 337: 657-2. 4. Koopman M.M.W., Prandoni P., Piovella F et al. Treatment of venous thrombosis with intravenous unfractioned heparin administered in the hospital as compared with subcutaneous low molecular weight heparin administered at home. N Engl J Med 1996; 334: 682-7.

80.- Masculino de 32 aos que inici con disfona y disfagia, es referido al servicio de endocrinologa donde es confirmado el diagnostico de ndulo tiroideo nico, El estudio recomendado es? a) Biopsia por aspiracin con aguja fina. b) Gammagrama tiroideo. c) Tomografa computada de cuello. d) Repetir Pruebas de funcin tiroidea.

El ndulo tiroideo solitario se define como el crecimiento localizado de la glndula tiroides, usualmente es benigno, la prevalencia es del 4 al 7% en la poblacin general. Aunque el cncer tiroideo es el tumor endocrino ms comn slo representa el 1% de todos los cnceres y 5% de todos los ndulos tiroideos. El estudio clnico diagnstico y teraputico debe iniciarse con historia clnica completa, exploracin fsica y exmenes de laboratorio que incluyan un perfil tiroideo para evaluar funcin de la glndula. La citologa tiroidea por aspiracin (CTA) es el principal procedimiento diagnstico en los pacientes con ndulo tiroideo solitario, por ser capaz de diferenciar lesiones benignas de las malignas, sus principales ventajas son: segura, reduce costos de atencin mdica, selecciona mejor los pacientes que sern sometidos a tratamiento quirrgico y se realiza en pacientes ambulatorios.

1. Torres AP, Hernndez SE, Caracas PN, Serrano GI et al Diagnstico y tratamiento del ndulo tiroideo. Rev Edocrinol Nutr 2000; 8 (3): 87-93.

81.- Un lactante varn de 4 semanas de vida acude a consulta inicialmente con una historia de 2 semanas de vmitos crecientes y escasa ganancia ponderal. Los vmitos, a menudo proyectados, han persistido pese a que se ha cambiado varias veces de frmula. En las ltimas 24 horas tiene vmitos en posos de caf. Las heces han sido firmes y el nio defeca un da si y otro no. La exploracin fsica revela un lactante delgado e irritable con abdomen plano. En el hipocondrio derecho se nota una masa firme en forma de aceituna. Cul de los siguientes estudios de laboratorio debe acometerse primero? a) b) c) d) Radiografa de abdomen. Ecografa abdominal. Valoracin de electrlitos en suero. Recuento sanguneo completo.

La historia y la exploracin fsica son tpicas de un diagnostico de estenosis hipertrfica del piloro. En este caso, la consideracin mas importante es el estado hidroelectroltico del paciente, ya que en este trastorno pueden observarse anomalas graves. Es ms comn una alcalosis metablica hipoclormica, y la terapia inicial anterior a la correccin quirrgica, debe incluir normalizacin de los electrolitos del suero. En muchos lactantes con estenosis Pilarica se observa hematemesis, normalmente como resultado de una gastritis superficial. Sin embargo, rara vez es de consecuencia hemodinmica. La confirmacin del diagnstico puede realizarse fcilmente en la mayora de los casos mediante ecografa de abdomen, que revela un canal pilrico alargado y engrosado. La radiografa de abdomen puede mostrar un estomago distendido y lleno de aire. La radiografa con contraste revela un canal pilrico alargado con retraso del vaciamiento gstrico. Un recuento sanguneo completo no tiene ningn valor en el diagnstico de la estenosis Pilrica.

LECTURA RECOMENDADA: Acta Pediatr Mex 2010;31(2):50-54 INP. Estenosis hipertrfica del ploro. Estudio clnico-epidemiolgico. Dr. Carlos Baeza-Herrera,* Dra. Aln Villalobos-Castillejos,** Dr. Arturo Arcos-Aponte,*** Dr. Javier Lpez- Castellanos,**** Dr. Luis Manuel Garca-Cabello*****

82.- Masculino recin nacido que a la exploracin fsica presenta peso 3.900kg, talla 55cm, ausencia de lanugo y vrmix gaseosa, la sospecha diagnstica en ste paciente es: a) Prematuro b) Trmino c) Postmaduro d) Hipotrfico

Definicin:
Se define como recin nacido postrmino a cualquier nio nacido ms all de 42 semanas de gestacin, cualquiera que sea su peso. El recin nacido postmaduro es el producto de una gestacin postrmino o gestacin prolongada que presenta signos de postmadurez.

Signos ms comunes:
Peso normal o disminuido y talla aumentada. Piel seca y descamada, puede ser apergaminada. Uas largas y quebradizas. Manos tumefactas con descamacin precoz e intensa. Abundante cabello. Lanugo y vrmix caseoso ausentes. Depsitos de grasos disminuidos. Coloracin verde, marrn o amarilla de la piel y uas por impregnacin de meconio. Actitud vivas y facies despierta.

BIBLIOGRAFA:

Garca-Monroy L., Embarazo Prolongado en: Nuez-Maciel E. et al. Tratado de Ginecologa y Obstetricia, Volumen II Cap.50.Mxico, Ed. Cullar, S.A de C.V; 2001. pp. 687-694. Matute GMM. Inicio del trabajo de parto. En: Nuez-Maciel E. et al. Tratado de Ginecologa y Obstetricia, Volumen I Cap.10. Mxico, Ed. Cullar, S.A de C.V; 2001. pp. 187194. Clinicas Obsttricas y Ginecologicas Vol 1:2002. McGraw-Interamericana. lvarez-Ponce VA., Lugo-Snchez AM., lvarez-Snchez AZ., Muiz-Rizo ME. Comportamiento del embarazo prolongado en el servicio de cuidados perinatales. Rev Cubana Obstet Ginecol 2004.

83.- Una mujer embarazada, puede afectar al feto y hacerlo contraer lesiones importantes durante el embarazo o al salir al exterior (atravesando el canal de parto), s la gestante se encuentra afectada de la siguiente patologa:

a) Tricomonas b) Gardenerella c) Herpes genital d) Gonococos

Herpes genital La prevalencia de herpes simplex genital o tipo 2 (VHS-2) en mujeres embarazadas vara entre 7 y 33% en distintas series. La prevalencia ha experimentado un sostenido aumento durante los ltimos aos. Se estima que aproximadamente 1 a 3% de las mujeres adquiere cada ao la infeccin. En el caso de parejas discordantes, la tasa de adquisicin aumenta hasta 10 a 30% anual. La adquisicin durante el embarazo es ~2%. La transmisin al hijo ocurre principalmente cuando la mujer embarazada adquiere una infeccin primaria. La transmisin es de 30 a 50% cuando la infeccin primaria ocurre cerca del momento del parto. La mayor transmisin (85%) ocurre durante el parto. Sin embargo, tambin puede ocurrir transmisin intrauterina (5-8%) y post-natal (8-10%). Los factores que inciden en la transmisin son: infeccin primaria mucho mayor eficiencia que infeccin recurrente, parejas discordantes, ttulos de anticuerpos maternos y procedimientos obsttricos invasores, (los que estn absolutamente contraindicados). Las manifestaciones en la mujer embarazada son principalmente bajo la forma de herpes genital localizado, muy raramente ocurre diseminacin cutnea y visceral, situacin de elevada mortalidad (50%). La infeccin en el nio, si ocurre en las primeras 20 semanas del embarazo, puede provocar aborto en 25%, malformaciones cerebrales, cicatrices, corioretinitis, RCIU. Si ocurre despus de las 20 semanas, puede causar parto prematuro, RCIU, o herpes neonatal. Esta condicin clnica tiene tres formas de presentacin, las dos primeras de elevada mortalidad y secuelas: Herpes diseminado y encefalitis herptica o infeccin localizada en piel, ojo y boca.

Referencias Bibliogrficas:
Pass R, Weber T, Whitley RJ. Herpesvirus infections in pregnancy. Recommendations from the International Herpes Management Forum. Management Strategies Workshop and 7th Annual Meeting.

Whitley R J. Varicella - Zoster virus. Mandell, Douglas and Bennett's Principles and Practice of Infectious Diseases. Mandell G, Bennett J, Dolin R, editors. Fifth edition, 2000 Churchill Livingstone, Philadelphia, pp: 1586-98. Abarca K, Cohen B J, Vial P. Seroprevalence of parvovirus B19 in urban Chilean children and young adults, 1990 and 1996. Epidemiol Infect 2002; 128: 59-62.

84.- Femenino de 23 aos de edad, refiere que desde hace un par de meses ha presentado hemorragia irregular o postcoital, actualmente presenta disuria y dolor abdominal usted debe sospechar en cervicitis por:

a) Chlamydia b) Micoplasma c) Gardnerella d) Candida

Segn los datos de la Organizacin Mundial de la Salud, anualmente se detectan 89 000 000 de nuevas infecciones por Chlamydia trachomatis en el mundo. Esta infeccin provoca uretritis y cervicitis, y las secuelas incluyen enfermedad inflamatoria plvica, embarazo ectpico, infertilidad por dao tubrico, epididimitis, proctitis y artritis reactiva. Se considera principalmente un problema de salud en la mujer, en ella las manifestaciones y consecuencias son ms dainas para la salud reproductiva. Los individuos infectados con Chlamydia trachomatis pueden portar el microorganismo por meses o aos y transmitir la enfermedad a sus parejas sexuales. Su diagnstico sigue siendo un reto, ya que quienes la padecen presentan sntomas muy leves o son portadores asintomticos.

Manifestaciones clnicas:

La cervicitis es la manifestacin clnica ms frecuente de la infeccin por C. trachomatis en la mujer. Sin embargo, el 70% de las mujeres infectadas no tienen sntomas, mientras que en el tercio restante las evidencias clnicas son poco especficas de infeccin, como flujo genital, dolor abdominal o pelviano, sangrado y/o disuria.

La presencia de disuria puede indicar una uretritis acompaante, lo que sucede en el 35% de los casos. En otras oportunidades, solo la uretra est comprometida, y la infeccin uretral se manifiesta como piuria o disuria con cultivo negativo (23% de los casos). El diagnstico se realiza al examinar el hisopado endocervical, que muestra flujo amarillento o verdoso con ms de 10 PMN por campo de inmersin en el examen de Gram. Este resultado define la cervicitis mucopurulenta (CMP) la cual tambin puede ser producida en casos de infeccin por gonococo o mixta (C. trachomatis y gonococo). Por lo tanto, el diagnstico debe confirmarse mediante estudios de mayor especificidad, como las tcnicas moleculares (test de ligasa, PCR), que tienen una sensibilidad del 96% aproximadamente, o la deteccin del antgeno por tcnica de ELISA, con una sensibilidad del 75%. Tambin se ha demostrado que las tcnicas moleculares en el primer chorro de orina son especficas y altamente sensibles.

Cuatro de cada diez mujeres con cervicitis no tratada adquieren enfermedad inflamatoria pelviana (EPI), con mayor riesgo de sufrir embarazo ectpico, infertilidad y dolor crnico pelviano. El riesgo de infertilidad se eleva segn el nmero e intensidad de los episodios: alrededor del 10% despus de un episodio, del 30% despus de dos, y mayor del 50% si ha habido tres o ms episodios. Por otro lado, el embarazo ectpico es cinco a siete veces ms frecuente cuando se trata de pacientes con antecedentes de EPI.

Tratamiento
CLAMIDIA TRACHOMATIS

Azitromicina 1 g VO dosis nica Doxiciclina 100 mg VO cada 12 horas por 7 das Eritromicina 500 mg VO cada 6 horas por 7 das Ofloxacina 300 mg VO cada 12 horas por 7das Levofloxacina 500 mg VO cada 24 horas por 7 das

Basado en Guas Clnicas para el manejo de las ITS, OMS, 2003 / Norma Oficial Mexicana NOM -039-SSA2-2002.

Berek J. (2002) Ginecologa de NOVAK. Mxico. Ed. Mc Graw Hill Interamericana. Pag 293.

85.- El estudio donde se toman un grupo de personas sanas que se clasifican en subgrupos segn su exposicin a una causa o enfermedad se conoce como:

a) b) c) d)

Cohorte Prospectivo Observacional Doble ciego

En los estudios de cohorte se eligen dos grupos uno de expuesto y otro de no expuestos los cuales son seguidos a travs del tiempo para detectar las posibles consecuencias. Posteriormente se analiza la informacin calculado la incidencia en el grupo de expuestos y en el grupo de no expuestos; y una vez obtenidos se calcula el Riesgo Relativo.

Ruiz M. A. Epidemiologa Clnica, Panamericana, 1 . Ed. 2004; pgs: 287- 289.

86.- Femenino de 50 aos de edad, radicada en la Ciudad de Mxico refiere rubor, tumefaccin y rigidez de las articulaciones interfalngicas dstales desde hace tres meses, pero no presenta otras molestias articulares. Cul de los siguientes diagnsticos es ms probable? a) Osteoartritis erosiva b) Artritis reumatoide c) Espondilitis anquilosante d) Esclerodermia

Allen R. M. MMS Medicina Interna. 5. Edicin. National Medical Series. Mc. Graw Hill. 2006. (captulo 10 V E 1). Tpicamente, la osteoartritis erosiva afecta las articulaciones interfalngicas dstales en mujeres de edad madura. Es improbable que esos sntomas articulares dstales prominentes sucedan en pacientes con artritis reumatoide o con lupus eritematoso diseminado sin molestias articulares ms generalizadas. No hay pruebas que indiquen espondilitis anquilosante o esclerodermia.

87.- Acude a consulta un hombre de 35 aos de edad, soltero, previamente sano. Sus familiares refieren que en los 5 das previos ha presentado fiebre, cefalea, cambios del estado de nimo, y en el ltimo da, somnolencia que se ha convertido en estupor. A la exploracin fsica no encuentra alteraciones. Realiza una puncin lumbar y encuentra como nica alteracin del lquido cerebroespinal la presencia de eritrocitos. Se solicit una resonancia magntica de crneo. Usted esperara encontrar para apoyar su diagnstico: a) b) c) d) Afeccin de ganglios basales. Afeccin de lbulo frontal. Afeccin de lbulo temporal. Lesin intraaxial ocupativa en puente.

El cuadro clnico y los hallazgos del lquido cerebroespinal sugieren fuertemente infeccin por virus del herpes simple. Los cambios en el estudio de imagen ms caractersticos, sobre todo en resonancia magntica, se observan en los lbulos temporales. Kasper DL, Braunwald E, Fauci AS, Hauser SL, Longo DL, Jameson JL. Harrisons Principles of Internal Medicine. McGraw Hill. 16 Ed. 2471-2490 p.

88.- Paciente masculino de 58 aos de edad con antecedentes de DM tipo II e HTAS, tratado con amlodipino y hipoglucemiantes orales (glimeprida). Hace unas 2 semanas present un cuadro de lumbalgia por el que recibi tratamiento con diclofenaco. Actualmente tratatado con TMP SMZ por IVU. Acude a urgencias del hospital por presentar desde hace 48 horas una erupcin cutnea confluyente en tronco y extremidades. A la exploracin se observan lesiones maculares eritematoviolceas muy extensas (>70% de la superficie cutnea) sobre las cuales aparecen ampollas y erosiones. Existe afectacin (lesiones erosivo-costrosas) de mucosa labial, oral y conjuntiva. El signo de Nikolsky es positivo. El diagnstico probable es:

a) Penfigoide ampolloso. b) Pnfigo vulgar. c) Pnfigo paraneoplsico. d) Necrolisis txica epidrmica.

La necrlisis epidrmica txica (NET) y el sndrome de Stevens-Johnson (SSJ) constituyen un espectro de la misma enfermedad, compartiendo aspectos etiolgicos, patogenticos, histolgicos y teraputicos. Casi todos (si no todos) los casos son inducidos por frmacos. NET y SSJ pueden distinguirse clnicamente del eritema multiforme, que debe considerarse una enfermedad distinta. La patogenia de la NET y del SSJ es poco conocida, pero se acepta que intervienen reacciones inmunolgicas y un mecanismo final de apoptosis masiva de queratinocitos epidrmicos. El tratamiento consiste en la retirada del frmaco causal y medidas de soporte, evitando la administracin de corticosteroides. Se han descrito tratamientos que pretenden detener la evolucin del cuadro, entre ellos ciclofosfamida, ciclosporina, plasmafresis, pentoxifilina e inmunoglobulinas i.v. El cuadro tpico de NET, descrito por Lyell (8), se caracteriza por la aparicin brusca, tras un prdromo catarral, de lesiones eritematosas, dolorosas, inicialmente distribuidas de forma simtrica en cara y tronco, aunque luego se puedan extender hacia las partes acras. Las lesiones comienzan siendo mculas eritematosas, pero pronto aparece el signo caracterstico de la enfermedad: la necrosis y desprendimiento de la epidermis que produce ampollas flcidas y signo de Nikolsky con despegamiento de amplias zonas epidrmicas que dejan erosiones exudativas. La extensin de este despegamiento es uno de los factores pronsticos principales. Un pequeo porcentaje de enfermos presenta slo eritema confluente y zonas erosivas, pero el 90% tienen adems lesiones aisladas, salpicadas en la proximidad de las erosiones. stas son mculas de borde mal definido, con forma irregular, y pueden tener un centro ms oscuro o ampolloso. Casi todos los pacientes presentan lesiones mucosas, incluyendo erosiones dolorosas orales y farngeas, lesiones oculares (que conllevan un riesgo elevado de secuelas) y genitales. El dao de otros epitelios, como el respiratorio o digestivo, y las complicaciones de la insuficiencia cutnea aguda agravan el cuadro. Signo de NIKOLSKY: Desprendimiento de las capas de la piel, aparentemente sana, por efecto de la presin tangencial del dedo, con una ventosa o con un esparadrapo.

Referencias Bibliogrficas:
1. Avakian R, Flowers FP, Arajo OE, Ramos-Caro FA. Toxic epidermal necrolysis: a review. J Am Acad Dermatol 1991; 25:69-79. 2. Roujeau J-C, Chosidow O, Saiag P, Guillaume J-C. Toxic epidermal necrolysis (Lyell syndrome). J Am Acad Dermatol 1990;23:1039-58. 3. Champion RH. Disorders of blood vessels. En: Champion RH, Burton JL, Ebling FJG, eds. Textbook of dermatology, 5.a ed. Oxford: Blackwell Scientific Publications; 1992. p. 18.3418.38. 4. Fritsch PO, Elias PM. Erythema multiforme and toxic epidermal necrolysis. En: Fitzpatrick TB, Eisen AZ, Wolff K, Freedberg IM, Austen KF, eds. Dermatology in general medicine, 4.a ed. Nueva York: McGraw-Hill; 1993. p. 585-600. 5. Bttiger LE, Strandberg I, Westernholm B. Drug induced febrile mucocutaneous syndrome. Acta Med Scand 1975; 198:226-33.

89.- Masculino de 47 aos con datos positivos de alcoholismo crnico, al cual se le diagnstica pancreatitis aguda Cul es la primera medida teraputica a adoptar? a) Iniciar antibiticos de amplio espectro. b) Administracin de inhibidores de la bomba de protones. c) Suspensin de la va oral. d) Aspiracin nasogstrica.

El 80 % de los pacientes con pancreatitis aguda se tratan mediante medidas de sostn, suspensin de la va oral, hidratacin intravenosa, analgsicos y alimentacin parenteral cuando la va oral se restringe ms all de la semana.

La indicacin de la suspensin de la va oral se basa en el dolor y la intolerancia digestiva. No debe ser prolongada y se debe restablecer secuencialmente luego de 48 horas sin dolor. La alimentacin parenteral no tiene ningn sentido si el restablecimiento de la va oral se realiza dentro de los primeros 7 das. De no ser as se sabe que esta patologa grave provoca una agresin severa que determina un estado hpercatablico por lo tanto debe implementarse soporte nutricional para evitar la desnutricin y las complicaciones que ella trae consigo (alteracin de la modulacin de la respuesta inflamatoria, translocacin bacteriana, inmuno supresin, etc.)

El 60 % de esta enfermedad presenta hipermetabolismo (1,5 veces el metabolismo basal), el 40 % son normo o hipometablicos. Segn la Conferencia de Consenso de Nutricin de la Agresin (Francia 1998), los triglicridos no son contraindicados a menos que el paciente presente una hipertrigliceridemia importante. La necesidad de aporte debe ser de 0,25 a 0,30 g/Kg. La suplementacin de micronutrientes fundamentalmente antioxidantes (vitaminas A, C, E y selenio) y zinc estn indicadas. La suplementacin en base a glutamina, inmunomoduladores, nuevas emulsiones lipdicas en base a aceite de oliva necesitan aun ser confirmadas para la pancreatitis severa. La va enteral se debe privilegiar ya que no solo es mas fisiolgica sino que tambin presenta menor costo y complicaciones habiendo actualmente estudios que muestran un menor ndice de morbimortalidad en estos pacientes ya que la integridad de la barrera intestinal limitara la sobreinfeccin bacteriana pero fundamentalmente la fngica (Kalfarentzos 97, Windsor 98, Pupelis 2000)

Referencias Bibliogrficas: 1. Banks PA, Freeman ML; Practice Parameters Committee of the American College of Gastroenterology. Practice guidelines in acute pancreatitis. Am J Gastroenterol. 2006 Oct; 101(10):2379-400.

2. Frossard JL, Steer ML, Pastor CM. Acute pancreatitis. Lancet. 2008;371:143-152.

90. - A 55-year-old woman presents to the Emergency Department with 8 hours of severe left lower quadrant abdominal pain, focal peritonitis, and guarding on physical examination. She has nonspecific abdominal radiographs and a leukocytosis. Which of the following is indicated at this point?

a) b) c) d)

IV antibiotics. CT scan. Colonoscopy. Surgery.

ABSCESO DIVERTICULAR: La formacin de un absceso diverticular complicado depende de la capacidad de los tejidos periclicos de controlar (localizar) la diseminacin del proceso inflamatorio. En general, los abscesos intra-abdominales se forman por: Fuga anastomtica = 35% Enfermedad diverticular = 23%

Signos/Sntomas fiebre+/- leucocitosis a pesar de antibiticos adecuados, tumoracin dolorosa.

Tratamiento:
Absceso periclico pequeo - 90% responde a los antibiticos y manejo conservador. Drenaje percutneo de los abscesos (DPA) es el tratamiento de eleccin para las colecciones simples, bien definidas. 100% de los abscesos uniloculares simples se resolvieron con DPA y antibiticoterapia. Schauer P, Ramos P, Ghiatas A, Sirinek K. Virulent diverticular disease in young obese men.Am J Surg;164:443-8. Pubmed-Medline.

91.- Femenino de 66 aos que acude al servicio de neurologa para valoracin, al realizar exploracin fsica, usted determina que ste, comprende pero no puede articular palabra, Que tipo de afasia tiene?

a) Wernike b) Broca c) Global d) Anmica

Afasia:
De broca, motriz o de expresin (no fluente) De Wernicke (no comprende) De conduccin (no denomina no repite) Anmica (no denomina) Transcortical motora (no fluente) Transcortical sensitiva (no denomina no comprende) Global (no fluente-no denomina-no repite-no comprende) Talmicas y putaminales (lenguaje espontaneo incomprensible)

Referencia Bibliogrfica:
1. Aronson A y cols. Examen clnico neurolgico, 3 Edicin. La Prensa Mdica Mexicana, Mxico, 1995. 2. Uribe CS, Arana A, Pombo PL. Neurologa, 5 Edicin. Corporacin para investigaciones biolgicas. Colombia, 1996. 3.-Adams R, Vctor M. Principles of Neurology. Mc Graw Hill. 7th ed. USA 2001. 4.-Bradley W.G. Neurology in clinical practice. Butterworth Heinemann. 4th ed. Philadelphia USA, 2004. 5.-Silberstein SD, Lipton RB, Goadsby PJ. Headache in clinical practice. Isis Medical Media. Oxford University Press, UK, 1998. 6.-Barinagarrementera F. Cant Interamericana. Mxico D.F. 1997. C. Enfermedad vascular cerebral. McGrawHill

92.- Femenino de 19 aos con 7 episodios al mes de cefalea hemicraneal, de un da de duracin, pulstil, acompaada de vmitos, fotofobia y sonofobia y con examen fsico normal. El tratamiento preventivo de eleccin, entre los siguientes, es:

a) Carbonato de litio. b) Ergotamina. c) Propranolol. d) Sumatriptn.

Profilaxis de Migraa Debe ser considerada en aquellos pacientes que tienen 2-3 ataques en el mes o en aquellos que tienen 1 ataque pero prolongado. La profilaxis puede ser necesaria 6-9 meses e incluso, en forma definitiva. Los principales agentes utilizados son los Beta bloqueadores adrenrgicos Propanolol, Atenolol... (Excepto asmticos, insuficiencia cardaca, diabetes). En un estudio controlado se demostr una reduccin de migraa en relacin con placebo de 76%. Bloqueadores de los canales del Calcio Nifedipino Diltiazem... Vasodilatadores, Fluranizina.

REFERENCIAS BIBLIOGRAFICAS: 1. 2. 3. 4. 5. Lewis PJ Neurology 1997 48 (6) 1542-50. Mathew NT Neurol Clin. 1997; 15:61-83. Goadsby PJ Neurol Clin 1997; 15 (1):27-42. MLTfeld-Hansen P Neurol Clin 1997; 15: 153-165. Diamond S., Diamond ML Postgrad Med. 1997; 10: 169-172.

93.- Femenino de 62 aos portadora de DM tipo 2 de 10 aos de evolucin tratada con hipoglucemiantes orales actualmente en descontrol. Se enva al servicio de oftalmologa por probable retinopata. La primera lesin que se observa en la retinopata diabtica temprana es:

a) b) c) d)

Microaneurismas Exudados blandos Exudados duros Hemorragias intrarretinianas

La retinopata diabtica tiene cuatro etapas:

1. Retinopata no proliferativa ligera. Esta es la etapa ms temprana de la enfermedad en la que aparecen los microaneurismas. Estas son pequeas reas de inflamacin, que parecen ampollas, en los pequeos vasos sanguneos de la retina. 2. Retinopata no proliferativa moderada. Segn avanza la enfermedad, algunos vasos sanguneos que alimentan la retina se obstruyen. 3. Retinopata no proliferativa severa. En esta etapa muchos ms vasos sanguneos se bloquean, haciendo que varias partes de la retina dejen de recibir sangre. Entonces estas reas de la retina envan seales al cuerpo para que haga crecer nuevos vasos sanguneos. 4. Retinopata proliferativa. En esta etapa avanzada, las seales enviadas por la retina para alimentarse causan el crecimiento de nuevos vasos sanguneos. Esto se llama la retinopata proliferativa. Estos nuevos vasos sanguneos son anormales y frgiles. Crecen a lo largo de la retina y de la superficie del gel vtreo, el gel incoloro que llena el interior del ojo.

Retinopata diabtica. Forma leve de retinopata diabtica, en la que destacan microaneurismas /microhemorragias de predominio en arcada temporal superior, junto con exudados duros que se acercan a la mcula.

Referencia Bibliogrfica:
1. The Diabetes Control and Compications Trial Research Grop. The Effect of Intensive treatment of Diabetes on the development and progression of long term complications in insulin dependent Diabetes Mellitus N Engl J. Med; 1993; 329: 977-986. 2. Early treatment Diabetic Retinopathy Study Research Group Early photocoagulation for diabetic Retinopathy ETDRS Report 9. Ophthalmology, 1991; 98: 1316-26. 3. Kahn HA Hiller R. Blindness caused by diabetic retinopathy Am. J. Ophthalmol, 1974; 78; 58-67. 4. Ladas ID. Theossiadis GA Long term efectiveness of modified grid photocoagulation for diffuse Macular edema Acta Ophthalmol, 1993; 71(3); 393-7. 5. Klein R, Klein BEK , Moss SE , et al The Eisconsin Epidemiology Group Ophthalmology, 1984 ; 91 : 1464-74.

94.- Se trata de masculino de 61 aos, fumador de 10 cigarrillos diarios, historia de tos y expectoracin matutina habitual, consulta por disnea de mnimos esfuerzos y ortopnea de dos almohadas. Exploracin fsica: TA 180/100 mmHg, presin venosa normal, auscultacin pulmonar con crepitantes bibasales, auscultacin cardaca rtmica a 120 lpm con soplo sistlico eyectivo I/VI en foco artico y tercer ruido. ECG: Ritmo sinusal y criterios de hipertrofia ventricular izquierda. El diagnstico ms probable es:

a) b) c) d)

Cardiopata isqumica con disfuncin sistlica. Cardiopata hipertensiva en insuficiencia cardaca. Cor pulmonale crnico. Insuficiencia cardaca congestiva en paciente con EPOC.

La cardiopata hipertensiva constituye la complicacin principal de la hipertensin arterial, pues es la primera causa de morbi-mortalidad del paciente hipertenso. En la hipertensin arterial la composicin histolgica del ventrculo izquierdo se altera globalmente, resultando lesiones que afectan a los propios cardiomiocitos, al intersticio miocrdico y a la pared de las arterias intramiocrdicas. En el origen del desarrollo de esas lesiones participan tanto la sobrecarga mecnica de la pared ventricular impuesta por la presin arterial elevada, como factores humorales sistmicos y locales que actan directamente sobre el parnquima y los vasos miocrdicos, por ejemplo, la angiotensina II. Las consecuencias funcionales de las lesiones estructurales miocrdicas son diversas, aunque la ms representativa es la que tiene que ver con el desarrollo de insuficiencia cardiaca congestiva La insuficiencia cardiaca (IC) es un sndrome clnico complejo en el que los pacientes presentan sntomas tpicos de IC (disnea en reposo o con esfuerzo), signos tpicos de IC (taquicardia, taquipnea, estertores pulmonares, derrame pleural, elevacin de la presin yugular venosa, edema perifrico, hepatomegalia) y evidencia objetiva de una anomala estructural o funcional del corazn en reposo (cardiomegalia, tercer tono, soplos cardacos, anomalas electrocardiogrficas, concentraciones elevadas de pptidos natriurticos) (ESC, 2008).

Cardiopata hipertensiva MARIO BENDERSKY*, DANIEL PISKORZ#, DANIEL BOCCARDO. * Profesor de Farmacologa, Facultad de Ciencias Mdicas, Universidad Nacional de Crdoba. Unidad de Hipertensin Arterial. Instituto Modelo de Cardiologa de Crdoba. # Instituto de Cardiologa del Sanatorio Britnico de Rosario. Escuela de Cardiologa, Universidad Catlica de Crdoba. Departamento de Tcnicas No Invasivas y Arritmias, Instituto Modelo de Cardiologa de Crdoba.

Direccin postal: Instituto Modelo de Cardiologa. Av. Sagrada Familia 359. 5003 Crdoba. Argentina. Sanatorio Britnico de Rosario. Paraguay 40. 2000 Rosario. Pcia. de Santa Fe. Argentina.

95.- Al referirnos al concepto de estereotipia como sntoma psiquitrico nos referimos a la psicopatologa de: a) b) c) d) La orientacin. La psicomotricidad. El pensamiento. La memoria.

La estereotipia motora consiste en una o varias secuencias de movimiento de carcter reiterativo, que no configura un programa de accin especfico.

Est presente en trastornos psiquitricos y neurolgicos. Puede ser secundaria a frmacos antipsicticos (diskinesia tarda o al uso abusivo de sustancias psicoestimulantes como la cocana o anfetamina (punding). Cuando se trata de la reiteracin insistente de pensamientos, acompaada de una viscosidad en el desplazamiento atencional y rigidez ideativa, se constituye la forma clnica cognitiva, que puede o no acompaar a la motora. Esta perseveracin cognitiva es un signo deficitario o negativo del propio cuadro clnico de la esquizofrenia. Sin embargo el uso al largo plazo de agentes antipsicticos tpicos puede generar el sndrome deficitario por neurolpticos que incrementa el fenmeno de la perseverancia cognitiva y causa un dficit motor simultneo.

En el trastorno obsesivo compulsivo la perseveracin cognitiva se presenta bajo la forma compulsiva de reiteracin de pensamientos, que puede generar rituales motores. El trastorno de Tourette tambin cursa con estereotipias, ya sean los tics motores o vocales. La enfermedad de Parkinson o la corea de Huntington tambin pueden presentar diskinesia como expresin de eventos de neuroplasticidad. En el Parkinson pueden presentarse en el curso del tratamiento con L-Dopa, y en el Huntington forman parte del cuadro clnico coreo-atetsico. Las estereotipias motoras y cognitivas pueden ser desencadenadas por aumento del tono dopaminrgico en el estriado, y pueden ser neutralizadas por agentes bloqueantes dopaminrgicos que actan en el estriado. Asimismo pueden ser generadas por la disminucin del tono dopaminrgico en el estriado. Ejemplos de la estimulacin dopaminrgica en el estriado lo constituyen sustancias de abuso como la anfetamina o cocana, frmacos como L-dopa, apomorfina, enfermedades como la corea de Huntington y el T. de Tourette.

Ejemplos de bloqueo dopaminrgico en el estriado lo constituyen los neurolpticos tpicos usados al largo plazo que son capaces de generar diskinesia tarda.

Referencias Bibliogrficas:
American Psychiatric Association. Diagnostic and Statistical Manual of Mental Disorders. 4th ed. Washington, DC: American Psychiatric Association; 1994. Albert, M.S., Levkoff, S.E. & Reilly, C.H. (1992): The delirium symptom interview. An interview for the detection of delirium in hospitalized patients. Journal of Geriatric Psychiatry and Neurology, 5, 14-21. American Psychiatric Association (2002). Diagnostic and statistical manual of mental disorders (DSM-IV-TR). Washington, D.C.: American Psychiatric Association. Belsky J:K. (1996): Psicologa intervenciones. MASSON, S.A. del envejecimiento: Teora, investigaciones e

Berg R, Franzen M., Wedding D. (1990): Exploracin del deterioro orgnico cerebral. Biblioteca de Psicologa. Editorial Desclee de Brouwer, S.A., Bilbao. Berggren D, Gustafson Y, Eriksson B et al. (1987): Postoperative confusion after anesthesia in the elderly patients with femoral neck fractures. Anesth Analg; 66: 497504. Breitbat W, Rosenfeld B, Roth F et al. (1997): The memorial delirium assessment scale. J Pain Symptom Manage; 13: 128-137.

96. - Femenino de 54 aos, que presenta un trastorno de la marcha junto con parestesias de ambas extremidades inferiores. Exploracin Fsica: Sensibilidad vibratoria abolida. Existe sospecha clnica de un dficit de cobalaminas a pesar de que sus niveles plasmticos son de 250pg/mL (normal: 200-900pg/mL). El examen de laboratorio que confirma el diagnostico de esta paciente es: a) Repetir la cuantificacin de cobalaminas sricas junto a la de folatos intracelulares. b) Cuantificar la homocistena total plasmtica. c) Realizar un test de Schilling. d) Cuantificar el cido metil-malnico y la homocistena total plasmticas.

Diagnstico del dficit de vitamina B12

El dficit de vitamina B12 se basa en demostrar niveles bajos de la misma en sangre, menores de 200 pg/mL. Deben de demostrarse estos niveles descendidos en al menos dos ocasiones separadas 1. La presencia de niveles elevados de homocisteina (mayores de 13 mol/L) o cido metil-malnico (mas de 0.4 mol/L) asociados a una nica determinacin baja de vitamina B12 en plasma (menor de 200 pg/mL) en ausencia de dficit de flico, vitamina B6 o insuficiencia renal (ya que pueden estar elevados los niveles de homocisteina y metilmalnico por una disminucin de su eliminacin) es otro criterio diagnstico (tabla III)1. Existen otras causas de elevacin de homocisteina y cido metil-malnico diferentes al dficit de flico o B12 (tabla IV), por lo que no es recomendable realizar el diagnstico de anemia megaloblstica con niveles elevados de estos metabolitos sino se demuestra simultneamente una disminucin de B12 y/o flico 3. Algunos autores sugieren que se puede establecer el diagnstico de dficit de B12 con cifras normales de esta vitamina en sangre si se demuestra una elevacin concomitante de homocisteina o cido metilmalnico y se descartan otras causas (dficit de flico, de B6 e insuficiencia renal) aunque este criterio no es admitido por todos. Anemias carenciales II: Anemia megaloblstica y otras anemias carenciales Javier Bilbao Garay. Internista, Mdico Adjunto. Servicio de Medicina Interna. Hospital Universitario Puerta de Hierro. Madrid. Sistema Nacional de Salud. Volumen 30, N 3/2006.

97.- Masculino de 4 aos de edad con antecedentes de otitis media a los doce meses de edad, actualmente cursa con otalgia, otorrea e hipoacusia, acompaado de malestar general. A la exploracin en conducto izquierdo se aprecia Hiperemia timpnica, hipervascularizacin y engrosamiento timpnico. El tratamiento de eleccin en esta patologa es:

a) Antihistamnicos de forma exclusiva. b) Solo medidas generales. c) Drenaje, AINES y esteroides locales. d) Antibiticos, antihistamnicos y AINES.

Otitis media que se acompaa con secrecin y signos locales, sistmicos, o ambos. Instituto Nacional de la Nutricin. Salvador Zubiran Manual de teraputica mdica y procedimientos de urgencias. Cuarta edicin. Pag. 443 450. Mac Graw-Hill Interamericana. Mxico.

La otitis media se define como la inflamacin del odo medio que se acompaa de secrecin y signos locales, sistmicos, o ambos, de enfermedad aguda. Esta enfermedad predomina en la infancia, sin embargo tambin existe en el adulto. Su distribucin tiene una clara periodicidad estacional, sobre todo en otoo e invierno. Los agentes de mayor prevalencia en nuestro pas son: Streptococcus pneumoniae, Haemophilus influenzae, Morexella catarrhalis estreptococo del grupo A y S. aureuse. Los antibiticos ms aceptados en estos casos son la amoxicilina, la amoxicilina con clavulanato, la calritromicina y el trimetropim con slfametoxazol. Se acepta el uso de antihistamnicos y antiinflamatorios no esteroideos para disminuir la congestin de la mucosa para resolver la obstruccin de la trompa farngotimpnica; en realidad no se ha demostrado efecto sobre la duracin de los sntomas. El drenaje del derrame solo esta indicado cuando ste persiste por ms de tres semanas.

98.- La patologa que asocia distensin de los espacios areos respiratorios distales a los bronquolos terminales acompaada de destruccin de los tabiques alveolares:

a) Bronquitis crnica b) Enfisema c) Asma d) Asbestosis

Un enfisema se define en trminos anatomopatolgicos por el agrandamiento permanente de los espacios areos distales a los bronquiolos respiratorios, con destruccin de la pared alveolar, con o sin fibrosis manifiesta.1 Es una enfermedad crnica comprendida junto con la bronquitis crnica en la Enfermedad Pulmonar Obstructiva Crnica (EPOC).2 El nombre viene del griego emphysema que significa "soplar el aire" o "insuflar'.3

Fisiopatologa del enfisema


Le enfermedad por sus efectos incapacita y debilita enormemente la calidad de vida del paciente, sometindolo a una vida restringida y sedentaria. El deterioro progresivo de su funcin pulmonar, desencadena no slo cambios fsicos en el paciente, sino tambin alteraciones a nivel psicolgico.

El enfisematoso, se ve ceido a realizar mnimos esfuerzos. Cualquier tipo de actividad fsica, se convierte en verdadero sufrimiento y evita realizarlas. Las ms elementales necesidades fisiolgicas, requieren de enormes esfuerzos y gastos extras de energa. El enfisema es una enfermedad crnica, progresiva, caracterizada por un agrandamiento anormal y permanente de los espacios areos distales al bronquiolo terminal, acompaado de destruccin de sus paredes sin fibrosis (8) La caracterstica ms relevante de la enfermedad, es su limitacin al flujo areo durante los movimientos respiratorios. Esta limitacin, medido en valores del volumen espiratorio forzado al primer segundo (VEF 1) por debajo de los valores predictivos del paciente (< 1,0 l), ocasiona una gran morbilidad, con deterioro severo de su capacidad vital (CV). El 25% de la resistencia total pulmonar al flujo areo, ocurre normalmente en las vas areas < 3mm, pero estos valores se aproximan al 80% en los pacientes enfisematosos (9,10). Este flujo, esta determinado por el juego entre presin retroceso pulmonar elstico. Como el volumen pulmonar durante la expiracin disminuye, ocurre cierre prematuro de las vas areas perifricas, debido a la prdida de elasticidad pulmonar y a la destruccin de los alvolos que se encuentran fijados a las delgadas paredes de los bronquiolos. Al final, estos pulmones enfisematosos son de mayores dimensiones a lo normal. Ello se traduce en disminucin del VEF 1, y otros flujos (FEF 25-75, FEF 50), aumento del volumen residual (VR) de la capacidad residual funcional (CRF) y de la capacidad pulmonar total (CPT), as como, una disminucin a la prueba de difusin al monxido de carbono (DLCO). El atrapamiento de aire en las zonas enfisematosas, se traduce en hallazgos radiolgicos tpicos; un pulmn hiperaireado, con escasa vasculatura pulmonar, aplanamiento de los diafragmas y un trax ms alargado. Al examen fsico, vemos un trax con dimensiones mayores en el dimetro antero posterior y escaso desarrollo de la masa muscular. Hay hipersonoridad a la percusin y los sonidos pulmonares se encuentran muy disminuidos. Todos estos cambios en la funcin pulmonar, ocasionan desigualdad en la relacin ventilacin / perfusin (V/Q), resultado de un incremento de espacio muerto, hipoxemia sola en fases tempranas, acompaadas de hipercapnia en las fases terminales. Basndose en estos datos, la indicacin quirrgica no slo se limita al pulmn enfisematoso puro, sino tambin, al pulmn con enfermedad bulosa (11-16).

Referencias Bibliogrficas: 1. Gordon LS. Emphysema: The first two centuries and beyond. A historial overview, with suggestions for future research: Part I. Am Rev Respir Dis 1992; 146:13341344.

2. Gaast A, Molard-Dietmenn A, Pelletier A, Pauli G, Bieth JG. The antielastase screen of the lower respiratory tract of alpha I-proteinase inhibitor suficiente patients with emphysema or pneumotorax. Am. Rev. Respir Dis. 1990; 141:880-883.

3. Janoff A. Elastases and emphysema, current assesment of the ProteaseAntiprotease hypothesis. Am Rev Respir Dis 1985;417-433. 4. Silverman EK, Speizer FE. Risk factors for the development of chronic pulmonary disease. Med Clin North Amer 1996;80:501-522. 5. Deslaries J. A perspective on the role of surgery in chronic obstructive lung disease. Chest Surg Clin North Amer 1995;5:575-602. 6. Brantigan OC, Mller E. Surgical treatment of pulmonary emphysema. Am Surg 1957;23:789-804. 7. Cooper JD, Trulock EP, Triantafillou AN, Patterson GA, Pohl MS, Doloney PA, et al. Bilateral pneumonectomy (volume reduction) for chronic pulmonary disease. J T Horac. Cardiovasc Surg 1995,109:106-119.

99.- Masculino de un ao de edad que acude al servicio con antecedentes de aumento de temperatura hasta de 39.5 oC de dos das de evolucin controlada con paracetamol, el da de hoy presenta datos de exantema. Se realiza diagnstico de rosola ya que el exantema se caracteriza por ser: a) b) c) d) Mculopapular violceo, que no desaparece al dgito presin. Mculopapular eritemotoso, con descamacin. Mculopapular eritemotoso, de inicio sbito al desaparecer la fiebre. Mculas, vesculas y costras, pruriginoso.

Exantema sbito
Manifestaciones clnicas: Conocida tambin como "rosola infantum" o "fiebre de los tres das", comienza con fiebre alta - en ocasiones hasta de 40,5C - siendo caracterstico en estos nios que conservan un relativo buen estado general, pese a la fiebre. No es infrecuente que consulten en el servicio de urgencia por convulsiones febriles. Actualmente se sabe que el virus tiene tropismo hacia el sistema nervioso central y las convulsiones pueden representar un efecto directo del virus sobre el tejido cerebral. Despus de 3 a 4 das de fiebre, aparece el exantema que es macular o papular y se palpa granuloso; es de color rosado plido y no es pruriginoso. Se observa primero en el tronco, cuello y en la regin retroauricular; su generalizacin es infrecuente. El exantema dura alrededor de 2 a 3 das. Agente etiolgico: Virus herpes tipo 6 (HHV6), virus ADN, miembro de la familia Herpesviridae. Epidemiologa: Afecta a los lactantes entre los 6 a 18 meses de vida, y su mecanismo de transmisin se presume por la va respiratoria y contacto directo con algn husped que est excretando el virus. El perodo de incubacin es de 9 a 10 das.

Diagnstico: Se hace por el cuadro clnico y la exclusin de otras enfermedades febriles agudas. Existen estudios serolgicos especficos an en evaluacin para diagnstico de infeccin aguda. Aislamiento del paciente hospitalizado: Precauciones estndar. Tratamiento: Manejo sintomtico de la fiebre y eventualmente de episodios convulsivos.

Bibliografa:
Gonzlez-Saldaa N, Macas-Parra M. Vacunas en pediatra. 1er ed. Mxico. McGraw-Hill Interamericana; 1998. Comittee on Infectious Diseases 1997-2000. Red Book 2000, 25 ed. American Academy of pediatrics; 2000.

100.- Femenino de 27 aos, acude al servicio de consulta externa refiriendo presentar desde hace varias semanas temblor fino distal, sensacin de angustia, sudoracin palmar, ha perdido peso en los ltimos meses. Sin antecedentes de importancia. Exploracin fsica: T-A 130/86 mm hg, peso 54 kg., talla 160 cm., exoftalmos, sudoracin palmar, piel hmeda y caliente, FC 110 lpm, abdomen con ruidos peristlticos incrementados, con aumento en el numero de evacuaciones. El tratamiento de primera eleccin para esta paciente es: a) b) c) d) Yodo 131. Yoduro. Metimazole. Tiroidectoma.

El bocio txico difuso (BTD) constituye la forma ms frecuente de hiperfuncin de la glndula tiroidea (70 % de los casos), que puede aparecer a cualquier edad, aunque por lo general aparece entre la tercera y cuarta dcada de la vida. Esta enfermedad es ms frecuente en la mujer, donde se observa un predominio de 7:1 en relacin con los hombres en regiones no bocigenas. Esta relacin se reduce en las zonas de bocio endmico. Los factores genticos desempean un papel esencial en la etiologa y existe una predisposicin familiar a esta enfermedad de Graves-Basedow.

El BTD se caracteriza por la presencia de hipertiroidismo, bocio difuso y elstico, oftalmopata, dermopata, acropaquia tiroidea y onicolisis. Es importante el diagnstico y tratamiento precoz del hipertiroidismo para evitar complicaciones, principalmente las cardiovasculares.

TABLA I 1. Piel: Piel fina, caliente y sudorosa. Prurito. Pelo fino y frgil. Onicolisis. 2. Sistema cardiovascular: Taquicardia, palpitaciones y fibrilacin auricular. Insuficiencia cardiaca, angor pectoris, disnea de esfuerzo, vasodilatacin. Disminucin de la respuesta a la digital. 3. Aparato digestivo: Hiperdefecacin. Disfuncin heptica: hipertransaminasemia. 4. Aparato locomotor: Debilidad y atrofia de la musculatura proximal. Osteoporosis. Aumento de la maduracin sea en nios. Hiperreflexia, temblor distal, mioclonias. 5. Sistema nervioso: Irritabilidad, nerviosismo e insomnio. Psicosis, hipercinesia. 6. Otros: Prdida de peso a pesar de la polifagia. Intolerancia al calor. Alteraciones menstruales y disminucin de la fertilidad en mujeres.

Pruebas de funcin tiroidea: a) Determinacin de hormonas tiroideas: L-tiroxina (T4) que circula en plasma unida en su mayora a la protena transportadora (TBG), y menos del 0,1% libre. Aunque slo esta pequea porcin est libre, es la concentracin de T4 libre ms que la T4 total la que indica la actividad tiroidea; por lo tanto, en la mayora de los casos slo es necesario determinar la T4 libre, que estar elevada en los casos de hipertiroidismo. L-triyodotironina (T3): se produce por la desyodacin perifrica de T4; es regulada por factores independientes de la funcin tiroidea. En algunos pacientes hipertiroideos la concentracin de T3 est elevada cuando no lo est la de T4.

TSH, producida por clulas de la adenohipfisis, que controla la funcin tiroidea por accin directa positiva y que es controlada a su vez por la TRH hipotalmica con efecto positivo estimulador. En los casos de hipertiroidismo primario la TSH estar inhibida por el sistema de retroalimentacin debido al exceso de hormonas tiroideas perifricas (T4 y T3).

La TSH tiene una mayor sensibilidad para el diagnstico del hipertiroidismo que la T4 libre. Sin embargo, no es del todo especfica, es decir, una TSH baja no siempre indica hipertiroidismo. TRH sintetizada a nivel hipotalmico. No se suele determinar en la prctica clnica. Existen algunos casos en los que la concentracin de T3 yT4 se encuentran en el lmite superior de la normalidad y persiste la sospecha de hipertiroidismo. En estos casos la administracin de TRH no produce ningn in cremento en los niveles de TSH en los hipertiroidismos primarios (test de TRH para TSH). As pues, para el diagnstico de un hipertiroidismo utilizaremos bsicamente los niveles de TSH y T4 libre.

TRATAMIENTO:
Drogas antitiroideas metimazol 15 a 75 mg/da propiltiouracilo 150 a 750 mg/da durante 18 a 24 meses 40 a 60% de recadas I 131 Ciruga Sintomtico: bloqueo.

Bibliografa:
Hipertiroidismo: Clnica, diagnstico y tratamiento. I. M. RECHE MOLINA, B. VALERA, C. HIDALGO, L. LEN, G. PIDROLA.

Servicios de Medicina Interna y Endocrinologa. Hospital Universitario Virgen de las Nieves. Granada.

Anda mungkin juga menyukai